100% found this document useful (1 vote)
134 views97 pages

A03 Abs. Value & Inequalities

The document discusses absolute value and inequalities. It covers topics such as absolute value expressions and equations, distance on the number line, solving absolute value inequalities and compound inequalities. Various examples are provided to illustrate absolute value concepts and solving absolute value equations and inequalities.

Uploaded by

Azizmanva
Copyright
© © All Rights Reserved
We take content rights seriously. If you suspect this is your content, claim it here.
Available Formats
Download as PDF, TXT or read online on Scribd
100% found this document useful (1 vote)
134 views97 pages

A03 Abs. Value & Inequalities

The document discusses absolute value and inequalities. It covers topics such as absolute value expressions and equations, distance on the number line, solving absolute value inequalities and compound inequalities. Various examples are provided to illustrate absolute value concepts and solving absolute value equations and inequalities.

Uploaded by

Azizmanva
Copyright
© © All Rights Reserved
We take content rights seriously. If you suspect this is your content, claim it here.
Available Formats
Download as PDF, TXT or read online on Scribd
You are on page 1/ 97

INEQUALITIES

30 MARCH 2024
REVISION: 2752

AZIZ MANVA
[email protected]
Get all the files at: https://bit.ly/azizhandouts
Aziz Manva ([email protected])

TABLE OF CONTENTS
TABLE OF CONTENTS ................................. 2 2. ABSOLUTE VALUE TOPICS ................. 61
1. ABSOLUTE VALUE & INEQUALITIES .... 3 2.1 Quadratics 61
2.2 Logarithms 68
1.1 Absolute Value 3 2.3 Trigonometry 71
1.2 Quadratic Absolute Value 13
1.3 Number Line 13 3. “NAMED” INEQUALITIES .................... 79
1.4 Set Operations on Intervals 22
3.1 Some Basics 79
1.5 One Step Inequalities 28
3.2 𝑹𝑴𝑺 ≥ 𝑨𝑴 ≥ 𝑮𝑴 ≥ 𝑯𝑴 (In)equality 81
1.6 Two Step Inequalities 37
3.3 Further Topics 97
1.7 Compound Inequalities 42
1.8 Absolute Value Inequalities 46

P a g e 2 | 97
Get all the files at: https://bit.ly/azizhandouts
Aziz Manva ([email protected])

1. ABSOLUTE VALUE & INEQUALITIES


1.1 Absolute Value
A. Definition

1.1: Absolute Value


The value of a number without regard to its sign is called its absolute value.

𝑥, 𝑥 ≥ 0
|𝑥| = {
−𝑥, 𝑥 < 0

Example 1.2
1
Find the exact value of 𝑦 = |7| + |−3| + |−3 4| + |−𝜋|
1 1
|7| = 7, |−3| = 3, |−3 | = 3 , |−𝜋| = 𝜋
4 4
1 1
𝑦 = 7 + 3 + 3 + 𝜋 = 13 + 𝜋
4 4

1.3: Property
|𝑥| = |−𝑥|

|7 − 5| = 2 = |5 − 7|

Example 1.4
Given that 𝑒 is an irrational number with approximate 2.71, and 𝜋 is an irrational number with approximate
value 3.14, find the exact value of |𝑒 − 𝜋|

|𝑒 − 𝜋| = −(𝑒 − 𝜋) = 𝜋 − 𝑒

Example 1.5
Find the value of |𝑥 − 𝑦| if:
A. 𝑥 and 𝑦 are positive integers such that 𝑥 > 𝑦
B. 𝑥 and 𝑦 are positive integers such that 𝑥 < 𝑦
C. 𝑥 and 𝑦 are negative integers such that 𝑥 > 𝑦
D. 𝑥 and 𝑦 are negative integers such that 𝑥 < 𝑦

𝑃𝑎𝑟𝑡 𝐴 & 𝑃𝑎𝑟𝑡 𝐶: |𝑥 − 𝑦| = 𝑥 − 𝑦


𝑃𝑎𝑟𝑡 𝐵 & 𝑃𝑎𝑟𝑡 𝐷: |𝑥 − 𝑦| = 𝑦 − 𝑥

𝑥 = −3, 𝑦 = −5 ⇒ −3 − (−5) = −3 + 5 = 2 = +𝑣𝑒


𝑥 = −3, 𝑦 = −2 ⇒ −3 − (−2) = −3 + 2 = −1 = −𝑣𝑒
B. Expressions

Example 1.6: Calculating Values by Substitution


Prove that

Example 1.7: Simplification

P a g e 3 | 97
Get all the files at: https://bit.ly/azizhandouts
Aziz Manva ([email protected])

Prove that

Example 1.8
Nesting means absolute value inside absolute value. Like brackets (or parentheses), these are solved inside out.
Start from the innermost expression, and keep simplifying.
Simplify
A. |−3 + |2 − 3|| = |−3 + 1| = |−2| = 2

B. |1 − |2 − |3 − |4 − |5 − |6 − 7||||||

1 2 1 1 3 1 1
|− + |− + || = |− + | = |− | =
3 3 4 3 12 12 12

|1 − |2 − |3 − |4 − |5 − 1||||| = |1 − |2 − |3 − |4 − 4|||| = |1 − |2 − |3 − 0||| = |1 − |−1|| = 0

C. Distance Interpretation

1.9: Distance Interpretation


Distance between 𝑥 and 𝑦 on the Number Line:
= 𝐴𝑏𝑠𝑜𝑙𝑢𝑡𝑒 𝑉𝑎𝑙𝑢𝑒 𝑜𝑓 𝐷𝑖𝑓𝑓𝑒𝑟𝑒𝑛𝑐𝑒 𝑜𝑓 𝑥 𝑎𝑛𝑑 𝑦

We can convert this into a formula by writing:


𝐷𝑖𝑠𝑡𝑎𝑛𝑐𝑒 = |𝑥 − 𝑦| = |𝑦 − 𝑥|
➢ The order of the two numbers is not important, since we take the absolute value. Both the expressions
give the same answer.
➢ The absolute value is important because distance can never be negative.

Example 1.10
Find the distance between 3 and −4 on the number line.

Graphical Method
Draw a number line.

Count the distance from 0 to 3, which is


3 𝑈𝑛𝑖𝑡𝑠 𝑟𝑖𝑔ℎ𝑡 𝑓𝑟𝑜𝑚 𝑧𝑒𝑟𝑜
Count the distance from 0 to -4, which is:
4 𝑈𝑛𝑖𝑡𝑠 𝑙𝑒𝑓𝑡 𝑓𝑟𝑜𝑚 𝑧𝑒𝑟𝑜

And hence, the total distance is:


3+4=7

Another calculate the same distance is to calculate the distance of zero from both points.
Total Distance = 𝐷(3,0) and 𝐷(0, −4) = 3 + 4 = 7

P a g e 4 | 97
Get all the files at: https://bit.ly/azizhandouts
Aziz Manva ([email protected])

Algebraic Method
Distance between two points on the Number Line
= 𝐴𝑏𝑠𝑜𝑙𝑢𝑡𝑒 𝑉𝑎𝑙𝑢𝑒 𝑜𝑓 𝐷𝑖𝑓𝑓𝑒𝑟𝑒𝑛𝑐𝑒 𝑜𝑓 𝑡ℎ𝑒 𝑡𝑤𝑜 𝑛𝑢𝑚𝑏𝑒𝑟𝑠
= | 3 − (−4)| = |3 + 4| = |7| = 7

Equivalently, we can also do:


Distance = |−4 − 3| = |−7| = 7

Example 1.11
In each part, find the distance between the numbers on the number line:
A. 1,3
B. −1,3
C. −1, −3
D. 1, −3

Part A
We want to find the distance between 1 and 3 on the
number line (graphed alongside).

Substitute 𝑥 = 1, 𝑦 = 3 in |𝑥 − 𝑦|
|3 − 1| = |2| = 2
Substitute 𝑥 = 1, 𝑦 = 3 in |𝑦 − 𝑥|
|1 − 3| = |−2| = 2

Part A
Substitute 𝑥 = −1, 𝑦 = 3 in |𝑥 − 𝑦|
|−1 − 3| = |−4| = 4

Substitute 𝑥 = −1, 𝑦 = 3 in |𝑦 − 𝑥|
|3 − (−1)| = |3 + 1| = 4
Part C
Substitute 𝑥 = −1, 𝑦 = −3 in |𝑥 − 𝑦|
|−1 − (−3)| = |−1 + 3| = |2| = 2

Part D
Substitute 𝑥 = 1, 𝑦 = −3 in |𝑥 − 𝑦|
|1 − (−3)| = |1 + 3| = 4

Example 1.12
Find the distance between the given points on the number line:
A. 2,5
B. −3, 7
C. 4, −5
D. −1, −8
1 1
E. 2 , 3
2 1
F. 3
,−5
1 1
G. −4,3

P a g e 5 | 97
Get all the files at: https://bit.ly/azizhandouts
Aziz Manva ([email protected])
1 1
H. − 3 , − 5

1 1 3 2 1 1
| − |=| − |=| |=
2 3 6 6 6 6
2 1 10 3 13 13
| − (− )| = | + | = | | =
3 5 15 15 15 15
1 1 3 4 7 7
|− − | = |− − | = |− | =
4 3 12 12 12 12

Example 1.13: Horizontal and Vertical Distance on the Co-ordinate Plane


Given points 𝐴(3,4), 𝐵(3, 9) and 𝐶(5,4). Find the sum of the distances from 𝐴 to 𝐵, and 𝐴 to 𝐶.

𝐷(𝐴, 𝐵) = |4 − 9| = |−5| = 5
𝐷(𝐴, 𝐶) = |3 − 5| = |−2| = 2
𝐷(𝐴, 𝐵) + 𝐷(𝐴, 𝐶) = 5 + 2

Example 1.14: Creating Expressions


A. The number 𝑥 is 2 units away from 1 on the number line. Draw a number line and use it to find the
possible values of 𝑥. Use the formula for distance on the number to get an absolute value expression for
𝑥.
B. The maximum number of guests that I expect at my party is 6. The minimum number of guests that I
expect is 2. Represent the max, min and average number of guests on a number line. Use the average
that you found to create an absolute value expression that gives the maximum and minimum number of
guests.

Part A

Distance on the number line between two points 𝑥 and 𝑦 is given by


|𝑥 − 𝑦|
Substitute 𝑦 = 1 in the above expression:
|𝑥 − 1| = 2
Part B
6+2 8
𝐴𝑣𝑒𝑟𝑎𝑔𝑒 = = =4
2 2

|𝑔 − 4| = 2
D. Equations

1.15: Removing the absolute value sign


Every time, you remove an absolute value sign, you must solve two separate equations.
|𝒙| = 𝒏 ⇔ 𝒙 = ±𝒏
For example, we can say that:
|𝑥| = 6 ⇔ 𝑥 = ±6

Example 1.16

P a g e 6 | 97
Get all the files at: https://bit.ly/azizhandouts
Aziz Manva ([email protected])

Solve the following equations algebraically. Also solve them graphically.


A. |𝑥| = 5
3
B. |𝑎| = 4
C. |𝑏| = 0.4
D. |𝑐| = 0
E. |𝑒| = −1

Part A
Algebraically
|𝑥| = 5 ⇒ 𝑥 = ±5 ⇒ 𝑥 ∈ {−5, +5}
Graphically
The absolute value of a number represents its distance from the origin. Hence, we want numbers that are a
distance of 5 units away from the origin.
And this is given by:

Part E
Algebraically
The absolute value of a number can never be negative.
Hence, the equation has no solutions.

Graphically
Absolute value represents the distance from the origin.
And distance can never be negative.

Hence, the equation has no solution.

Example 1.17
The absolute value of a number is double of the number itself. Identify the number.

|𝑥| = 2𝑥
The only number that will satisfy is
𝑥=0

Example 1.18: Absolute Value Expressions


If you have an expression inside the absolute value sign, the strategy remains the same.
Solve the equation. Check your answers by substituting the values.
Integers
A. |𝑥 + 3| = 4
B. |𝑎 + 2| = 5
C. |𝑦 + 1| = 4
D. |𝑏 − 2| = 4
E. |𝑐 + 3| = 10
F. |𝑧 − 3| = 5
Decimals
G. |𝑠 + 1.2| = 5.4
H. |𝑡 − 0.3| = 0.7

P a g e 7 | 97
Get all the files at: https://bit.ly/azizhandouts
Aziz Manva ([email protected])

Fractions
1 3
I. |𝑝 + 2| = 4
2 1
J. |𝑠 − 3| = 5

Integers
𝑥 + 3 = ±4 ⇒ 𝑥 = ±4 − 3 ⇒ 𝑥 ⏟
∈ {−7,1}
𝒃𝒆𝒍𝒐𝒏𝒈𝒔 𝒕𝒐
Substitute the values to check:
𝑥 = −7 ⇒ 𝐿𝐻𝑆 = |𝑥 + 3| = |−7 + 3| = |−4| = 4 = 𝑅𝐻𝑆

𝑥 = 1 ⇒ 𝐿𝐻𝑆 = |1 + 3| = |1 + 3| = |4| = 4 = 𝑅𝐻𝑆


|𝑎 + 2| = 5 ⇒ 𝑎 + 2 = ±5 ⇒ 𝑎 ± 5 − 2 ⇒ 𝑎 ∈ {−7,3}
|𝑦 + 1| = 4 ⇒ 𝑦 + 1 = ±4 ⇒ 𝑦 = ±4 − 1 ⇒ 𝑦 ∈ {, −5,3}
|𝑏 − 2| = 4 ⇒
𝑦 + 1 = ±4 ⇒ 𝑦 = ±4 − 1 ⇒ 𝑦 = {3, −5}
Decimals
𝑠 + 1.2 = ±5.4 ⇒ 𝑠 ∈ {−6.6,4.2}
Fractions

Example 1.19: Isolating Absolute Values


In solving linear equations, we isolate the variable to find its value. Similarly, if we are able to isolate the
absolute value on one side of the equation, then we can apply the above property.

1 1
Find the values of z that satisfy |𝑧 − 2| − 3 = 0

1 1 1 1 1 5
|𝑧 − | = ⇒ 𝑧 − = ± ⇒ 𝑧 ∈ { , }
2 3 2 3 6 6

Example 1.20: Variable Terms outside the Absolute Value


The equations we have seen so far have had the variable inside the absolute value. The next example shows how
to handle variables outside the absolute value sign. The general idea remains the same: Isolate the absolute
value expression on one side of the equation.

Solve
A. 𝑥 + |𝑥 + 1| = 3
B. 2𝑥 + |3𝑥 − 5| = 4

Part A
Isolate the absolute value sign:
|𝑥 + 1| = 3 − 𝑥
Apply the property |𝑥| = 𝑎 ⇒ 𝑥 = ±𝑎:
𝑥 + 1 = ±(3 − 𝑥) ⇒ 𝑥 = ±(3 − 𝑥) − 1
Case I: Positive Sign
𝑥 = 3 − 𝑥 − 1 ⇒ 𝑥 = 1 ⇒ 𝑾𝒐𝒓𝒌𝒔
Case II: Negative Sign
𝑥 = −(3 − 𝑥) − 1 ⇒ 𝑥 = −3 + 𝑥 − 1 ⇒ 𝟎 = −𝟒 ⇒ 𝒙 = 𝝓(𝐍𝐨 𝐬𝐨𝐥𝐮𝐭𝐢𝐨𝐧𝐬)
Hence, the final answer is:

P a g e 8 | 97
Get all the files at: https://bit.ly/azizhandouts
Aziz Manva ([email protected])

𝑥=1
Checking for Validity of Solutions
In these kinds of questions, it is important to go back and check by substituting the candidate solutions in the
original equation to see if they satisfy it.
In the question, we got two candidate solutions, but only one satisfied the original equation.

Part B

|3𝑥 − 5| = 4 − 2𝑥 ⇒ 3𝑥 − 5 = ±(4 − 2𝑥)


Case I
9
3𝑥 − 5 = 4 − 2𝑥 ⇒ 5𝑥 = 9 ⇒ 𝑥 =
5
Case II
3𝑥 − 5 = −(4 − 2𝑥) ⇒ 3𝑥 − 5 = −4 + 2𝑥 ⇒ 𝑥 = 1

Example 1.21: Two Absolute Value Expressions


Put one absolute value expression on one side, and the other absolute expression on the other side

Consider the equation


|𝑦| = |𝑥|
A. Consider the four points:(2,2), (−2,2), (2, −2), (−2, −2) and show that these points satisfy the equation.
B. Consider the four points:(𝑛, 𝑛), (−𝑛, 𝑛), (𝑛, −𝑛), (−𝑛, −𝑛) and show that these points satisfy the
equation.
C. Hence, graph the equation. Break the graph of the equation into two linear graphs, and write the
equations of those graphs.

Part A

𝑥 𝑦 |𝑥| |𝑦|
2 2 2 2
−2 2 2 2
2 −2 2 2
−2 −2 2 2

Part B

We can break the solution into two parts:


𝑦 = 𝑥 𝑂𝑅 𝑦 = −𝑥
And the above solutions can be combined into a single solution by writing
𝑦 = ±𝑥

P a g e 9 | 97
Get all the files at: https://bit.ly/azizhandouts
Aziz Manva ([email protected])

|𝒚| = |𝒙| ⇔ 𝒚 = ±𝒙

Example 1.22
Solve
A. |2𝑥 + 4| = |3𝑥 − 5|
B. |3𝑥 + 2| = |5𝑥 − 7|
3 5 2 5
C. |2 𝑥 + 7| = |5 𝑥 − 3|

Part A
𝐶𝑎𝑠𝑒 𝐼: 2𝑥 + 4 = +(3𝑥 − 5) ⇒ 𝑥 = 9
1
𝐶𝑎𝑠𝑒 𝐼𝐼: 2𝑥 + 4 = −(3𝑥 − 5) ⇒ 5𝑥 = 1 ⇒ 𝑥 =
5
Part B
9
𝐶𝑎𝑠𝑒 𝐼: 3𝑥 + 2 = 5𝑥 − 7 ⇒ 2𝑥 = 9 ⇒ 𝑥 =
2
5
𝐶𝑎𝑠𝑒 𝐼𝐼: 3𝑥 + 2 = −5𝑥 + 7 ⇒ 8𝑥 = 5 ⇒ 𝑥 =
8
Part C
Case I: Take the positive sign
3 5 2 5
𝑥 + = +( 𝑥 − )
2 7 5 3
Collect like terms:
3 2 5 5
𝑥− 𝑥=− −
2 5 3 7
Take the LCM:
15 4 35 15
𝑥− 𝑥=− −
10 10 21 21
Simplify:
11 50
𝑥=−
10 21
11
Divide both sides 10:
50 11 50 10 500
𝑥=− ÷ =− × =−
21 10 21 11 231
Case II: Take the negative sign
3 5 2 5
𝑥 + = −( 𝑥 − )
2 7 5 3
3 5 2 5
𝑥+ =− 𝑥+
2 7 5 3
3 2 5 5
𝑥+ 𝑥= −
2 5 3 7
15 4 35 15
𝑥+ 𝑥= −
10 10 21 21
19 20
𝑥=+
10 21
20 10 200
𝑥= × =
21 19 399

P a g e 10 | 97
Get all the files at: https://bit.ly/azizhandouts
Aziz Manva ([email protected])

E. No Solutions and Infinite Solutions


When the coefficients of 𝑥 on both sides of an equation are Variable
the same, we get: Coefficients
➢ No Solutions if the constants are different
➢ Infinite Solutions if the constants are the same
Same Different
Example 1.23
Solve Constants Constants One
|𝑥 + 4| − |𝑥 − 3| = 0 Same Different Solution
Rewrite this so the LHS has one absolute expression, and the
RHS has one absolute value expression: Infinite No
|𝑥 + 4| = |𝑥 − 3| Solutions Solutions
Use the property that |𝑦| = |𝑥| ⇔ 𝑦 = ±𝑥:
𝑥 + 4 = ±(𝑥 − 3)
Case I: Take the positive sign
𝑥 + 4 = +(𝑥 − 3) ⇒ 𝑥 + 4 = 𝑥 − 3 ⇒ +4 = −3 ⇒ 𝑵𝒐 𝑺𝒐𝒍𝒖𝒕𝒊𝒐𝒏𝒔 ⇒ 𝒙 = 𝝓
Case II: Take the negative sign
1
𝑥 + 4 = −(𝑥 − 3) ⇒ 𝑥 + 4 = −𝑥 + 3 ⇒ 𝑥 = − ⇒ 𝑾𝒐𝒓𝒌𝒔
2

Example 1.24
Solve
|2𝑥 + 4| − |−2𝑥 − 4| = 0

|2𝑥 + 4| = |−2𝑥 − 4|
Use the property that |𝑦| = |𝑥| ⇔ 𝑦 = ±𝑥:
2𝑥 + 4 = ±(−2𝑥 − 4)
Case I: Take the positive sign
2𝑥 + 4 = +(−2𝑥 − 4) ⇒ 2𝑥 + 4 = −2𝑥 − 4 ⇒ 4𝑥 = −8 ⇒ 𝑥 = −2 ⇒ 𝑾𝒐𝒓𝒌𝒔
Case II: Take the negative sign
2𝑥 + 4 = −(−2𝑥 − 4) ⇒ 2𝑥 + 4 = 2𝑥 + 4 ⇒ 4 = 4 ⇒ 𝑰𝒏𝒇𝒊𝒏𝒊𝒕𝒆 𝑺𝒐𝒍𝒖𝒕𝒊𝒐𝒏𝒔 ⇒ 𝒙 ∈ ℝ

Example 1.25
|𝑥 + 5| = 𝑥 + 5

Case I: 𝑥 + 5 is +𝑣𝑒 or zero:


𝑥 + 5 = 𝑥 + 5 ⇒ 𝐼𝑛𝑓𝑖𝑛𝑖𝑡𝑒 𝑆𝑜𝑙𝑢𝑡𝑖𝑜𝑛𝑠
𝑥 + 5 ≥ 0 ⇒ 𝑥 ≥ −5
Case I: 𝑥 + 5 is −𝑣𝑒 or zero:
−(𝑥 + 5) = 𝑥 + 5 ⇒ −𝑥 − 5 = 𝑥 + 5 ⇒ −10 = 2𝑥 ⇒ 𝑥 = −5
𝑥 + 5 ≤ 0 ⇒ 𝑥 ≤ −5

Hence, the final solution is the interval:


𝑥 ≥ −5
F. Nested Absolute Value Equations
We have already seen nested value expressions. These are solved inside out. However, nested value equations
are solved outside in.

P a g e 11 | 97
Get all the files at: https://bit.ly/azizhandouts
Aziz Manva ([email protected])

We remove the outermost absolute value sign, to get two equations. Since, these equations are themselves
absolute value equations, they need to be further solved, each with two cases.

Example 1.26
Solve
|𝑥 + |2𝑥 + 2|| = 5

Remove the Outermost sign


Use the property that |𝑦| = 𝑘 ⇒ 𝑦 = ±𝑘
𝑥 + |2𝑥 + 2| = ±5
Take the Positive Sign
|2𝑥 + 2| = 5 − 𝑥 ⇒ 2𝑥 + 2 = ±(5 − 𝑥)
Case IA: 2𝑥 + 2 = 5 − 𝑥 ⇒ 𝑥 = 𝟏
CaseIB: 2𝑥 + 2 = 𝑥 − 5 ⇒ 𝑥 = −𝟕
Take the Negative Sign
𝑥 + |2𝑥 + 2| = −5 ⇒ |2𝑥 + 2| = −5 − 𝑥
𝟕
CaseIIA: 2𝑥 + 2 = −5 − 𝑥 ⇒ 𝑥 = −
𝟑
CaseIIB: 2𝑥 + 2 = 5 + 𝑥 ⇒ 𝑥 = 𝟑

𝑭𝒊𝒏𝒂𝒍 𝑺𝒐𝒍𝒖𝒕𝒊𝒐𝒏: 𝒙 = {𝟏, −𝟕}


G. Prime Solutions

Example 1.27
If sixteen is subtracted from the square of an integer, the absolute value of the difference is a prime.
𝐹𝑖𝑛𝑑 𝑎𝑙𝑙
⏟ such numbers.
𝑹𝒆𝒒𝒖𝒊𝒓𝒆𝒔 𝒑𝒓𝒐𝒐𝒇 𝒕𝒉𝒂𝒕 𝒕𝒉𝒆𝒔𝒆
𝒂𝒓𝒆 𝒕𝒉𝒆 𝒐𝒏𝒍𝒚 𝒔𝒐𝒍𝒖𝒕𝒊𝒐𝒏𝒔

Set up the equation


|𝑥 2 − 16| = 𝑝, 𝑝 ∈ 𝑃𝑟𝑖𝑚𝑒 𝑁𝑢𝑚𝑏𝑒𝑟𝑠
Factoring the LHS
Factor the LHS as a difference of squares:
|𝑥 2 − 16| = 𝑝 ⇒ |(𝑥 + 4)(𝑥 − 4)| = 𝑝 ⇒ (𝑥 + 4)(𝑥 − 4) = ±𝑝

Now, the RHS is a prime number, and can only be factored as shown in the table below:

−𝑝 +𝑝
(𝑥 + 4) −1 1 𝑝 −𝑝 1 𝑝
(𝑥 − 4) 𝑝 −𝑝 −1 1 𝑝 1
Case I II III IV V VI
Case I, IV and V
In all situations, 𝑥 + 4 is greater than 𝑥 − 4.

A negative number cannot be more than a positive number, but this is precisely what happens in Case I and Case
IV:
𝐶𝑎𝑠𝑒 𝐼: −1⏟ > ⏟ 𝑝 ⇒ 𝑪𝒐𝒏𝒕𝒓𝒂𝒅𝒊𝒄𝒕𝒊𝒐𝒏
𝑁𝑒𝑔𝑎𝑡𝑖𝑣𝑒 𝑃𝑜𝑠𝑖𝑡𝑖𝑣𝑒
𝑁𝑢𝑚𝑏𝑒𝑟 𝑃𝑟𝑖𝑚𝑒

P a g e 12 | 97
Get all the files at: https://bit.ly/azizhandouts
Aziz Manva ([email protected])

𝐶𝑎𝑠𝑒 𝐼𝑉: −𝑝
⏟ > ⏟
1 ⇒ 𝑪𝒐𝒏𝒕𝒓𝒂𝒅𝒊𝒄𝒕𝒊𝒐𝒏
𝑁𝑒𝑔𝑎𝑡𝑖𝑣𝑒 𝑃𝑜𝑠𝑖𝑡𝑖𝑣𝑒
𝑃𝑟𝑖𝑚𝑒 𝐼𝑛𝑡𝑒𝑔𝑒𝑟
In Case V, the smallest value that 𝑝 can take is 2, but this is more than 1:
𝐶𝑎𝑠𝑒 V: 1 > 𝑝 ⇒ 𝑪𝒐𝒏𝒕𝒓𝒂𝒅𝒊𝒄𝒕𝒊𝒐𝒏

In general, if we have
𝑥 + 4 > 𝑥 − 4 ⇒ 𝑪𝒐𝒏𝒕𝒓𝒂𝒅𝒊𝒄𝒕𝒊𝒐𝒏
Hence, we do not obtain any solutions from Cases I, IV and V.
Case VI
This case gives an integer solution, but the difference is not prime. So, it is not valid:
𝑥 − 4 = 1 ⇒ 𝑥 = 5 ⇒ 𝑥 2 − 16 = 9 ⇒ 𝑵𝒐𝒕 𝑷𝒓𝒊𝒎𝒆 ⇒ 𝑵𝒐𝒕 𝑽𝒂𝒍𝒊𝒅
Case II and III
These are the only two cases that result in valid solutions:
𝑥 + 4 = 1 ⇒ 𝑥 = −3 ⇒ 𝑥 2 − 16 = −7 ⇒ |𝑥 2 − 16| = 𝟕| ⇒ 𝑽𝒂𝒍𝒊𝒅
𝑥 − 4 = −1 ⇒ 𝑥 = 3 ⇒ 𝑥 2 − 16 = −7 ⇒ |𝑥 2 − 16| = 𝟕| ⇒ 𝑽𝒂𝒍𝒊𝒅
𝑭𝒊𝒏𝒂𝒍 𝑺𝒐𝒍𝒖𝒕𝒊𝒐𝒏 𝑺𝒆𝒕 = {−𝟑, 𝟑}

1.2 Quadratic Absolute Value


A. Graphs
B. Expressions
C. Equations
D. Range

1.3 Number Line


A. Representing Inequalities on a Number Line
We now look at representing inequalities on a number line. This is a key skill that is needed for inequality
questions.

1.28: Inequalities on a Number Line


The region that satisfies the inequality is given a different color.

Example 1.29
Graph the inequalities below on a number line.
A. 𝑥 ≥ 2
B. 𝑥 ≤ 2

1.30: Endpoints
If the endpoints are included, then the circle is filled (as in the example above).

P a g e 13 | 97
Get all the files at: https://bit.ly/azizhandouts
Aziz Manva ([email protected])

If the endpoints are not included, then the circle is empty (as in the example below).

Example 1.31
Graph the inequalities below on a number line.
A. 𝑥 > 2
B. 𝑥 < 2

Example 1.32
Graph the following inequalities:
A. 𝑥 ≥ 3
B. 𝑥 ≤ 3
C. 𝑥 > 3
D. 𝑥 < 3

B. Compound Inequalities and Endpoints


A compound inequality has two inequalities put together. Consider the following inequalities in terms of 𝑛:

1.33: Endpoints
A compound inequality has four cases with respect to its endpoints.

𝑥
⏟< 𝑛 < 𝑦
𝑬𝒏𝒅𝒑𝒐𝒊𝒏𝒕𝒔 𝒏𝒐𝒕 𝒊𝒏𝒄𝒍𝒖𝒅𝒆𝒅
𝑥≤𝑛≤𝑦

𝑬𝒏𝒅𝒑𝒐𝒊𝒏𝒕𝒔 𝒊𝒏𝒄𝒍𝒖𝒅𝒆𝒅
𝑥
⏟≤ 𝑛 < 𝑦
𝑳𝒆𝒇𝒕 𝒆𝒏𝒅𝒑𝒐𝒊𝒏𝒕 𝒊𝒏𝒄𝒍𝒖𝒅𝒆𝒅
𝑹𝒊𝒈𝒉𝒕 𝒆𝒏𝒅𝒑𝒐𝒊𝒏𝒕 𝒆𝒙𝒄𝒍𝒖𝒅𝒆𝒅

P a g e 14 | 97
Get all the files at: https://bit.ly/azizhandouts
Aziz Manva ([email protected])

𝑥<𝑛≤𝑦

𝑳𝒆𝒇𝒕 𝒆𝒏𝒅𝒑𝒐𝒊𝒏𝒕 𝒆𝒙𝒄𝒍𝒖𝒅𝒆𝒅
𝑹𝒊𝒈𝒉𝒕 𝒆𝒏𝒅𝒑𝒐𝒊𝒏𝒕 𝒊𝒏𝒄𝒍𝒖𝒅𝒆𝒅

Example 1.34
Graph the following inequalities:
3<𝑛<5

𝑬𝒏𝒅𝒑𝒐𝒊𝒏𝒕𝒔 𝒏𝒐𝒕 𝒊𝒏𝒄𝒍𝒖𝒅𝒆𝒅

3≤𝑛≤5

𝑬𝒏𝒅𝒑𝒐𝒊𝒏𝒕𝒔 𝒊𝒏𝒄𝒍𝒖𝒅𝒆𝒅

3≤𝑛<5

𝑳𝒆𝒇𝒕 𝒆𝒏𝒅𝒑𝒐𝒊𝒏𝒕 𝒊𝒏𝒄𝒍𝒖𝒅𝒆𝒅
𝑹𝒊𝒈𝒉𝒕 𝒆𝒏𝒅𝒑𝒐𝒊𝒏𝒕 𝒆𝒙𝒄𝒍𝒖𝒅𝒆𝒅

3<𝑛≤5

𝑳𝒆𝒇𝒕 𝒆𝒏𝒅𝒑𝒐𝒊𝒏𝒕 𝒆𝒙𝒄𝒍𝒖𝒅𝒆𝒅
𝑹𝒊𝒈𝒉𝒕 𝒆𝒏𝒅𝒑𝒐𝒊𝒏𝒕 𝒊𝒏𝒄𝒍𝒖𝒅𝒆𝒅

C. Continuous Numbers

1.35: Continuous Numbers


When we represent numbers on an interval on the number line, we are referring to continuous numbers. This
means that all numbers in between are included.

Example 1.36
How many numbers does the number line
alongside represent?

It represents an infinite number of numbers because it includes numbers like


3.1, 3.01, 3.001, …
And there no is limit to which you can take the decimal point.

1.37: Infinite Number of Numbers


P a g e 15 | 97
Get all the files at: https://bit.ly/azizhandouts
Aziz Manva ([email protected])

Between any two numbers on the real number line lie an infinite number of numbers.

Example 1.38
Find some numbers between:
4, 4.001

4.0005, 4.00051,4.000501, ….

Example 1.39 (Calculator Allowed)


√32, √56

√32 ≈ 5.65
√56 ≈ 7.48

6, 6.1, 6.01, 6.001, 6.0001, … ..

Example 1.40 (Calculator Allowed)


3 3
√−3, √−9

3
√−3 ≈ −1.44
3
√−9 ≈ −2.08

−1.7, −1.71, −1.701, −1.7001, …


D. Interval Notation
Interval Notation is an algebraic to write the solution sets that we have been graphing on the number line so far.

1.41: Interval Notation


Interval Notation is written as:
(𝑆𝑡𝑎𝑟𝑡 𝑃𝑜𝑖𝑛𝑡, 𝐸𝑛𝑑 𝑃𝑜𝑖𝑛𝑡)
And it includes all the numbers in between.

For example:
(3,4)
Means all the real numbers between 3 and 4.

1.42: Round Bracket Notation


Round brackets mean that the endpoint is not included.
They are equivalent to empty circles on the number line.

(3,4)
Does not include either 3 or 4, but includes all the numbers in
between.
This is equivalent to two empty circles on the number line, and the
number line version is shown alongside.

Example 1.43
Write (3,4) as an inequality.

P a g e 16 | 97
Get all the files at: https://bit.ly/azizhandouts
Aziz Manva ([email protected])

3<𝑥<4

1.44: Square Bracket Notation


Square brackets mean that the endpoint is included.
They are equivalent to filled circles on the number line.

[3,4]
Includes 3 and 4, and all the numbers in between.
This is equivalent to two empty circles on the number line, and
the number line version is shown alongside.

Example 1.45
Write [3,4] as an inequality.

3≤𝑥≤4

1.46: Summary of Notation

Meaning Number Line Interval Notation Inequality Notation


Endpoint Not Included Empty Circle Round Bracket < 𝑂𝑅 >
Endpoint Included Filled Circle Square Bracket ≤ 𝑂𝑅 ≥

( ⏟
𝑥 , 𝑦 ),
⏟ [ ⏟
𝑥 , 𝑦
⏟ ], ( ⏟
𝑥 , 𝑦 ],
⏟ [ ⏟
𝑥 , 𝑦 )

⏟𝑺𝒕𝒂𝒓𝒕 𝑷𝒐𝒊𝒏𝒕 𝑬𝒏𝒅 𝑷𝒐𝒊𝒏𝒕 ⏟𝑺𝒕𝒂𝒓𝒕 𝑷𝒐𝒊𝒏𝒕 𝑬𝒏𝒅 𝑷𝒐𝒊𝒏𝒕 ⏟𝑺𝒕𝒂𝒓𝒕 𝑷𝒐𝒊𝒏𝒕 𝑬𝒏𝒅 𝑷𝒐𝒊𝒏𝒕 ⏟𝑺𝒕𝒂𝒓𝒕 𝑷𝒐𝒊𝒏𝒕 𝑬𝒏𝒅 𝑷𝒐𝒊𝒏𝒕
𝑬𝒏𝒅𝒑𝒐𝒊𝒏𝒕𝒔 𝒏𝒐𝒕 𝒊𝒏𝒄𝒍𝒖𝒅𝒆𝒅 𝑬𝒏𝒅𝒑𝒐𝒊𝒏𝒕𝒔 𝒂𝒓𝒆 𝒊𝒏𝒄𝒍𝒖𝒅𝒆𝒅 𝑳𝒆𝒇𝒕 𝑬𝒏𝒅𝒑𝒐𝒊𝒏𝒕 𝑬𝒙𝒄𝒍𝒖𝒅𝒆𝒅 𝑳𝒆𝒇𝒕 𝑬𝒏𝒅𝒑𝒐𝒊𝒏𝒕 𝑰𝒏𝒄𝒍𝒖𝒅𝒆𝒅
𝑹𝒊𝒈𝒉𝒕 𝑬𝒏𝒅𝒑𝒐𝒊𝒏𝒕 𝑰𝒏𝒄𝒍𝒖𝒅𝒆𝒅 𝑹𝒊𝒈𝒉𝒕 𝑬𝒏𝒅𝒑𝒐𝒊𝒏𝒕 𝑬𝒙𝒄𝒍𝒖𝒅𝒆𝒅

1.47: Set Notation


The notation
{ }
Stands for a set. It includes only those elements which are specified in the
set, not the elements in between.

{3,4}, which is graphed alongside, includes only the discrete numbers


3 𝑎𝑛𝑑 4

Example 1.48
Graph the inequality below on a number line. And also write it in interval notation.
A. 3<𝑛<5

𝑬𝒏𝒅𝒑𝒐𝒊𝒏𝒕𝒔 𝒏𝒐𝒕 𝒊𝒏𝒄𝒍𝒖𝒅𝒆𝒅
B. 3≤𝑛≤5

𝑬𝒏𝒅𝒑𝒐𝒊𝒏𝒕𝒔 𝒊𝒏𝒄𝒍𝒖𝒅𝒆𝒅
C. 3≤𝑛<5

𝑳𝒆𝒇𝒕 𝒆𝒏𝒅𝒑𝒐𝒊𝒏𝒕 𝒊𝒏𝒄𝒍𝒖𝒅𝒆𝒅
𝑹𝒊𝒈𝒉𝒕 𝒆𝒏𝒅𝒑𝒐𝒊𝒏𝒕 𝒆𝒙𝒄𝒍𝒖𝒅𝒆𝒅
D. 3<𝑛≤5

𝑳𝒆𝒇𝒕 𝒆𝒏𝒅𝒑𝒐𝒊𝒏𝒕 𝒆𝒙𝒄𝒍𝒖𝒅𝒆𝒅
𝑹𝒊𝒈𝒉𝒕 𝒆𝒏𝒅𝒑𝒐𝒊𝒏𝒕 𝒊𝒏𝒄𝒍𝒖𝒅𝒆𝒅

P a g e 17 | 97
Get all the files at: https://bit.ly/azizhandouts
Aziz Manva ([email protected])

Part A
Interval from 3 to 5, but not want the endpoints
(3,5)
Part B
Interval from 3 to 5, with the endpoints:
[3,5]
Part C
Interval from 3 to 5.
➢ Use a square bracket on the left to include
the left endpoint.
➢ Use a round bracket on the right to exclude the right endpoint.
[3,5)

Part D
We want the interval from 3 to 5.
➢ Use a square bracket on the right to include the left endpoint.
➢ Use a round bracket on the left to exclude the right endpoint.
(3,5]

Example 1.49
Write the following inequalities in Interval Notation.
1 3
A. 7
<𝑛<4
1 3
B. ≤𝑛<
7 4
1 3
C. 7
≤𝑛≤ 4
1 3
D. 7
<𝑛≤ 4

1 3
( , )
7 4
1 3
[ , )
7 4
1 3
[ , ]
7 4
1 3
( , ]
7 4
E. Converting from Interval to Inequality Notation

Example 1.50
Write the below in inequality notation:
A. I will meet you at a time 𝑡 after 3 pm, and before 5 pm.
B. I will meet you at a time 𝑡 between 3 and 5 pm.
C. I will meet you at a time 𝑡 from 3 to 5 pm.

𝑆𝑡𝑎𝑡𝑒𝑚𝑒𝑛𝑡 𝐴: 𝑥 > 3 𝐴𝑁𝐷 𝑥 < 5


𝑆𝑡𝑎𝑡𝑒𝑚𝑒𝑛𝑡 𝐵: 3 < 𝑥 < 5
𝑆𝑡𝑎𝑡𝑒𝑚𝑒𝑛𝑡 𝐶: 3 ≤ 𝑥 ≤ 5

Example 1.51

P a g e 18 | 97
Get all the files at: https://bit.ly/azizhandouts
Aziz Manva ([email protected])

Convert the following into inequality notation.


A. (3,5)
B. [3,5)
C. (3,5]
D. [3,5]

𝑥 > 3 𝐴𝑁𝐷 𝑥 < 5 ⇒ 3 < 𝑥 < 5


𝑥 ≥ 3 𝐴𝑁𝐷 𝑥 < 5 ⇒ 3 ≤ 𝑥 < 5
𝑥 > 3 𝐴𝑁𝐷 𝑥 ≤ 5 ⇒ 3 < 𝑥 ≤ 5
𝑥 ≥ 3 𝐴𝑁𝐷 𝑥 ≤ 5 ⇒ 3 ≤ 𝑥 ≤ 5

Example 1.52
Find 𝐴 ∩ 𝐵 given that:
𝐴 = {𝑥: 𝑥 > 3, 𝑥 ∈ ℝ}, 𝐵 = {𝑥: 𝑥 < 5, 𝑥 ∈ ℝ}

(3,5)

In the above question, if 𝑥 ∈ ℝ had not been a part of the question statement, which set of numbers would have
you taken for 𝑥?

You would still have taken real numbers.

Example 1.53
Find 𝐴 ∩ 𝐵 given that:
𝐴 = {𝑥: 𝑥 > 3, 𝑥 ∈ ℤ}, 𝐵 = {𝑥: 𝑥 < 5, 𝑥 ∈ ℤ}

𝐴 = {4,5,6, … }
𝐵 = {… ,2,3,4}
𝐴 ∩ 𝐵 = {4}
F. Concept of Infinity
If we have a number which is greater than three, this is written in inequality notation as
𝑥>3
In interval notation, there is no higher bound for the value of 𝑥, and hence we write:
(3, ∞)
Infinity will always take a round bracket, since no number, howsoever large, will include infinity.

Example 1.54
Write the following inequalities in interval notation.
A. 𝑥 > 5
B. 𝑥 < −3
P a g e 19 | 97
Get all the files at: https://bit.ly/azizhandouts
Aziz Manva ([email protected])

C. 𝑥 ≥ 0
D. 𝑥 ≤ 9

(5, ∞)
(−∞, −3)
[0, ∞)
(−∞, 9]

Example 1.55
Write the following inequalities in interval notation.
A. −2 < 𝑥 < 5
B. −8 < 𝑥 ≤ −3
C. 5 ≤ 𝑥 ≤ 6

(−2,5)
(−8, −3]
[5,6]
G. Types of Inequalities

Sign Meaning Sign Meaning


> Strictly greater than < Strictly less than

≥ 𝑜𝑟 >= Greater than or equal to ≤ 𝑜𝑟 <= Less than or equal to

H. Endpoints
The maximum number of mistakes are made in terms of including or not including endpoints.
The value at the end of the range of an inequality is called its endpoint. If an inequality takes the value at the
end of its range, the endpoint is included. If not, the endpoint is excluded.

Example 1.56
Write the following statements into inequalities.
A. The passing marks(𝑚) in SSC board for the English subject are 35 or more marks. The maximum marks
in the subject are 100. Marks are given in increments of 1.
B. I need more than three hundred balloons(𝑏) for that child’s party.
C. I need more than three hundred rupees(𝑟) in my bank account to not pay a fine. The bank calculates my
bank balance in rupees and paise, but not fractions of a paise.

SSC Board does not give negative marks to students, and hence, the number of marks must be a whole number:
35 ≤ 𝑚 ≤ 100, 𝑚∈𝕎
You cannot have half a balloon in the given context; hence the variable must take a value from the set of
integers:
𝑏 > 300, 𝑏∈ℕ
The money in my bank account can be modelling using a number from the set of Rational Number.
𝑟 > 300, 𝑟∈ℚ

Example 1.57
Write the following statements into interval notation:

P a g e 20 | 97
Get all the files at: https://bit.ly/azizhandouts
Aziz Manva ([email protected])

A. The passing marks(𝑚) in SSC board for the English subject is 35 or more marks. The maximum marks in
the subject are 100.
B. I need more than three hundred balloons(𝑏) for that child’s party.
C. I need more than three hundred rupees(𝑟) in my bank account to not pay a fine.

35 ≤ 𝑚 ≤ 100, 𝑚 ∈ 𝕎 ⇒ 𝑚 ∈ {35,36,37, … ,100}


𝑏 > 300, 𝑏 ∈ ℕ ⇔ {301,302,303, … }
𝑟 > 300, 𝑟 ∈ ℚ ⇔ {300.01,300.02, … }

I. Set Theory Notation


This is the most formal. It uses concepts from set theory to write the solution set as a valid set.

Example 1.58
Write the following statements into interval notation:
A. The passing marks(𝑚) in SSC board for the English subject is 35 or more marks. The maximum marks in
the subject are 100.
B. I need more than three hundred balloons(𝑏) for that child’s party.
C. I need more than three hundred rupees(𝑟) in my bank account to not pay a fine.

{𝑚 ⏟| 35 ≤ 𝑚 ≤ 100, 𝑚 ∈𝕎}
𝒔𝒖𝒄𝒉 𝒕𝒉𝒂𝒕

{𝑏 ⏟| 𝑏 > 300, 𝑏 ∈ℕ}


𝒔𝒖𝒄𝒉 𝒕𝒉𝒂𝒕

{𝑟 ⏟| 𝑟 > 300, 𝑟 ∈ℝ}


𝒔𝒖𝒄𝒉 𝒕𝒉𝒂𝒕

J. Number Line
The number line is a visual notation. It is useful in solving questions since it provides an easy way to find the
intersection, or the union of two solutions.

Example 1.59
𝑥 is a real number greater than five. Write this in the different notation formats.

𝑥
⏟> 5 ⇔ (5, +∞ )
⏟ {𝑥 | 𝑥 > 5,
⇔⏟ 𝑥 ∈ ℝ}
𝑰𝒏𝒆𝒒𝒖𝒂𝒍𝒊𝒕𝒚 𝑵𝒐𝒕𝒂𝒕𝒊𝒐𝒏 𝑰𝒏𝒕𝒆𝒓𝒗𝒂𝒍 𝑵𝒐𝒕𝒂𝒕𝒊𝒐𝒏 𝑺𝒆𝒕 𝑵𝒐𝒕𝒂𝒕𝒊𝒐𝒏

Example 1.60: Notations: Number Line, Intervals, and Inequality


Write each of the following using the above three notations.
A. 𝑥 is a positive integer greater than 5.
B. 𝑦 is an odd integer less than 3.
C. 𝑧 is a real number less than or equal to 7.3
D. 𝑝 is a real number greater than or equal to 6
E. 𝑠 is a real number between 3 and 7
F. 𝑤 is a real number from to 2 and 9
G. 𝑟 is a real number greater than 5 and less than or equal to 12
{5 < 𝑟 ≤ 12, 𝑟 ∈ ℝ} ⇒ (5,12]

P a g e 21 | 97
Get all the files at: https://bit.ly/azizhandouts
Aziz Manva ([email protected])

Number Line Interval Notation Set Theory Notation


A [6,7,8 … ) {𝑥 > 5, 𝑥 ∈ 𝐼 + }
6 is included in the valid region for the inequality. Hence, we use a square bracket.
B (… − 3, −1, 0, 1] {𝑦 < 3, 𝑦 ∈ 2𝑛 + 1, 𝑛 ∈ 𝐼}
C (−∞, 7.3] {𝑧 ≤ 7.3, 𝑧 ∈ ℝ}
D [6, ∞) {𝑝 ≥ 6, 𝑧 ∈ ℝ}
6 is included in the valid region for the inequality. Hence, we use a square bracket.
Infinity can be never be included. That is why we use a round bracket.
E (3,7) {3 < 𝑠 < 7, 𝑠 ∈ ℝ}
3 and 7 are not included in the valid region for the inequality. Hence, we use a round bracket both sides
F [2,9] {2 ≤ 𝑠 ≤ 9, 𝑠 ∈ ℝ}
Both the endpoints (2 and 9) are included in the solution set. Hence, we use square brackets.

Example 1.61: Notations: Number Line, Intervals, and Inequality


The product of Hari’s age in years on his last birthday and his age now in complete months is 1800. Hari’s
current age is: (NMTC Sub-Junior/Screening 2004/19, Modified)

𝑦 = 𝑦𝑒𝑎𝑟𝑠, 𝑥 = 𝑚𝑜𝑛𝑡ℎ𝑠 𝑎𝑓𝑡𝑒𝑟 ℎ𝑖𝑠 𝑏𝑖𝑟𝑡ℎ𝑑𝑎𝑦 ⇒ 𝐴𝑔𝑒 𝑖𝑛 𝑀𝑜𝑛𝑡ℎ𝑠 = 12𝑦 + 𝑥

⏟ 2 +
𝑦(12𝑦 + 𝑥) = 1800 ⇒ 12𝑦 𝑥𝑦
⏟ = 1800
𝐿𝑎𝑟𝑔𝑒𝑟 𝑆𝑚𝑎𝑙𝑙𝑒𝑟

12𝑦 2 < 1800 ⇒ 𝑦 2 < 150 ⇒ 𝑦 < 13 ⇒ 𝑦 = 12, 12𝑦 2 = 1728


1728 + 12𝑥 = 1800 ⇒ 12𝑥 = 72 ⇒ 𝑥 = 6

1.4 Set Operations on Intervals


A. Union and Intersection

1.62: Union
The union sign in interval notation converts into OR in inequality notation
(𝑎, 𝑏) ∪ (𝑐, 𝑑) ⇔ 𝑎 < 𝑥 < 𝑏 𝑂𝑅 𝑐 < 𝑥 < 𝑑

1.63: Intersection
The intersection sign in interval notation converts into AND in inequality notation
(𝑎, 𝑏) ∩ (𝑐, 𝑑) ⇔ 𝑎 < 𝑥 < 𝑏 𝐴𝑁𝐷 𝑐 < 𝑥 < 𝑑

Example 1.64
Convert the following into inequality notation.
A. (−∞, −3)
B. (4, ∞)
C. (−∞, −3) ∪ (4, ∞)
D. (−∞, −3) ∩ (4, ∞)

Part A

P a g e 22 | 97
Get all the files at: https://bit.ly/azizhandouts
Aziz Manva ([email protected])

We might want to convert the interval into:


−∞ < 𝑥 < −3
But, this has the same meaning as:
𝑥 < −3
And that is how we will write it.

Part B

𝑥>4

Part C
𝑥 < −3 𝑂𝑅 𝑥 > 4
Part D
𝑥 < −3 𝐴𝑁𝐷 𝑥 > 4 ⇒ 𝑥 = 𝜙

Example 1.65
Convert the following into inequality notation.
A. (−∞, 5) ∪ {7}

𝑥 < 5 𝑂𝑅 𝑥 = 7

Example 1.66
Convert the following into inequality notation.
A. (−∞, 5) ∪ {3,8}

𝑥 < 5 𝑂𝑅 𝑥 = 3 𝑂𝑅 𝑥 = 8
But note that 3 is already a part of the first inequality, and hence it simplifies to:
𝑥 < 5 𝑂𝑅 𝑥 = 8

B. Logical Operators

Example 1.67
Hriday can have a class with his Philosophy Teacher on either a Tuesday or a Wednesday. Santosh can have a
class with his Philosophy Teacher on either a Monday or a Tuesday.
A. List the days when Hriday can have a class.
B. List the days when Santosh can have a class.
C. Hriday and Santosh want to have a class together with their philosophy teacher. What are the possible
days on which they can have it?
D. If the Philosophy Teacher wants to have a class with a single student, what are the different days on
which he can have the class? (Note: He can have two single back to back classes)

P a g e 23 | 97
Get all the files at: https://bit.ly/azizhandouts
Aziz Manva ([email protected])

Part A
{𝑇𝑢𝑒𝑠𝑑𝑎𝑦, 𝑊𝑒𝑑𝑛𝑒𝑠𝑑𝑎𝑦} ⇒ 𝐶𝑜𝑛𝑑𝑖𝑡𝑖𝑜𝑛 𝐼
Part B
{𝑀𝑜𝑛𝑑𝑎𝑦, 𝑇𝑢𝑒𝑠𝑑𝑎𝑦} ⇒ 𝐶𝑜𝑛𝑑𝑖𝑡𝑖𝑜𝑛 𝐼𝐼

Part C
To have a class with Hriday and Santosh, we must meet both of their conditions.
This is an AND condition.
Hence, the only day available is
𝑇𝑢𝑒𝑠𝑑𝑎𝑦
Part D
To have a class with either Hriday, or Santosh, we can meet any one of the conditions.
This is an OR condition.
Hence, the days available are:
{𝑀𝑜𝑛𝑑𝑎𝑦, 𝑇𝑢𝑒𝑠𝑑𝑎𝑦, 𝑊𝑒𝑑𝑛𝑒𝑠𝑑𝑎𝑦}
C. Revision: OR Conditions
The Logical OR means that we will accept either of the conditions.
A logical OR means we take a union of the given conditions.

Example 1.68: Logical AND (Intersection) and Logical OR (Union)


The following conditions are mentioned:
𝑒𝑖𝑡ℎ𝑒𝑟 𝑜𝑛 𝑡ℎ𝑒 𝑓𝑖𝑟𝑠𝑡 𝑜𝑟 𝑙𝑎𝑠𝑡 𝑑𝑎𝑦 𝑜𝑓 𝑡ℎ𝑒 𝑤𝑒𝑒𝑘, ⏟
⏟ 𝑜𝑛 𝑎 𝑤𝑒𝑒𝑘𝑒𝑛𝑑.
𝑪𝒐𝒏𝒅𝒊𝒕𝒊𝒐𝒏 𝑰 𝑪𝒐𝒏𝒅𝒊𝒕𝒊𝒐𝒏 𝑰𝑰

Note that Anshul’s week starts on Monday.


Write the days of the week on which Anshul can visit his parents, if
A. Either Condition I OR Condition II must be met
B. Condition I AND Condition II must be met
Part A
Our answer will be the union of Condition I, and Condition II:
𝑀𝑜𝑛𝑑𝑎𝑦
⏟ , 𝑆𝑢𝑛𝑑𝑎𝑦
⏟ ∪ ⏟ 𝑆𝑎𝑡𝑢𝑟𝑑𝑎𝑦 + 𝑆𝑢𝑛𝑑𝑎𝑦 ⇒ ⏟ 𝑀𝑜𝑛𝑑𝑎𝑦, 𝑆𝑎𝑡𝑢𝑟𝑑𝑎𝑦, 𝑆𝑢𝑛𝑑𝑎𝑦

𝑭𝒊𝒓𝒔𝒕 𝑫𝒂𝒚 𝑳𝒂𝒔𝒕 𝑫𝒂𝒚 𝑪𝒐𝒏𝒅𝒊𝒕𝒊𝒐𝒏 𝑰𝑰 𝑭𝒊𝒏𝒂𝒍 𝑳𝒊𝒔𝒕
𝑪𝒐𝒏𝒅𝒊𝒕𝒊𝒐𝒏 𝑰

Part B
Our answer will be the intersection of Condition I, and Condition II:
𝑀𝑜𝑛𝑑𝑎𝑦
⏟ , 𝑆𝑢𝑛𝑑𝑎𝑦
⏟ ∩ ⏟𝑆𝑎𝑡𝑢𝑟𝑑𝑎𝑦 + 𝑆𝑢𝑛𝑑𝑎𝑦 ⇒ 𝑆𝑢𝑛𝑑𝑎𝑦


𝑭𝒊𝒓𝒔𝒕 𝑫𝒂𝒚 𝑳𝒂𝒔𝒕 𝑫𝒂𝒚 𝑪𝒐𝒏𝒅𝒊𝒕𝒊𝒐𝒏 𝑰𝑰 𝑭𝒊𝒏𝒂𝒍 𝑳𝒊𝒔𝒕
𝑪𝒐𝒏𝒅𝒊𝒕𝒊𝒐𝒏 𝑰

D. AND in language, which means OR


It is not necessary that the word used in the language translates directly into the logical condition.

Example 1.69: Using AND to represent a Logical OR


Anshul is available to visit his parents on the first day of the week. And, he is also available to visit his parents
on the last day of the week. Anshul’s week starts on Tuesday. Write the days of the week on which Anshul can
visit his parents.

P a g e 24 | 97
Get all the files at: https://bit.ly/azizhandouts
Aziz Manva ([email protected])

We still need to find the union.


Anshul can visit on the first day (Tuesday), or on the last day (Saturday).

E. Revision: AND Conditions


An AND condition means that all the mentioned conditions have to satisfied simultaneously.

Example 1.70: Logical AND


Priyesh is a 𝑣𝑒𝑔𝑒𝑡𝑎𝑟𝑖𝑎𝑛
⏟ who ⏟
𝑑𝑜𝑒𝑠 𝑛𝑜𝑡 𝑙𝑖𝑘𝑒 𝑝𝑢𝑙𝑠𝑒𝑠. What food will he like?
𝑪𝒐𝒏𝒅𝒊𝒕𝒊𝒐𝒏 𝑰 𝑪𝒐𝒏𝒅𝒊𝒕𝒊𝒐𝒏 𝑰𝑰

We need to meet both the conditions. Therefore, a menu which is suitable for him must be
𝑣𝑒𝑔𝑒𝑡𝑎𝑟𝑖𝑎𝑛, 𝑎𝑛𝑑 𝑤𝑖𝑡ℎ𝑜𝑢𝑡 𝑝𝑢𝑙𝑠𝑒𝑠.

𝑴𝒆𝒆𝒕 𝒃𝒐𝒕𝒉 𝑪𝒐𝒏𝒅𝒊𝒕𝒊𝒐𝒏𝒔

F. Merging Inequalities
When we get an OR condition, we find either of the conditions
acceptable. In terms of set theory, we will find the union of the OR Condition
regions that satisfy each condition.

Depending upon the kind of question which is asked, the final Two Separate Entire Number
Single Region
Regions Line
answer may be a single region, two separate regions, or the entire
number line.

G. Single Region

Example 1.71
Combine into a single inequality:
𝑥 > 5 𝑂𝑅 𝑥 > 3

We can collapse the set of inequalities to a single inequality

And we can write the above using inequality notation, and interval notation:
𝑥 > 5 𝑂𝑅 𝑥 > 3 ⇒ 𝑥 > 3 ⇒ (3, ∞ )

Example 1.72

P a g e 25 | 97
Get all the files at: https://bit.ly/azizhandouts
Aziz Manva ([email protected])

Combine into a single inequality:


A. 𝑥 > −3 𝑂𝑅 𝑥 > 2
B. 𝑥 > 𝜋 𝑂𝑅 𝑥 > 3
C. 𝑥 > 1 𝑂𝑅 𝑥 ≥ 9
D. 𝑥 ≥ −2 𝑂𝑅 𝑥 > 12
E. 𝑥 ≥ −5 𝑂𝑅 𝑥 ≥ 1

𝑥 > −3 𝑂𝑅 𝑥 > 2 ⇒ 𝑥 > −3 ⇒ 𝑥 ∈ (−3, ∞)

𝑥 > 𝜋 𝑂𝑅 𝑥 > 3 ⇒ 𝑥 > 3


𝑥 > 1 𝑂𝑅 𝑥 ≥ 9 ⇒ 𝑥 > 1
𝑥 ≥ −2 𝑂𝑅 𝑥 > 12 ⇒ 𝑥 ≥ −2
𝑥 ≥ −5 𝑂𝑅 𝑥 ≥ 1 ⇒ 𝑥 ≥ −5

H. Two Separate Regions

Example 1.73
Determine the interval which satisfies each part below:
A. 𝑥 < 3 𝑂𝑅 𝑥 ≥ 5

This inequality cannot be converted into a single inequality. It comprises two separate regions, and the same is
reflected in both inequality as well as interval notation.

We want an OR, which means we want the union of the above two intervals.
(−∞, 3) ∪ [5, ∞)

Example 1.74
Determine the interval which satisfies:
A. 𝑥 ≤ 2 𝑂𝑅 𝑥 > 5
B.

(−∞, 2] ∪ (5, ∞)

Example 1.75: Entire Number Line


𝑥 < 3 𝑂𝑅 𝑥 ≥ 1

Note that the red inequality (representing 𝑥 < 3) overlaps the blue inequality (representing 𝑥 ≥ 1).

Hence, when we combine the two inequalities into a single area, we get the entire number line.
(−∞, 3) ∪ [1, ∞) ⇒ (−∞, ∞)

P a g e 26 | 97
Get all the files at: https://bit.ly/azizhandouts
Aziz Manva ([email protected])

A compact way of writing the above solution is to state 𝑥 can the value of any real number, that is, any value on
the number line.
𝑥∈ℝ
I. AND Conditions

Example 1.76
𝑥 > 2 𝐴𝑁𝐷 𝑥 < 3

Which can be written in interval notation as:


(2,3)
Example 1.77
𝑥 ≥ 2 𝐴𝑁𝐷 𝑥 < 3
𝑥 > 2 𝐴𝑁𝐷 𝑥 ≤ 3
𝑥 ≥ 2 𝐴𝑁𝐷 𝑥 ≤ 3

𝑥 ≥ 2 𝐴𝑁𝐷 𝑥 < 3 ⇒ [2,3)


𝑥 > 2 𝐴𝑁𝐷 𝑥 ≤ 3 ⇒ (2,3]
𝑥 ≥ 2 𝐴𝑁𝐷 𝑥 ≤ 3 ⇒ [2,3]

Example 1.78: No Solution


𝑥 < 3 𝐴𝑁𝐷 𝑥 > 5

𝑥 < 3 𝐴𝑁𝐷 𝑥 > 5 ⇒ (−∞, 3) ∩ [5, ∞) ⇒ 𝝓


⏟ ⇒ 𝑵𝒐 𝑺𝒐𝒍𝒖𝒕𝒊𝒐𝒏
𝑵𝒖𝒍𝒍 𝑺𝒆𝒕

−16
−9𝑥 + 2 > 18 ⇒ −9𝑥 > 16 ⇒ 𝑥 <
9
29
13𝑥 + 15 ≤ −14 ⇒ 13𝑥 ≤ −29 ⇒ 𝑥 ≤ −
13

Example 1.79
Solve the following inequalities
A. 𝑥 is a odd number between 7 and 12.
B. 𝑥 is an number fulfilling any of the following conditions:
a. It is more than or equal to 5
b. It is less than 3.

{7 < 𝑥 < 12, 𝑥 = 2𝑛 + 1, 𝑛 ∈ 𝐼}


𝑥 < 3 𝑂𝑅 𝑥 ≥ 5 ⇒ (−∞, 3) ∪ [5, ∞)

P a g e 27 | 97
Get all the files at: https://bit.ly/azizhandouts
Aziz Manva ([email protected])

1.5 One Step Inequalities


A. Addition

1.80: Types of Inequalities


There are four types of inequalities
𝑥 >𝑦
𝑥 <𝑦
𝑥 ≥𝑦
𝑥 ≤𝑦

We will state the properties for only the first type, but they apply to all four types of inequalities.

1.81: Adding the same number


You can add the same number to both sides of an inequality.
𝑥 > 𝑦 ⇒ 𝑥+𝑎 > 𝑦+𝑎

This is similar to solving an equation by adding the same number to both sides.
𝑥 = 𝑦 ⇒ 𝑥+𝑎 = 𝑦+𝑎

For example:
4> 3 ⇒ 4+1 > 3+1⇒ 5> 4

Example 1.82
Solve the following, and write your answer in interval notation.
A. 𝑥 − 7 = 4
B. 𝑥 − 7 > 4
1 1
C. 𝑥 − 2 > 3

Part A
Add 7 to both sides:
𝑥 − 7 + 7 = 4 + 7 ⇒ 𝑥 = 11
Part B
Add 7 to both sides of the inequality:
𝑥 − 7 + 7 > 4 + 7 ⇒ 𝑥 > 11

11 ⏟ ⇒ 𝑥 ∈ (11, ∞)
⏟ , ∞
𝑆𝑡𝑎𝑟𝑡 𝐸𝑛𝑑
𝑃𝑜𝑖𝑛𝑡 𝑃𝑜𝑖𝑛𝑡
Part C
1 1 1 1 5 5
𝑥 − + > + ⇒ 𝑥 > ⇒ ( , ∞)
2 2 3 2 6 6

Example 1.83
Find all possible value of the number in each case, and write your answer in interval notation:

P a g e 28 | 97
Get all the files at: https://bit.ly/azizhandouts
Aziz Manva ([email protected])
2 4
A. When 3 is subtracted from a number, the number is greater than 3. Find all possible values that the
number can take.
1 2
B. When 2 is subtracted from a number, the number is greater than or equal to 5. Find all possible values
that the number can take.
3 2
C. A number is at least 2 4 less than 1 3. Find the range of values that the number can take.

2 4 6
𝑥− > ⇒ 𝑥 > ⇒ 𝑥 > 2 ⇒ 𝑥 ∈ (2, ∞)
3 3 3
1 2 2 1 9 9
𝑥− ≥ ⇒𝑥≥ + ⇒𝑥≥ ⇒ [ , ∞)
2 5 5 2 10 10

2 3 5 11 20 − 33 13 1 13
𝑥 <1 −2 = − = =− = −1 ⇒ (−∞, − )
3 4 3 4 12 12 12 12

B. Constants

1.84: Some Constants


➢ 𝜋 is the ratio of the circumference of a circle to its diameter.
➢ 𝑒 is a number such that 𝑒 𝑥 grows at the same rate as itself.

𝐶
𝐶 = 𝜋𝐷 ⇒ = 𝜋 ⇒ 𝐶: 𝐷 = 𝜋: 1
𝐷
𝑒 ≈ 2.71
If want the answer in terms of the exact value of 𝜋 or 𝑒, treat it like a variable and keep solving as usual.

1.85: Approximate Value


➢ The exact value of 𝜋 or 𝑒 is represented using the symbol itself, that is, 𝜋. The exact value cannot be
represented using a real number.
➢ When we want an approximate value, we replace the constant with its approximation.

𝜋 ≈ 3.14
22
𝜋≈
7
Note that:
22
̅̅̅̅̅̅̅̅̅̅ ≠ 3.14
= 3. 142857
7

Example 1.86
A. A number is greater than 4 by at least 3. Find the exact range of values that the number can take.
B. A number is greater than 𝜋 by at least 𝑒. Find the exact range of values that the number can take.

Part A
𝑥 − 3 ≥ 4 ⇒ 𝑥 ≥ 7 ⇒ [7, ∞)
Part B
𝜋 ≈ 3.14 ⇒ 𝜋 ≠ 3.14
Since the questions asks for exact values, we will have to work using 𝜋 and 𝑒 themselves.

P a g e 29 | 97
Get all the files at: https://bit.ly/azizhandouts
Aziz Manva ([email protected])

𝑥 − 𝑒 ≥ 𝜋 ⇒ 𝑥 ≥ 𝜋 + 𝑒 ⇒ [𝜋 + 𝑒, ∞)
C. Integer Solutions

Example 1.87
A. If the number of customers coming in to Fred’s Diner for lunch everyday were to decrease by 7, he
would still get more than 20 customers. What is the number of customers that could be coming in to
Fred’s Diner currently?
B. After a flyover was introduced, the number of cars at a traffic signal reduced by 12. What was the
number of cars at the traffic signal before the flyover?
C. When 5 is subtracted from an integer 𝑛, the resulting number is less than −5. What is the value of 𝑛?
D. Last week, Alfred sold seven sheep, and now the number of sheep he has is less than nine. What was the
original number of sheep Alfred had?
E. Mustufa is dreaming that the number of homework problems he has to do magically reduced by 9.
However, he still has at least 12 hard and at least 6 very hard problems. If the number of problems
reduced is split between hard and very hard as far as possible, find the original number of hard and very
hard problems that Mustufa was supposed to solve.

Part A
𝑑 − 7 > 20 ⇒ 𝑑 > 27 ⇒ 𝑑 ≥ 28 ⇒ 𝑑 ∈ {28,29, … }

𝑑 ∈ [28, ∞) is wrong because that includes numbers like 28.3 which are not possible for the number of
customers.
Part B
𝐶𝑢𝑟𝑟𝑒𝑛𝑡 𝑛𝑢𝑚𝑏𝑒𝑟 𝑜𝑓 𝑐𝑎𝑟𝑠 ≥ 0
𝑂 = 𝑂𝑟𝑖𝑔𝑖𝑛𝑎𝑙 𝑛𝑢𝑚𝑏𝑒𝑟 𝑜𝑓 𝑐𝑎𝑟𝑠 ≥ 12 ⇒ 𝑂 ∈ {12,13, … }
Part C
𝑛 − 5 < −5 ⇒ 𝑛 < 0 ⇒ 𝑛 ∈ {… , −2, −1}
Part D
𝑠 − 7 < 9 ⇒ 𝑠 < 16 ⇒ 𝑠 ≤ 15 ⇒ 𝑠 ∈ {0,1,2, … ,15}
Part E
𝐻𝑎𝑟𝑑 𝑃𝑟𝑜𝑏𝑙𝑒𝑚𝑠 = ℎ, 𝑉𝑒𝑟𝑦 ℎ𝑎𝑟𝑑 𝑝𝑟𝑜𝑏𝑙𝑒𝑚𝑠 = 𝑣
There are two cases:
ℎ − 4 ≥ 12 ⇒ ℎ ≥ 16 ⇒ ℎ ∈ {16,17,18, … }
𝑣 − 5 ≥ 6 ⇒ 𝑣 ≥ 11 ⇒ 𝑣 ∈ {11,12,13, … }

ℎ − 5 ≥ 12 ⇒ ℎ ≥ 17 ⇒ ℎ ∈ {17,18,19, … }
𝑣 − 4 ≥ 6 ⇒ 𝑣 ≥ 10 ⇒ 𝑣 ∈ {10,11,12, … }

D. Subtraction

1.88: Subtracting the same number


You can subtract the same number from both sides of an inequality.
𝑥 > 𝑦 ⇒ 𝑥−𝑎 > 𝑦−𝑎

This is similar to solving an equation by subtracting the same number to both sides.
𝑥 = 𝑦 ⇒ 𝑥−𝑎 = 𝑦−𝑎

For example:

P a g e 30 | 97
Get all the files at: https://bit.ly/azizhandouts
Aziz Manva ([email protected])

4> 3 ⇒ 4−1 > 3−1⇒ 3> 2

Example 1.89
Solve the following:
A. 𝑥 + 5 = 9
B. 𝑥 + 5 < 9
3
C. 𝑥 + 4 > 2
D. 𝑥 + 0.01 > 3

Part A
Subtract 5 from both sides:
𝑥−5+5=9−5⇒𝑥 =4
Part B
Subtract 5 from both sides:
𝑥−5+5<9−5⇒𝑥 <4
Part C
3
Subtract 4 from both sides:
3 3 3 5
𝑥+ − >2− ⇒𝑥 >
4 4 4 4
Part D
Subtract 0.01 from both sides:
𝑥 + 0.01 − 0.01 > 3 − 0.01 ⇒ 𝑥 > 2.99

Example 1.90
A. When 5 is added to a number, it is less than 4. Find the range of possible values 𝑛 that the number can
take.
B. If the quantity of milk 𝑚 in a container is increased by 3.5 litres, the increased quantity is greater than
or equal to 5 Litres. Find the range that the original quantity could have been.
2 1
C. When a number 𝑥 is increased by 3, it becomes greater than 5. Find the range of values that the number
can take.
D. If the number of gorillas, 𝑔 (which is greater than 1), in a local forest population increases by 4, it will be
still be 3 or more less than 27 gorillas counted in the last census taken by a forest ranger. Find the range
of values that the gorillas can take.
E. Amogh could not finish his homework on Monday, and had three problems left to do. On Tuesday, he
could not finish two problems. On Wednesday, there was no homework, and on Thursday, he completed
his homework. If he didn’t touch the homework problems from Monday and Tuesday, and the total
number of homework problems that he had pending at the end of the week was less than twelve, then
find a range for the number of homework problems, 𝑝, that he could have had pending on Friday. (For
this problem, consider that homework is only assigned Monday to Friday, not over the weekend).
F. Jane has 𝑥 red cars and 𝑦 blue cars. She paired her red and blue cars. At least four blue cars did not have
red cars to pair with (and all red cars were paired). Find, in terms of 𝑦, the set of values that 𝑥 can take.

Part A
𝑛 + 5 < 4 ⇒ 𝑛 < −1 ⇒ 𝑛 ∈ (−∞, −1)
Part B
𝑚 + 3.5 ≥ 5 ⇒ 𝑚 ≥ 1.5 ⇒ 𝑚 ∈ [1.5, ∞)
Part C

P a g e 31 | 97
Get all the files at: https://bit.ly/azizhandouts
Aziz Manva ([email protected])

2 1 1 2 3 − 10 7 7
𝑥+ > ⇒𝑥> − = =− ⇒ 𝑥 ∈ (− , ∞)
3 5 5 3 15 15 15
Part D
𝑔 + 4 ≤ 27 − 3 ⇒ 𝑔 ≤ 20 ⇒ 𝑔 ∈ {2,3, … ,20}
Part E
3 + 2 + 𝑝 < 12 ⇒ 𝑝 < 7 ⇒ 𝑝 ∈ {0,1, … ,6} Mon Tue Wed Thu Fri Sat Sun
Pending 3 2 0 0 𝑝 0 0
Part F

Pairs of Red and Blue Cars


All red cars were paired. In the diagram, we
can add more pairs of red and blue cars
without violating the condition of the
question.
Unpaired Blue Cars
There were at least blue cars that were not
paired. In the diagram, we can add more blue
cars to the right, without violating the
condition of the question.
Framing an Inequality
From the above, we can see that the number of blue cars is at least 4 more than the number of red cards. And,
we can convert this into an inequality as follows:

⏟ 𝑦 ⇒ 𝑥 ≤ 𝑦 − 4 ⇒ 𝑥 ∈ (−∞, 𝑦 − 4]
𝑥 +4 ≤ ⏟
𝐵𝑙𝑢𝑒 𝑅𝑒𝑑

E. Division

1.91: Dividing by the same number


For positive values of 𝑎:
𝑥 𝑦
𝑥>𝑦⇒ >
𝑎 𝑎

For negative values of 𝑎:


𝑥 𝑦
𝑥>𝑦⇒ <
𝑎 𝑎
Note: If the number that you are dividing by is negative, then the sign of the inequality will reverse.

2 3
2<3⇒ <
5 5
2 < 3 ⇒ −2 > −3

Example 1.92
Solve
A. 3𝑥 < 4
B. −5𝑥 ≥ 9
C. −2𝑥 > −3
D. 6𝑧 < 144
E. 4𝑥 ≤ −4
F. 0.2𝑥 > 0.7
G. −0.7𝑥 > −0.4

P a g e 32 | 97
Get all the files at: https://bit.ly/azizhandouts
Aziz Manva ([email protected])
4 3
H. 3
𝑥 ≥4
I. −2.5𝑎 < 6.8

Part A
4 4
𝑥< ⇒ 𝑥 ∈ (−∞, )
3 3

Part B
9 9
𝑥 ≤ − ⇒ 𝑥 ∈ (−∞, − ]
5 5
Part C
3 3
𝑥< ⇒ 𝑥 ∈ (−∞, )
2 2

Part D
144
6𝑧 < 144 ⇒ 𝑧 < ⇒ 𝑧 < 24 ⇒ 𝑧 ∈ (−∞, 24)
6
Part E
𝑥 ≤ −1 ⇒ 𝑥 ∈ (−∞, −1]

Part F
0.7 7
𝑥> = ⇒ 𝑥 ∈ (3.5, ∞)
0.2 2

Part G
−0.4 4 4
𝑥< = ⇒ 𝑥 ∈ (−∞, )
−0.7 7 7

Part H
3 4
𝑥≥ × = 1 ⇒ [1, ∞)
4 3

Part I
6.8 68 68
−2.5𝑎 < 6.8 ⇒ 𝑎 > ⇒𝑎> ⇒ 𝑎 ∈ ( , ∞)
2.5 25 25

Example 1.93
A. Three times a number is less than 4. Find all possible values that the number can take.
B. One-fifth of a number is more than two-fifth. Find all possible values that the number can take.
C. Negative-two-fifth of a number is less than negative two-fifths. Find the values that the number can take.

4 4
3𝑥 < 4 ⇒ 𝑥 < ⇒ 𝑥 ∈ (−∞, )
3 3
𝑥 2
> ⇒ 𝑥 > 2 ⇒ 𝑥 ∈ (2, ∞)
5 5

Part C
2 2
− 𝑛<−
5 5
2
Divide both by − 5, and reverse the sign of the inequality since we dividing by a negative number:

P a g e 33 | 97
Get all the files at: https://bit.ly/azizhandouts
Aziz Manva ([email protected])

𝑛 > 1 ⇒ 𝑛 ∈ (1, ∞)

Example 1.94
Four times a positive number 𝑛 is greater than or equal to 5.
A. What is the range of the values that the number can take?
B. Which integer values of 𝑛 satisfy the condition above?
C. How many integer values of 𝑛 satisfy the condition above?

Part A
5 1 1
4𝑛 ≥ 5 ⇒ 𝑛 ≥ ⇒ 𝑛 ≥ 1 ⇒ 𝑛 ∈ [1 , ∞)
4 4 4

Part B
𝑛 ∈ {2,3,4,5, … }
Part C
𝐼𝑛𝑓𝑖𝑛𝑖𝑡𝑒

Example 1.95
An orchard has two trees, each with the same number of apples. The apples are all collected and divided into
three piles. Each pile has more than 17 apples.
A. Find the minimum number of apples that each tree can have.
B. Find all possible values for the number of apples that the orchard can have.

Let the number of apples on a tree be 𝑡.


2𝑡 3
≥ 18 ⇒ 𝑡 ≥ 18 × ⇒ 𝑡 > 27 ⇒ 𝑜 = 2𝑡 > 54
3 2

Example 1.96
𝑝 is a prime number. When 𝑝 is multiplied by the smallest prime number, and divided by the second smallest
prime number, the answer is less than the third smallest prime number. Find the product of all values that 𝑝 can
take.

2𝑝 3 15
<5⇒𝑝 <5× ⇒𝑝< ⇒ 𝑝 < 7.5 ⇒ 𝑝 ∈ {2,3,5,7} ⇒ 𝑃𝑟𝑜𝑑𝑢𝑐𝑡 = 210
3 2 2

Example 1.97
Define a 𝑓𝑎𝑐𝑡𝑜𝑟 to be a natural number that divides another number, leaving no remainder. For example, the
factors of 8 are 1, 2 and 4. A number 𝑛 is divided by the factors of 6, in succession. That is, it is divided by the
first factor, then the result is divided by the second factor, and so on, till the last factor. The final answer is less
than −3. Find the:
A. range of values that 𝑛 can take.
B. largest integer value that 𝑛 can take.
C. largest integer value that 𝑛 can take, if the final answer is an integer less than −3, and other things
remain the same.

𝐹𝑎𝑐𝑡𝑜𝑟𝑠 𝑜𝑓 6 = {1,2,3,6}

P a g e 34 | 97
Get all the files at: https://bit.ly/azizhandouts
Aziz Manva ([email protected])

𝑛÷1÷2÷3÷6
Convert the division into multiplication by taking the reciprocal of each divisor:
1 1 1 1 𝑛 𝑛
𝑛× × × × = =
1 2 3 6 1 × 2 × 3 × 6 36
Part A
Now, we can frame the inequality:
𝑛
< −3 ⇒ 𝑛 < −108 ⇒ 𝑛 ∈ (−∞, −108)
36

Part B
−109
Part C
𝑛
≤ −4 ⇒ 𝑛 ≤ −144 ⇒ 𝑛 ∈ (−∞, −144] ⇒ 𝐿𝑎𝑟𝑔𝑒𝑠𝑡 𝑣𝑎𝑙𝑢𝑒 𝑖𝑠 𝑛 = −144
36

Example 1.98
Reema has to order pizzas for a party. She and eight of her friends will have three slices each. Each pizza has
four slices. Reema is fine with ordering extra, but she should not fall short. Write and solve an inequality that
represents a valid range for the number of pizzas 𝑝 that Reema can order. (The pizza shop accepts orders for a
pizzas in positive integers).

27 3
4𝑝 > 9 × 3 ⇒ 4𝑝 > 27 ⇒ 𝑝 > ⇒ 𝑝 > 6 ⇒ 𝑝 = {7,8,9, … }
4 4
F. Multiplication

1.99: Multiplying by the same number


For positive values of 𝑎:
𝑥 > 𝑦 ⇒ 𝑎𝑥 > 𝑎𝑦

For negative values of 𝑎:


𝑥 > 𝑦 ⇒ 𝑎𝑥 < 𝑎𝑦
Note:
If the number that you are multiplying by is positive, then the sign of the inequality stays the same.
If the number that you are multiplying by is negative, then the sign of the inequality will reverse.

For example
2 < 3 ⇒ −2 > −3

Example 1.100
Solve:
𝑥
A. 3
>4
𝑥
B. −2
<3
𝑥
C. 5
≥ −9
−𝑥
D. ≤1
8
𝑥
E. > 0.2
0.1
𝑥 2
F. 1 <
3

2

P a g e 35 | 97
Get all the files at: https://bit.ly/azizhandouts
Aziz Manva ([email protected])
𝑥 0.3
G. 0.2 ≥ − 0.7
0.5
𝑥 3
H. 1 ≤ −7
−2
2
3

Part A
Multiply both sides by 3:
𝑥
> 4 ⇒ 𝑥 > 12 ⇒ 𝑥 ∈ (12, ∞)
3
Part B
Multiply both sides by −2, and flip the inequality:
𝑥 > −6
Part C
Multiply both sides by 5:
𝑥 ≥ −45
Part D
Multiply both sides by −8, and flip the inequality:
𝑥 ≥ −8
Part E
Multiply both sides by 0.1:
𝑥 > 0.02
Part F
1
Multiply both sides by − 2:
2 1 1
𝑥 > ( ) (− ) ⇒ 𝑥 > −
3 2 3
Part G
0.2
Multiply both sides by :
0.5
0.3 0.2 3 2 6
𝑥≥− ( ) ⇒ 𝑥 ≥ −( )( ) ⇒ 𝑥 ≥ −
0.7 0.5 7 5 35
Part H
1
− 1 2 1 3 3
2
The denominator of the LHS simplifies to 2 = − 2 ÷ 3 = − 2 × 2 = − 4 giving us:
3
𝑥 3
≤−
3 7
−4
3
Multiply both sides of the inequality by − 4, and flip the inequality:
3 𝑥 3 3 9
− × ≥ − (− ) ⇒ 𝑥 ≥
4 −3 7 4 28
4

1.101: Multiplying by the Reciprocal


𝑎
For positive values of 𝑏 :
𝑎 𝑏
𝑥>𝑦⇒𝑥> 𝑦
𝑏 𝑎
𝑎
For negative values of 𝑏 :
𝑎 𝑏
𝑥>𝑦⇒𝑥< 𝑦
𝑏 𝑎

P a g e 36 | 97
Get all the files at: https://bit.ly/azizhandouts
Aziz Manva ([email protected])

G. Review

1.102: Summary of Inequality Properties

Consider the inequality 𝒙 > 𝒚


Adding/Subtracting the same number (Valid)
𝑥±𝑎 >𝑦±𝑎
Multiplying/Dividing by a Negative number Multiplying/Dividing by a Positive number
Reverses the direction of the inequality. Valid (and maintains the direction of the inequality)
𝑎𝑥 < 𝑎𝑦, 𝑎<0 𝑎𝑥 > 𝑎𝑦, 𝑎>0

MCMC 1.103
𝑀𝑎𝑟𝑘 𝑎𝑙𝑙 𝑜𝑝𝑡𝑖𝑜𝑛𝑠 𝑤ℎ𝑖𝑐ℎ 𝑚𝑢𝑠𝑡 𝑏𝑒 𝑡𝑟𝑢𝑒.
We flip the sign of an inequality when:
A. We exchange the LHS and the RHS
B. We have a negative sign on the LHS, or on the RHS.
C. We multiply by a negative number.
D. We add a negative number.
E. We divide by a number.

MCQ 1.104
If we multiply both sides of an inequality by 2:
A. It is valid, and both sides become larger.
B. It is valid, and both sides become smaller
C. It is valid, and the LHS and the RHS can become larger or smaller.
D. It is not a valid operation. The inequality may not hold true.

Matrix Match 1.105 Column I Column II


2𝑧 1. Flip the sign of the inequality A. Multiply by 0.2, and divide by
− > 0.3
0.2 2. Do not flip the sign of the −2
Pick one option from Column I, and inequality B. Multiply by 0.2, and divide by 2
one option from Column II to give a 3. Change the inequality to ≥ C. Multiply by −10
set of steps that will solve the 4. Change the inequality to ≤ D. Divide by −10
inequality. Give 𝑎𝑙𝑙 𝑣𝑎𝑙𝑖𝑑 𝑝𝑎𝑖𝑟𝑠.

1.6 Two Step Inequalities


A. Justification of Steps

Example 1.106
4 1
Solve 3𝑥 + 2 > 5𝑥 − 2. Justify each step using an appropriate property.

1 4 5 4
3𝑥 > 5𝑥 − − = 5𝑥 − Subtract from both sides of the inequality
2
2 2 2

P a g e 37 | 97
Get all the files at: https://bit.ly/azizhandouts
Aziz Manva ([email protected])

5 Subtract 5𝑥 from both sides of the inequality


−2𝑥 > −
2
5 5 −1 5 Divide both sides of the inequality by −2.
𝑥 < − ÷ (−2) = − × =
2 2 2 4 Accordingly, reverse the sign of the inequality.
5 Simplify
𝑥<
4
5 Convert the inequality into interval notation
𝑥 ∈ (−∞, )
4

B. Basics

Example 1.107
Solve the following inequality:
3𝑥 + 4 > 6

2 2
3𝑥 > 2 ⇒ 𝑥 > ⇒ 𝑥 ∈ ( , ∞)
3 3

Example 1.108
−65𝑦 + 19 < −2𝑦 + 41

22
−63𝑦 + 19 < 41 ⇒ −63𝑦 < 22 ⇒ 𝑦 > −
63

Example 1.109
Solve the following inequality:
2𝑥 + 7 ≤ 8𝑥 − 5

−6𝑥 + 7 ≤ −5 ⇒ −6𝑥 ≤ −12 ⇒ 𝑥 ≥ 2 ⇒ 𝑥 ∈ [2, ∞)

Example 1.110
Solve the following inequality:
4𝑥 + 3 > 5𝑥 + 2

4𝑥 + 3 > 5𝑥 + 2 ⇒ 4𝑥 > 5𝑥 − 1 ⇒ −𝑥 > −1 ⇒ 𝑥 < 1 ⇒ 𝑥 ∈ (−∞, 1)

Example 1.111
Solve the following inequality:
2𝑥 + 5 ≤ 9𝑥 − 7

12 12 12
12 ≤ 7𝑥 ⇒ ≤𝑥⇒𝑥≥ ⇒ 𝑥 ∈ [ , ∞)
7 7 7

C. Word Problems on Numbers

Example 1.112
When a number is increased by five, and then decreased by six, the result is more than seven. Find the values(s)

P a g e 38 | 97
Get all the files at: https://bit.ly/azizhandouts
Aziz Manva ([email protected])

that the number can take.

𝑥 + 5 − 6 > 7 ⇒ 𝑥 > 8 ⇒ 𝑥 ∈ (8, ∞)

Example 1.113
When three times a number is decreased by seven, it is more than or equal to seven times the number increased
by four. Find the values(s) that the number can take.

11
3𝑥 − 7 ≥ 7𝑥 + 4 ⇒ 3𝑥 ≥ 7𝑥 + 11 ⇒ −4𝑥 ≥ 11 ⇒ 𝑥 ≤
−4

Example 1.114
Twice a number is more than the sum of the number and fourteen. What could the number be?

2𝑥 > 𝑥 + 14 ⇒ 2𝑥 > 14 ⇒ 𝑥 > 7 ⇒ 𝑥 ∈ (14, ∞)

Example 1.115
Five times a number decreased by four is greater than four times a number increased by seven. Find the
values(s) that the number can take.

5𝑥 − 4 > 4𝑥 + 7 ⇒ 𝑥 > 11 ⇒ 𝑥 ∈ (11, ∞)

Example 1.116
When four is subtracted from a number and the result is divided by four, the answer is greater than or equal to
four. Find the values(s) that the number can take.

𝑥−4
> 4 ⇒ 𝑥 − 4 > 16 ⇒ 𝑥 > 20 ⇒ 𝑥 ∈ (20, ∞)
4

Example 1.117
When a number is divided by four and then four subtracted from the result, the answer is greater than or equal
to four. Find the values(s) that the number can take.

𝑥 𝑥
− 4 > 4 ⇒ > 8 ⇒ 𝑥 > 32 ⇒ 𝑥 ∈ (32, ∞)
4 4

Example 1.118
Operation I: 𝑀𝑢𝑙𝑡𝑖𝑝𝑙𝑦 𝑏𝑦 2
Operation II: Add 3

A. Consider a number 𝑥. Perform Operation I followed by Operation II. The answer is less than 3. Find the
values that 𝑥 can take.
B. Consider a number 𝑦. Perform Operation II followed by Operation I. The answer is more than or equal to
3. Find the values that 𝑦 can take.

Part A
2𝑥 + 3 < 3 ⇒ 2𝑥 < 0 ⇒ 𝑥 < 0 ⇒ 𝑥 ∈ (−∞, 0)

P a g e 39 | 97
Get all the files at: https://bit.ly/azizhandouts
Aziz Manva ([email protected])

Part B
3 3
2(𝑦 + 3) ≥ 3 ⇒ 2𝑦 + 6 ≥ 3 ⇒ 2𝑦 ≥ −3 ⇒ 𝑦 ≥ − ⇒ 𝑦 ∈ [− , ∞)
2 2

Example 1.119
Alan takes a number, multiplies it by two, subtracts four, and then divides by three. Allen takes the same number
than Alan originally took, multiples it by three, adds four, and then divides by two. Alan’s final answer is less
than or equal to Allen’s answer. Find the values that the number can be.

2𝑛 − 4 3𝑛 + 4

⏟3 ⏟2
𝐴𝑙𝑎𝑛 𝐴𝑙𝑙𝑒𝑛
Multiply both sides by 6:
2(2𝑛 − 4) ≤ 3(3𝑛 + 4) ⇒ 4𝑛 − 8 ≤ 9𝑛 + 12
4𝑛 ≤ 9𝑛 + 20
−5𝑛 ≤ 20
𝑛 ≥ −4

D. Word Problems on Scenarios

Example 1.120

E. Integral Answers

Example 1.121
For a party, Shaun invites 𝑝 people over. He wants three slices of pizza for every person (and also for himself).
He also wants five slices extra. Every pizza has seven slices. What is the minimum (integral) number of pizzas
that he should order? (Answer in terms of 𝑝.)

𝑁𝑜. 𝑜𝑓 𝑆𝑙𝑖𝑐𝑒𝑠 𝑁𝑒𝑒𝑑𝑒𝑑 = ⏟


3 (𝑝 + 1) + ⏟
×⏟ 5 = 3𝑝 + 8
𝑆𝑙𝑖𝑐𝑒𝑠 𝑝𝑒𝑟 𝑁𝑜. 𝑜𝑓 𝐸𝑥𝑡𝑟𝑎
𝑃𝑒𝑟𝑠𝑜𝑛 𝑃𝑒𝑜𝑝𝑙𝑒 𝑆𝑙𝑖𝑐𝑒𝑠
𝑆𝑙𝑖𝑐𝑒𝑠 𝑝𝑒𝑟 𝑃𝑖𝑧𝑧𝑎: 7

𝑇𝑜𝑡𝑎𝑙 𝑆𝑙𝑖𝑐𝑒𝑠 3𝑝 + 8
𝑁𝑜. 𝑜𝑓 𝑃𝑖𝑧𝑧𝑎𝑠 = =
𝑆𝑙𝑖𝑐𝑒𝑠 𝑝𝑒𝑟 𝑃𝑖𝑧𝑧𝑎 7

To get the number of pizzas, we must round up the expression above:


3𝑝 + 8
⌈ ⌉
7
F. Variables

Example 1.122
Jason has three large bags of rice, each with 𝑙 kg of rice. He uses the rice to make lunch every day for a week, and
at the end of the week, he has 1 kg rice left over. The next week, he gets five medium sized bags of rice, each
with 𝑚 kg of rice. He makes lunch from the rice from Monday to Friday, and he has two kg rice left over. The
total quantity of rice used in the first week is more than the total quantity of rice used in the second week.
A. Find the value of 𝑚 in terms of 𝑙.

P a g e 40 | 97
Get all the files at: https://bit.ly/azizhandouts
Aziz Manva ([email protected])

B. Find the value of 𝑙 in terms of 𝑚.

5𝑚 − 1
3𝑙
⏟− 1 > 5𝑚 − 2
⏟ ⇒ 3𝑙 > 5𝑚 − 1 ⇒ 𝑙 >
𝐹𝑖𝑟𝑠𝑡 𝑊𝑒𝑒𝑘 𝑆𝑒𝑐𝑜𝑛𝑑 𝑊𝑒𝑒𝑘
3
3𝑙 + 1 3𝑙 + 1
3𝑙
⏟− 1 > 5𝑚 − 2
⏟ ⇒ 3𝑙 + 1 > 5𝑚 ⇒ >𝑚⇒𝑚<
𝐹𝑖𝑟𝑠𝑡 𝑊𝑒𝑒𝑘 𝑆𝑒𝑐𝑜𝑛𝑑 𝑊𝑒𝑒𝑘
5 5

Example 1.123

G. Fractions and Decimals

Example 1.124
Solve the following inequalities
2 1 3
𝑥− >
3 2 5

2 11 33 33
𝑥> ⇒𝑥> ⇒ 𝑥 ∈ ( ,∞ )
3 10 20 20

Example 1.125
What is the greatest integer value for 𝑚 such that 5𝑚 − 3 > 8𝑚 + 21? (MathCounts 1998 State Countdown)

Add 3 both sides:


5𝑚 > 8𝑚 + 24
Subtract 8𝑚 both sides:
−3𝑚 > 24
Divide both sides by −3, and flip the inequality:
24
𝑚<
−3

Example 1.126
Nine copies of a certain pamphlet cost less than $10.00 while ten copies of the same pamphlet (at the same
price) cost more than $11.00. How much does one copy of this pamphlet cost? (Answer to two decimal places)
(AMC 8 1985/18)

10 1
9𝑥 < 10 ⇒ 𝑥 < ⇒ 𝑥 < 1 = 1. 1̅
9 9

11 1
10𝑥 > 11 ⇒ 𝑥 > =1 = 1.10
10 10

1.10 < 𝑥 < 1. 1̅ ⇒ 𝑥 = 1.11

P a g e 41 | 97
Get all the files at: https://bit.ly/azizhandouts
Aziz Manva ([email protected])

H. No Solutions

Example 1.127: Inequalities with no solutions


Solve 3 + 4(3𝑥 + 1) > 4(3𝑥 + 2) + 2

3 + 4(3𝑥 + 1) > 4(3𝑥 + 2) + 2 ⇒ 3 + 12𝑥 + 4 > 12𝑥 + 8 + 2 ⇒ 7 > 10 ⇒ 𝑁𝑜 𝑆𝑜𝑙𝑢𝑡𝑖𝑜𝑛𝑠


𝑥=𝜙

I. Reciprocals

Example Property
Case I (Same Sign) 1 1 𝟏 𝟏
𝑥, 𝑦 are both + ve: 2 < 3 ⇒ > 𝒙<𝒚⇒ >
2 3 𝒙 𝒚
1 1 Reverse the inequality when taking
𝑥, 𝑦 are both − ve: − 3 < −2 ⇒ − > −
3 2 reciprocals if both sides have the
same sign.
Case II (Opposite Sign)

J. Squares
Let 𝑥 and 𝑦 be positive.
If both are positive:
𝑥 < 𝑦 ⇒ 𝑥2 < 𝑦2
3 < 4 ⇒ 32 < 42 ⇒ 9 < 16
If both are negative (Sign of the inequality reverses):
−𝑥 < −𝑦 ⇒ 𝑥 2 < 𝑦 2
−4 < −3 ⇒ 16 > 4

One is negative and one is positive:


−3 < 4 ⇒ 32 < 42
−5 < 4 ⇒ 25 > 16

K. Square Roots (and Absolute Value)


√𝑥 2 = |𝑥|
Take an example:
𝐶𝑎𝑠𝑒 𝐼: 𝑥 = 5 ⇒ 𝑥 2 = 25 ⇒ √𝑥 2 = √25 = 5
𝑦 = −5 ⇒ 𝑦 2 = 25 ⇒ √𝑦 2 = √25 = 5

1.7 Compound Inequalities


A. Compound Inequalities
Compound inequalities are inequalities where two conditions are written in the same inequality.

The simpler versions can be solved by manipulating the entire inequality in one go. The more complex versions
must be broken into two different inequalities, each of which must be solved separately, and the merged using
an intersection.

P a g e 42 | 97
Get all the files at: https://bit.ly/azizhandouts
Aziz Manva ([email protected])

Example 1.128: Simpler Version


3
A. 5
< 𝑥 < 4, 𝑥 ∈ ℤ

We can solve the inequality by manipulating it as one entity.


3 3 5𝑥 20
<𝑥<4⇒ < < ⇒⏟ 3 < 5𝑥 < 20 ⇒ 𝑥 ∈ {1,2, 3}
5 ⏟
5 5 5 𝑴𝒖𝒍𝒕𝒊𝒑𝒍𝒚 𝒃𝒚 𝟓
𝑴𝒂𝒌𝒆 𝒅𝒆𝒏𝒐𝒎𝒊𝒏𝒂𝒕𝒐𝒓𝒔 𝒕𝒉𝒓𝒐𝒖𝒈𝒉𝒐𝒖𝒕
𝒄𝒐𝒎𝒎𝒐𝒏

B. AND Conditions

Example 1.129
Solve:
13 9
𝑥≥− 𝐴𝑁𝐷 𝑥 > −
8 2

13
𝑥≥−
8

9 36
𝑥>− ⇒𝑥>−
2 8

13
𝑥≥−
8

Example 1.130
4 − 3𝑥 ≤ 13 𝐴𝑁𝐷 6 + 2𝑥 > −2

−3𝑥 ≤ 9 ⇒ 𝑥 ≥ −3
2𝑥 > −8 ⇒ 𝑥 > −4
Final Answer:
𝑥 ≥ −3

1.131: Tripartite Inequalities


𝑎<𝑏<𝑐

𝑎 < 𝑏 𝐴𝑁𝐷 𝑏 < 𝑐

Example 1.132

−5 < 2𝑛 − 1 ≤ 5
−4 < 2𝑛 ≤ 6
−2 < 𝑛 ≤ 3

P a g e 43 | 97
Get all the files at: https://bit.ly/azizhandouts
Aziz Manva ([email protected])

Example 1.133
2𝑥 < 𝑥 + 1 < 2𝑥 + 5

If you have variables in more than one part of the inequality, it is generally required to break the inequality into
two parts:
2𝑥 < 𝑥 + 1 ⇒ 𝑥 < 1
𝑥 + 1 < 2𝑥 + 5 ⇒ −𝑥 < 4 ⇒ 𝑥 > −4

𝑥 < 1 𝐴𝑁𝐷 𝑥 > −4 ⇒ −4 < 𝑥 < 1 ⇒ (−4,1)

Example 1.134
1 3 1 2
Consider the compound inequality 3𝑥 + 2 < 𝑥 − 4 < 2 𝑥 + 3
A. Find the solution set that satisfies the inequality.
B. What is the largest integer that satisfies the above inequality?
C. What is the largest natural number that satisfies the above inequality?

Part A
1 3 3 1 5 5
𝐹𝑖𝑟𝑠𝑡 𝐶𝑜𝑛𝑑𝑖𝑡𝑖𝑜𝑛: 3𝑥 +< 𝑥 − ⇒ 2𝑥 < − − = − ⇒ 𝑥 < −
2 4 4 2 4 8
3 1 2 1 2 3 17 17
𝑆𝑒𝑐𝑜𝑛𝑑 𝐶𝑜𝑛𝑑𝑖𝑡𝑖𝑜𝑛: 𝑥 − < 𝑥 + ⇒ 𝑥 < + = ⇒𝑥<
4 2 3 2 3 4 12 6
5 17 5 5
𝑀𝑒𝑟𝑔𝑒 𝑡ℎ𝑒 𝑡𝑤𝑜 𝐶𝑜𝑛𝑑𝑖𝑡𝑖𝑜𝑛𝑠: 𝑥 < − 𝐴𝑁𝐷 𝑥 < ⇒ 𝑥<− ⇒ 𝑥 ∈ (−∞, − )
8 6 ⏟ 8 ⏟ 8
𝑰𝒏𝒆𝒒𝒖𝒂𝒍𝒊𝒕𝒚 𝑵𝒐𝒕𝒂𝒕𝒊𝒐𝒏 𝑰𝒏𝒕𝒆𝒓𝒗𝒂𝒍 𝑵𝒐𝒕𝒂𝒕𝒊𝒐𝒏

Part B
5
We cannot have positive answers, nor zero. We look for the first integer that we will encounter to the left of − 8
on the number line, which is −1.
Part C
No natural numbers satisfy the inequality. Hence, there are no solutions.

Example 1.135
An electrical device supports a minimum temperature of −10°𝐶 and a maximum temperature of 40°𝐶 (both
5
inclusive). Convert the given range into degrees Fahrenheit using the conversion (𝐹 − 32) = 𝐶.
9

Write out the condition given in the question in inequality form:


−10 ≤ 𝐶 ≤ 40
Substitute the expression for degrees Fahrenheit:
5
−10 ≤ (𝐹 − 32) ≤ 40
9
9
Multiply through the inequality by 5:
9 9 5 9
( ) (−10) ≤ ( ) (𝐹 − 32) ≤ ( ) 40
5 5 9 5
Simplify:
−18 ≤ 𝐹 − 32 ≤ 72
Add 32 to all parts of the inequality:
−18 + 32 ≤ 𝐹 − 32 + 32 ≤ 72 + 32
Simplify:

P a g e 44 | 97
Get all the files at: https://bit.ly/azizhandouts
Aziz Manva ([email protected])

14 ≤ 𝐹 ≤ 104

Example 1.136
A bakery needs to make dough for 100 pizzas. Since there is some estimation involved, it does not need to make
exactly 100 pizzas. Rather, the final number of pizzas should be at least 90, or less than 110.
A. Write a compound inequality that the number of pizzas must satisfy.
B. Combine the two inequalities into a single inequality using absolute value.

The number of pizzas should be between 90 and 100 (both included).


90
⏟ ≤ 𝑝 ≤ 110

𝑀𝑖𝑛 𝑀𝑎𝑥
Subtract 100 from all three parts of the inequality:
−10 ≤ 𝑝 − 100 ≤ 10
Combine using absolute value:
|𝑝 − 100| ≤ 10
The absolute value of the difference of the number of pizzas and 100 should be less than or equal to 10.

Example 1.137
3 1
(4 − 𝑥) <= − (𝑥 + 2) 𝐴𝑁𝐷 (𝑥 − 1) + (3𝑥 + 6) > 2(𝑥 − 9)
5 3

Multiply the first inequality by 15 both sides:


9(4 − 𝑥) ≤ −5(𝑥 + 2)
36 − 9𝑥 ≤ −5𝑥 − 10
−4𝑥 ≤ −46
46
𝑥≥−
−4
23
𝑥≥
2

Simplify the second inequality:


4𝑥 + 5 > 2𝑥 − 18
2𝑥 > −23
23
𝑥>−
2

Example 1.138
One morning each member of Angela's family drank an 8-ounce mixture of coffee with milk. The amounts of
coffee and milk varied from cup to cup, but were never zero. Angela drank a quarter of the total amount of milk
and a sixth of the total amount of coffee. How many people are in the family?1 (AMC 10 2000/22, AMC 12
2000/13)

Method I If the milk had been shared equally, then Angela


Let the number of people in the family be 𝑝. would have got:
Angela drank a greater proportion of the milk(4 𝑡ℎ)
1 1
𝑜𝑓 𝑡ℎ𝑒 𝑚𝑖𝑙𝑘
1 𝑝
than she drank of the coffee(6 𝑡ℎ). But she drank a greater proportion of the milk than
the coffee:

1
An alternate solution to this using Remainders can be found in the Number Theory Note on Mod Arithmetic.

P a g e 45 | 97
Get all the files at: https://bit.ly/azizhandouts
Aziz Manva ([email protected])

1 1 The total number of ounces consumed is then:


< 𝑐 𝑚 2𝑐𝑝 + 3𝑚𝑝
𝑝
⏟ 4 = 8𝑝 = ( + ) 𝑝 =
𝑰𝒏𝒆𝒒𝒖𝒂𝒍𝒊𝒕𝒚 𝑰 6 4 12
But she drank a smaller proportion of the coffee And it is also:
than the milk: 𝑐+𝑚
1 1 Hence:
< 2𝑐𝑝 + 3𝑚𝑝
6
⏟ 𝑝 =𝑐+𝑚
𝑰𝒏𝒆𝒒𝒖𝒂𝒍𝒊𝒕𝒚 𝑰𝑰 12
Combine I and II to get: Multiply by 12:
1 1 1 2𝑐𝑝 + 3𝑚𝑝 = 12𝑐 + 12𝑚
< < ⇒6>𝑝>4 Rearrange to get 𝑐 terms on one side and 𝑚 terms
6 𝑝 4
And since 𝑝 is an integer, we must have: on the other side:
𝑝=5 2𝑐𝑝 − 12𝑐 = 12𝑚 − 3𝑚𝑝
Method II Factor:
We can do this by introducing variables for each 2𝑐(𝑝 − 6) = 3𝑚(4 − 𝑝)
quantity under consideration. The resulting Since 𝑐 > 0, 𝑚 > 0:
solution is lengthier and more cumbersome. 𝐵𝑜𝑡ℎ 𝑝 − 6 𝑎𝑛𝑑 4 − 𝑝 𝑚𝑢𝑠𝑡 ℎ𝑎𝑣𝑒 𝑡ℎ𝑒 𝑠𝑎𝑚𝑒 𝑠𝑖𝑔𝑛
Let If both are positive:
𝑇𝑜𝑡𝑎𝑙 𝑎𝑚𝑡 𝑜𝑓 𝑐𝑜𝑓𝑓𝑒𝑒 = 𝑐 𝑝−6>0⇒𝑝>6
𝑇𝑜𝑡𝑎𝑙 𝑎𝑚𝑡 𝑜𝑓 𝑚𝑖𝑙𝑘 = 𝑚 4 − 𝑝 > 0 ⇒ 𝑝 < 4 ⇒ 𝑵𝒐 𝑺𝒐𝒍𝒖𝒕𝒊𝒐𝒏
𝑁𝑜. 𝑜𝑓 𝑝𝑒𝑜𝑝𝑙𝑒 = 𝑝 Hence, both must be negative:
𝑝−6<0⇒𝑝<6
Note that Angela drank: 4−𝑝 <0⇒4<𝑝 ⇒𝑝 >4
𝑐 𝑚 And since 𝑝 ∈ ℕ:
+ = 8 𝑜𝑢𝑛𝑐𝑒𝑠
6 4 𝑝=5

C. Multiple Conditions

Example 1.139
(𝑥 > 5 𝑎𝑛𝑑 𝑥 < 10) 𝑎𝑛𝑑 (𝑥 > 7)

5 < 𝑥 < 10 𝐴𝑁𝐷 𝑥 > 7

7 < 𝑥 < 10 ⇒ (7,10)


Example 1.140
(𝑥 > 0 𝑎𝑛𝑑 𝑥 < 7) 𝑂𝑅 (𝑥 < 0 𝑎𝑛𝑑 𝑥 > −7)

(−7 < 𝑥 < 0) 𝑂𝑅 (0 < 𝑥 < 7)

(−7,0) ∪ (0,7)

1.8 Absolute Value Inequalities


A. Warmup
Before we introduce some rules, we solve some questions based only on logic. In the next heading, we
generalize these questions to get properties that will be useful in solving inequalities.

P a g e 46 | 97
Get all the files at: https://bit.ly/azizhandouts
Aziz Manva ([email protected])

Example 1.141
Match the column below. Solve the inequalities for integer values. Use this to find the number of solutions, and
hence, solve them for the valid interval on the number line.

Inequality Inequality Interval Integral Solutions No. of Integral


Notation Solutions
𝐴. |𝑥| < 3 −3 < 𝑥 < 3 I. 𝑥 ∈ (−2,2) X. {−2, −1,0,1,2} 3
𝐵. |𝑥| ≤ 3 −3 ≤ 𝑥 ≤ 3 𝐼𝐼. 𝑥 ∈ [−2,2] 𝑌. {−3, −2, −1,0,1,2,3} 5
𝐶. |𝑥| < 2 −2 < 𝑥 < 2 𝐼𝐼𝐼. 𝑥 ∈ (−3,3) 𝑍. {−1,0,1} 7
𝐷. |𝑥| ≤ 2 −2 ≤ 𝑥 ≤ 2 𝐼𝑉. 𝑥 ∈ [−3,3] 9

𝐴 − 𝐼𝐼𝐼 − 𝑋 − 5
The largest value of 𝑥 which satisfies |𝑥| < 3 is a number just less than three. Any number more than three, or
three itself will have an absolute value more than 3.
|3.5| = 3.5 > 3
Similarly, any value of 𝑥 which is less than −3 will not satisfy the inequality.
For example
|−3.5| = 3.5 > 3
Hence, the numbers which satisfy the inequality are the number between −3 and 3. The endpoints are not
included since the inequality is strict (it does have the equals sign).
Therefore, the interval that satisfies the inequality is
(−3,3) ⇒ 𝐴𝑙𝑙 𝑟𝑒𝑎𝑙 𝑛𝑢𝑚𝑏𝑒𝑟𝑠 𝑏𝑒𝑡𝑤𝑒𝑒𝑛 − 3 𝑎𝑛𝑑 3 𝑜𝑛 𝑡ℎ𝑒 𝑛𝑢𝑚𝑏𝑒𝑟 𝑙𝑖𝑛𝑒

𝐵 − 𝐼𝑉 − 𝑌 − 7
The only difference between A and B is that B also includes the endpoints of the range, since it has an equals
sign.
𝐶−𝐼−𝑍−3
𝐷 − 𝐼𝐼 − 𝑋 − 5

Example 1.142
Match the column below. Solve the inequalities for integer values. Use this to find the number of solutions, and
hence, solve them for the valid interval on the number line.

Inequality Inequality Notation Interval Integral Solutions


𝐴. |𝑥| > 4 I. 𝑥 ∈ (−∞, −3) ∪ (3, ∞) X. {−5, −4, −3} ∪ {3,4,5 … }
𝐵. |𝑥| ≥ 4 II. 𝑥 ∈ (−∞, −4) ∪ (4, ∞) 𝑌. {… − 6, −5, −4}
∪ {4,5,6 … }
𝐶. |𝑥| > 3 III. 𝑥 ∈ (−∞, −5) ∪ (5, ∞) 𝑍. {… − 7, −6, −5}
∪ {5,6,7 … }
𝐷. |𝑥| ≥ 3 IV. 𝑥 ∈ (−∞, −3] ∪ [3, ∞)
V. 𝑥 ∈ (−∞, −4] ∪ [4, ∞)
VI. 𝑥 ∈ (−∞, −5] ∪ [5, ∞)

Part A
|𝑥| > 4 will take minimum value 5, if 𝑥 is an integer. But if 𝑥 is not restricted to be an integer, it will take values
between 4 and 5 as well.
4.1 is a valid solution for A. But if we match A with III, we do not get 4.1 in our solutions.
𝐴 − 𝐼𝐼 − 𝑍
Part B

P a g e 47 | 97
Get all the files at: https://bit.ly/azizhandouts
Aziz Manva ([email protected])

The ≥ sign lets us include the endpoints. Hence, in interval notation, the answer remains the same in terms of
numbers, but the round brackets change to square brackets.
𝐵−𝑉−𝑌
Part C
𝐶−𝐼−𝑌
Part D
𝐷 − 𝐼𝑉 − 𝑋

B. Properties

Example 1.143
Find the solution set that satisfies
|𝑥| < 5
To structure your thinking, break the problem down into three parts:
A. Positive Numbers
B. Zero
C. Negative Numbers

Part A
Any number larger than 5 is not going to work.
|100| = 100 > 5 ⇒ 𝐷𝑜𝑒𝑠 𝑛𝑜𝑡 𝑤𝑜𝑟𝑘
|6| = 6 > 5 ⇒ 𝐷𝑜𝑒𝑠 𝑛𝑜𝑡 𝑤𝑜𝑟𝑘
In fact,
|5| = 5 ⇒ 𝐴𝑙𝑠𝑜 𝑑𝑜𝑒𝑠 𝑛𝑜𝑡 𝑤𝑜𝑟𝑘
Because we want a number that is less than 5.

Hence, the positive values that will work are:


𝑥 < 5, 𝑥 𝑖𝑠 + 𝑣𝑒
Part B
Zero also works

Part C
Any number smaller than −5 is not going to work.
|−100| = 100 > 5 ⇒ 𝐷𝑜𝑒𝑠 𝑛𝑜𝑡 𝑤𝑜𝑟𝑘
|−6| = 6 > 5 ⇒ 𝐷𝑜𝑒𝑠 𝑛𝑜𝑡 𝑤𝑜𝑟𝑘
In fact,
|−5| = 5 ⇒ 𝐴𝑙𝑠𝑜 𝑑𝑜𝑒𝑠 𝑛𝑜𝑡 𝑤𝑜𝑟𝑘
Because we want a number that is less than 5.

Hence, the negative values that will work are:


𝑥 > −5, 𝑥 𝑖𝑠 − 𝑣𝑒

Combine the above three to get:


𝑥 < 5, 𝑥 𝑖𝑠 + 𝑣𝑒
𝑥=0
𝑥 > −5, 𝑥 𝑖𝑠 − 𝑣𝑒

And these three can be written in one inequality as:

P a g e 48 | 97
Get all the files at: https://bit.ly/azizhandouts
Aziz Manva ([email protected])

−5 < 𝑥 < 5

|𝑥| < 5 ⇔ −5 < 𝑥 < 5 ⇔ 𝑥 ∈ (−5,5)

1.144: Less Than Absolute Value Inequalities


A less than absolute value inequality will convert into an AND inequality and 𝒗𝒊𝒄𝒆 𝒗𝒆𝒓𝒔𝒂.

Assume that 𝑐 is a positive number.

Property Absolute Value Inequality Notation Interval Notation


Notation
I A |𝑥| ≤ 𝑐 −𝑐 ≤ 𝑥 ≤ 𝑐 𝑥 ∈ [−𝑐, 𝑐]
B |𝑥| < 𝑐 −𝑐 < 𝑥 < 𝑐 𝑥 ∈ (−𝑐, 𝑐)

1.145: “Negative” Absolute Values


For a positive number 𝑐:
|𝑥| < −𝑐 ⇒ 𝑥 ∈ 𝝓
|𝑥| ≤ −𝑐 ⇒ 𝑥 ∈ 𝝓

There are no solutions to the inequalities above, because an absolute value expression can never be negative.

Example 1.146
Find the solution set for the inequalities below. Write your answer in both inequality notation, and interval
notation.
A. |𝑥| < 4
B. |𝑥| ≤ 3
1
C. |𝑥| < 2
D. |𝑥| < −7
E. |𝑥| + 2 < 5
1 1
F. |𝑥| + 2 > |𝑥| + 1
3 2
G. |𝑥| ≤ −9
H. |𝑥 + 1| ≤ 4

Part A |𝑥| < 3 ⇔ 𝑥 ∈ (−3,3)


−4 < 𝑥 < 4 ⇔ 𝑥 ∈ (−4,4) Part F
Part B 1 1
|𝑥| > |𝑥| − 1 ⇒ 2|𝑥| > 3|𝑥| − 6 ⇒ −|𝑥| > −6
|𝑥| ≤ 3 ⇔ −3 ≤ 𝑥 ≤ 3 ⇔ [−3,3] 3 2
Part C |𝑥| < 6 ⇒ 𝑥 ∈ (−6,6)
1 1 Part G
𝑥 ∈ [− , ] |𝑥| ≤ −9 ⇒ 𝑥 ∈ 𝝓
2 2
Part D Part I
|𝑥| < −7 ⇒ 𝑥 ∈ 𝝓 |𝑥 + 1| ≤ 4
Part E −4 < 𝑥 + 1 < 4
−5 < 𝑥 < 3

P a g e 49 | 97
Get all the files at: https://bit.ly/azizhandouts
Aziz Manva ([email protected])

1.147: Absolute Values greater than a Negative Number


For a positive number 𝑐:
|𝑥| > −𝑐 ⇒ 𝑥 ∈ (−∞, ∞) ⇔ 𝑥 ∈ ℝ
|𝑥| ≥ −𝑐 ⇒ 𝑥 ∈ (−∞, ∞) ⇔ 𝑥 ∈ ℝ

An absolute value expression can never be negative. Hence, all inputs are valid for the expression.
Hence, the solution is the real number line.

Equality will never be achieved in the condition above, since the absolute value of an expression is never
negative.
∴ |𝑥| = −𝑐 ⇒ 𝑥 = 𝝓

Example 1.148
Solve
A. |𝑥| > −7
B. |𝑥| ≥ −9

|𝑥| > −7 ⇒ 𝑥 ∈ (−∞, ∞) ⇔ 𝑥 ∈ ℝ


|𝑥| ≥ −9 ⇒ 𝑥 ∈ (−∞, ∞) ⇔ 𝑥 ∈ ℝ

1.149: Absolute Values greater than a positive number


A greater than absolute value inequality will convert into an OR inequality and 𝒗𝒊𝒄𝒆 𝒗𝒆𝒓𝒔𝒂.
|𝑥| > 𝑐 ⇒ 𝑥 < −𝑐 𝑂𝑅 𝑥 > 𝑐 ⇔ 𝑥 ∈ 𝑥 ∈ (−∞, −𝑐) ∪ (𝑐, ∞)
|𝑥| ≥ 𝑐 ⇒ 𝑥 ≤ −𝑐 𝑂𝑅 𝑥 ≥ 𝑐 ⇔ 𝑥 ∈ (−∞, −𝑐] ∪ [𝑐, ∞)

Example 1.150
Solve:
A. |𝑥| > 7
B. |𝑥| ≥ 9
C. |𝑥 + 8| > 1

|𝑥| > 7 ⇔ 𝑥 < −7 𝑂𝑅 𝑥 > 7


|𝑥| ≥ 9 ⇔ 𝑥 ≤ −9 𝑂𝑅 𝑥 ≥ 9

|𝑥 + 8| > 1
𝑥 + 8 > 1 ⇒ 𝑥 > −7
𝑥 + 8 < −1 ⇒ 𝑥 < −9

1.151: Summary Table


|𝑥| < 𝑐 ⇒ −𝑐 < 𝑥 < 𝑐, 𝑐>0
|𝑥| < −𝑐 ⇒ 𝑥 ∈ 𝜙, 𝑐>0
|𝑥| > −𝑐 ⇒ 𝑥 ∈ ℝ, 𝑐>0
|𝑥| > 𝑐 ⇒ 𝑥 > 𝑐 𝑂𝑅 𝑥 < −𝑐, 𝑐>0

Property Example
I |𝒙| < 𝟓 ⇔ −𝟓 < 𝒙 < 𝟓 ⇔ 𝒙 ∈ (−𝟓, 𝟓)
|𝒙| ≤ 𝝅 ⇔ −𝝅 ≤ 𝒙 ≤ 𝝅 ⇔ 𝒙 ∈ [−𝝅, 𝝅]
II |𝒙| ≤ −𝟑 ⇔ 𝒙 ∈ 𝝓

P a g e 50 | 97
Get all the files at: https://bit.ly/azizhandouts
Aziz Manva ([email protected])

|𝒙| ≤ −𝒆 ⇔ 𝒙 ∈ 𝝓
III |𝒙| > −𝟑 ⇒ 𝒙 ∈ (−∞, ∞) ⇔ 𝒙 ∈ ℝ
IV |𝒙| > 𝟑 ⇒ 𝒙 ∈ (−∞, −𝟑) ∪ (𝟑, ∞) ⇔ 𝒙 < −𝟑 𝑶𝑹 𝒙 > 𝟑

C. Solving Absolute Value Inequalities

Example 1.152
A. |𝑥| > 4
B. |𝑥| < 4
C. |2𝑐 − 5| < 9
D. |3𝑐 + 2| < 4
E. |𝑥 + 7| ≤ 2
F. |𝑥 − 1| ≥ 3
G. 3|2 − 2𝑥| − 7 ≤ 29
H. |2𝑥 + 1| − 5 < 0
I. 4 − 3|2𝑥 + 1| > −2
J. |2𝑥 − 5| ≤ 3𝑥

Part A |1 − 𝑥| ≤ 6
𝑥 > 4 𝑂𝑅 𝑥 < −4
Part B −6 ≤ 1 − 𝑥 < 6
𝑥 < 4 𝐴𝑁𝐷 𝑥 > −4 ⇒ −4 < 𝑥 < 4 −7 ≤ −𝑥 < 5
Part C 7 ≥ 𝑥 > −5
Recall that if |𝑥| < 𝑐 ⇔ −𝑐 < 𝑥 < 𝑐: −5 ≤ 𝑥 ≤ 7
−9 < 2𝑐 − 5 < 9 Part H
−4 < 2𝑐 < 14 |2𝑥 + 1| − 5 < 0
−2 < 𝑐 < 7 |2𝑥 + 1| < 5
𝑥 ∈ (−2,7) −5 < 2𝑥 + 1 < 5
Part D −6 < 2𝑥 < 4
−4 < 3𝑐 + 2 < 4 −3 < 𝑥 < 2
Subtract 2 from all parts: 𝑥 ∈ (−3,2)
−6 < 3𝑐 < 2
Divide all three parts by 3: Part I
2 2 −3|2𝑥 + 1| > −6 ⇒ |2𝑥 + 1| < 2
−2 < 𝑐 < ⇒ 𝑥 ∈ (−2, )
3 3 Apply Property I
Part E −2 < 2𝑥 + 1 < 2
−2 ≤ 𝑥 + 7 ≤ 2 Subtract one throughout:
−9 ≤ 𝑥 + 7 ≤ −5 −3 < 2𝑥 < 1
𝑥 ∈ [−5, −9] Divide by 2 throughout:
Part F 3 1
𝑥 − 1 ≥ 3 ⇒ 𝑥 ≥ 4 ⇒ 𝑥 ∈ [4, ∞) − <𝑥<
2 2
𝑥 − 1 ≤ −3 ⇒ 𝑥 ≤ −2 𝑥 ∈ (−∞, −2] Write it in interval notation:
3 1
𝑥 ∈ (− , )
(−∞, −2] ∪ [4, ∞) 2 2
Part G Part J
Add 7 to both sides, and factor out 2 from the Use the property that |𝑦| ≤ 𝑎 ⇔ −𝑎 < 𝑦 < 𝑎:
absolute value: −3𝑥 ≤ 2𝑥 − 5 ≤ 3𝑥
6|1 − 𝑥| ≤ 36 First Part:

P a g e 51 | 97
Get all the files at: https://bit.ly/azizhandouts
Aziz Manva ([email protected])

−3𝑥 ≤ 2𝑥 − 5 ⇒ −5𝑥 ≤ −5 ⇒ 𝑥 ≥ 1
Second Part: Both the inequalities need to be satisfied:
2𝑥 − 5 ≤ 3𝑥 ⇒ −𝑥 ≤ 5 ⇒ 𝑥 ≥ −5 𝑥 ≥ 1 𝐴𝑁𝐷 𝑥 ≥ −5 ⇒ 𝑥 ≥ 1

D. Critical Points
A critical point of an expression is a value at which the expression changes its behavior. For example, consider
the equation
𝑦 =𝑥+5
Which is graphed alongside.

From the graph, we see that:


𝑥 < −5 ⇒ 𝑦 < 0
𝑥 > −5 ⇒ 𝑦 > 0
𝑥 = −5 ⇒ 𝑦 = 0

𝑥 −8 −7 −6 −5 −4 −3 −2 −1 0
𝑥+5 −3 −2 −1 0 1 2 3 4 5
𝑁𝑒𝑔𝑎𝑡𝑖𝑣𝑒 𝐼𝑛𝑡𝑒𝑟𝑣𝑎𝑙 𝑪𝒓𝒊𝒕𝒊𝒄𝒂𝒍 𝑷𝒐𝒊𝒏𝒕 𝑃𝑜𝑠𝑖𝑡𝑖𝑣𝑒
(−∞, −5) (−5, ∞)
OR OR
𝑥 < −5 𝑥 > −5

We are interested in finding the value of the critical point. Tabulating or graphing is not feasible for every
expression.

A continuous function will not change sign from positive to negative, or negative to positive, without crossing
the 𝑥-axis.
Hence, we find all points where the expression is zero. This lets us identify critical points.

Critical Points for Products

Critical Points for Products


𝑎𝑏𝑐 = 0
𝑎𝑏𝑐 is a product. Hence, if any of a, b or c become zero, the entire expression is zero.

Critical Points for Sum of Absolute Values

|𝑥| + |𝑦| + |𝑧| = 0


Analyzing
𝑥 = 0, 𝑦 = 0, 𝑧=0
is useful since the behaviour of the expression changes
𝑓𝑜𝑟 𝑥 > 0, 𝑎𝑛𝑑 𝑥 < 0

Example 1.153
3|2 − 2𝑥| − 7 ≤ 29

Add 7 to both sides, and factor out 2 from the absolute value:
6|1 − 𝑥| ≤ 36

P a g e 52 | 97
Get all the files at: https://bit.ly/azizhandouts
Aziz Manva ([email protected])

|1 − 𝑥| ≤ 6

Apply casework on the absolute value:


1 − 𝑥 ≥ 0 ⇒ −𝑥 ≥ −1 ⇒ 𝑥 ≤ 1
Case I: 𝑥 ≤ 1
1−𝑥 ≤6
−𝑥 ≤ 5
𝑥 ≥ −5
Case II: 𝑥 > 1
𝑥−1≤6
𝑥≤7

Combine the results from the two cases to get:


−5 ≤ 𝑥 ≤ 7

Example 1.154
Solve
4 − 3|2𝑥 + 1| > −2

Just like absolute value equations, here also we isolating the absolute value expression on side:
−3|2𝑥 + 1| > −6 ⇒ |2𝑥 + 1| < 2
Apply Property I
Break into two separate inequalities
1
𝐶𝑎𝑠𝑒 𝐼: 2𝑥 + 1 < 2 ⇒ 2𝑥 < 1 ⇒ 𝑥 <
2
3
𝐶𝑎𝑠𝑒 𝐼𝐼: 2𝑥 + 1 > −2 ⇒ 2𝑥 > −3 ⇒ 𝑥 > −
2
3 1
𝑥 ∈ (− , )
2 2
Solve the compound inequality directly
−2 < 2𝑥 + 1 < 2
Subtract one throughout:
−3 < 2𝑥 < 1
Divide by 2 throughout:
3 1
− <𝑥<
2 2
Write it in interval notation:
3 1
𝑥 ∈ (− , )
2 2

Example 1.155
|2𝑥 − 5| ≤ 3𝑥

5
2𝑥 − 5 ≥ 0 ⇒ 𝑥 ≥
2
5
Case I: 𝑥 ≥ 2
|2𝑥 − 5| = 2𝑥 − 5

P a g e 53 | 97
Get all the files at: https://bit.ly/azizhandouts
Aziz Manva ([email protected])

2𝑥 − 5 ≤ 3𝑥 ⇒ −𝑥 ≤ 5 ⇒ 𝑥 ≥ −5 ⇒ 𝑥 ∈ [−5, ∞)
5
But since 𝑥 ≥ :
2
5
𝑥 ∈ [ , ∞)
2

5
Case II: 𝑥 < 2
|2𝑥 − 5| = 5 − 2𝑥
5 − 2𝑥 ≤ 3𝑥 ⇒ −5𝑥 ≤ −5 ⇒ 𝑥 ≥ 1 ⇒ 𝑥 ∈ [1, ∞)
5
But since 𝑥 < :
2
5
𝑥 ∈ [1, )
2
And then we can combine both cases:
5 5
𝑥 ∈ [1, ) ∪ [ , ∞) ⇒ 𝑥 ∈ [1, ∞)
2 2

Example 1.156
|2𝑥 − 5| ≤ 𝑥 − 4

5
2𝑥 − 5 ≥ 0 ⇒ 2𝑥 ≥ 5 ⇒ 𝑥 ≥
2
5
Case I: 𝑥 ≥ 2
2𝑥 − 5 ≤ 𝑥 − 4 ⇒ 𝑥 ≤ 1 ⇒ 𝐶𝑜𝑛𝑡𝑟𝑎𝑑𝑖𝑐𝑡𝑖𝑜𝑛

5
Case I: 𝑥 < 2
5 − 2𝑥 ≤ 𝑥 − 4 ⇒ −3𝑥 < −9 ⇒ 𝑥 > 3 ⇒ 𝐶𝑜𝑛𝑡𝑟𝑎𝑑𝑖𝑐𝑡𝑖𝑜𝑛

Hence, there are no solutions to the above inequality.

Example 1.157
13 − 2|3𝑥 − 4| ≥ 3

Isolate the absolute value:


−2|3𝑥 − 4| ≥ −10 ⇒ |3𝑥 − 4| ≤ 5

Method I: Convert to a compound inequality


1
−5 ≤ 3𝑥 − 4 ≤ 5 ⇒ −1 ≤ 3𝑥 ≤ 9 ⇒ − ≤ 𝑥 ≤ 3
3
Method II: Critical Points
4
3𝑥 − 4 > 0 ⇒ 3𝑥 > 4 ⇒ 𝑥 >
3

4 4
Case I: 𝑥 ≥
⇒ 3𝑥 − 4 ≤ 5 ⇒ 3𝑥 ≤ 9 ⇒ 𝑥 ≤ 3 ⇒ 𝑥 ∈ [ , 3]
3 3
4 1 1 4
Case II: 𝑥 ≤ ⇒ 4 − 3𝑥 ≤ 5 ⇒ −3𝑥 ≤ 1 ⇒ 𝑥 ≥ − ⇒ 𝑥 ∈ [− , ]
3 3 3 3
1
[− , 3]
3

P a g e 54 | 97
Get all the files at: https://bit.ly/azizhandouts
Aziz Manva ([email protected])

Example 1.158
Solve
|4𝑥 + 2| > 𝑥 + 3

1
4𝑥 + 2 > 0 ⇒ 4𝑥 > −2 ⇒ 𝑥 > −
2
1
Case I: 𝑥 ≥ − 2
1
4𝑥 + 2 ≥ 𝑥 + 3 ⇒ 3𝑥 ≥ 1 ⇒ 𝑥 ≥
3
1
Case I: 𝑥 < − 2
−4𝑥 − 2 > 𝑥 + 3
−5 > 5𝑥
−1 > 𝑥
𝑥 < −1

Final Answer
1 1
𝑥 < −1 𝑂𝑅 𝑥 > ⇔ (−∞, −1) ∪ [ , ∞)
3 3

Example 1.159
Find the critical points of
|𝑥 + 2| + |𝑥 − 3| = 0
First Critical Point
The first critical point is obtained by equating the first expression inside the absolute value to zero:
𝑥 + 2 = 0 ⇒ 𝑥 = −2
-2 divides the real number line into two regions:
(−∞, −2) ∪ (−2, ∞)
We will check the behaviour of the expression 𝑥 + 2 for a single value in each region. A single value is sufficient
because the expression will not its behaviour sign in the same region.
Right of -2:
𝑊ℎ𝑒𝑛 𝑥 = 0: 𝑥 + 2 = 0 + 2 = 2 = +𝑣𝑒
Hence, we can conclude:
|𝑥 + 2| = 𝑥 + 2, 𝑤ℎ𝑒𝑛 𝑥 > −2

Left of -2:
𝑊ℎ𝑒𝑛 𝑥 = −3: 𝑥 + 2 = −3 + 2 = −1 = +𝑣𝑒
Similarly, we can conclude:
|𝑥 + 2| = −(𝑥 + 2), 𝑤ℎ𝑒𝑛 𝑥 < −2
Conclusion in Piece Wise Notation
The same thing can be written in piece-wise notation as follows:
𝑥 + 2, 𝑤ℎ𝑒𝑛 𝑥 > −2
|𝑥 + 2| = {
−(𝑥 + 2), 𝑤ℎ𝑒𝑛 𝑥 < −2

Second Critical Point


𝑥−3=0⇒𝑥 =3

𝑥 = 4 ⇒ 𝑥 − 3 = 4 − 3 = 1 = +𝑣𝑒 ⇒ |𝑥 − 3| = 𝑥 − 3, 𝑥 ∈ {3, ∞}
𝑥 = 0 ⇒ 𝑥 − 3 = 0 − 3 = −3 = −𝑣𝑒 ⇒ |𝑥 − 3| = −(𝑥 − 3), 𝑥 ∈ {∞, 3}

P a g e 55 | 97
Get all the files at: https://bit.ly/azizhandouts
Aziz Manva ([email protected])

We don’t need to write the piece wise notation here. It just expresses the same information in a more compact
manner.

Example 1.160
Find the length of the largest interval that satisfies (𝐴 ∪ 𝐵)′ over the domain 𝑈 ∈ (−3,3) given that:
𝐴: |3𝑥 + 4| < 2𝑥 + 5
𝐵: |4𝑥 − 5| < 3𝑥 + 2

Condition A 𝑥<7
Find the critical point 𝑥≤3
4 5
Case II: 𝑥 < 4:
3𝑥 + 4 > 0 ⇒ 𝑥 > −
3
4 −(4𝑥 − 5) < 3𝑥 + 2
Case I: 𝑥 > − : 4𝑥 − 5 > −3𝑥 − 2
3
3𝑥 + 4 < 2𝑥 + 5 ⇒ 𝑥 < 1 7𝑥 > 3
4
Case II: 𝑥 < − : 3
3 𝑥>
−(3𝑥 + 4) < 2𝑥 + 5 7
3
3𝑥 + 4 > −2𝑥 − 5 𝐵: ( , 3]
5𝑥 > −9 7
Combine the Conditions
9
𝑥>− 9
5 (𝐴 ∪ 𝐵) ∈ (− , 3]
9 5
𝐴: (− , 1) 9
5 (𝐴 ∪ 𝐵)′ ∈ [−3, − ]
Condition B 5
There is only a single interval, whose length is given
5
4𝑥 − 5 > 0 ⇒ 𝑥 > by:
4 9 −15 9 −6
5
Case I: 𝑥 > 4: |−3 − (− )| = | + | = | | = 1.2
5 5 5 5
4𝑥 − 5 < 3𝑥 + 2

E. Two Absolute Value Expressions


If you have more than one absolute value expression, you will need to determine the range over which expression
is positive or negative.
Then, you can will have to solve based on cases.

Example 1.161: Solving Using Inequalities


|𝑥 + 3| ≥ |6𝑥 + 9|
Strategy

P a g e 56 | 97
Get all the files at: https://bit.ly/azizhandouts
Aziz Manva ([email protected])

There are two ways of solving this.


➢ The first one, using inequalities, is
more conceptual, but also has more
involved calculations.
➢ The second one, uses equations
instead of inequalities. But it is
necessary to understand the method
behind it first.
Finding the Critical points
|𝑥 + 3| ≥ |6𝑥 + 9|
First, we find the regions where each absolute
value expressions is greater than zero.
𝑥 + 3 ≥ 0 ⇒ 𝑥 ≥ −3 ⇒ 𝐶𝑟𝑖𝑡𝑖𝑐𝑎𝑙 𝑃𝑜𝑖𝑛𝑡 = 𝑥 = −3
9 3 1
6𝑥 + 9 ≥ 0 ⇒ 6𝑥 ≥ −9 ⇒ 𝑥 ≥ − ⇒ 𝑥 ≥ − ⇒ 𝐶𝑟𝑖𝑡𝑖𝑐𝑎𝑙 𝑝𝑜𝑖𝑛𝑡: 𝑥 = −1
6 2 2
Finding the Intervals
The critical points divide the number line into three intervals
1 1
(−∞, −3 ) ∪ (−3, −1 ) ∪ (−1 , ∞)
2 2
Solve the inequality for each Interval
Case I: 𝒙 < −𝟑
6
−(𝑥 + 3) ≥ −(6𝑥 + 9) ⇒ 𝑥 + 3 ≤ 6𝑥 + 9 ⇒ −5𝑥 ≤ 6 ⇒ 𝑥 ≥ − ⇒ 𝑥 = 𝝓
5
𝟏 𝟕
Case II: −𝟑 < 𝒙 < −𝟏 ⇒ −𝟑 < 𝒙 < −𝟏
𝟐 𝟏𝟒
12 60
𝑥 + 3 ≥ −(6𝑥 + 9) ⇒ 𝑥 + 3 ≥ −6𝑥 − 9 ⇒ 7𝑥 ≥ −12 ⇒ 𝑥 ≥ − =−
7 35
𝟏 𝟓
Case III: 𝒙 > −𝟏 𝟐 ⇒ 𝒙 > −𝟏 𝟏𝟎
6 42
𝑥 + 3 ≥ 6𝑥 + 9 ⇒ −5𝑥 ≥ 6 ⇒ 𝑥 ≤ − = −
5 35
Merge the Cases
60 42 60 42 12 7
{𝜙} 𝑂𝑅 [− , ∞) 𝑂𝑅 (−∞, − ] ⇒ [− , − ] = [− , − ]
35 35 35 35 7 6

Example 1.162
Find the critical points of
A. |𝑥 + 3|
B. |6𝑥 + 9|

Find the Critical Points


𝑥 + 3 = 0 ⇒ 𝑥 = −3 ⇒ 𝐶𝑟𝑖𝑡𝑖𝑐𝑎𝑙 𝑃𝑜𝑖𝑛𝑡: 𝑥 = −3
9 3 1 1
6𝑥 + 9 = 0 ⇒ 𝑥 = − = − = −1 ⇒ 𝐶𝑟𝑖𝑡𝑖𝑐𝑎𝑙 𝑝𝑜𝑖𝑛𝑡𝑠: 𝑥 = −1
6 2 2 2
Find the positive and negative intervals
1 1
(−∞, −3) ∪ ⏟
𝑥 + 3: ⏟ (−3, ∞) , 6𝑥 + 9: (−∞, −1 ) ∪ (−1 , ∞)
⏟ 2 ⏟ 2
𝑵𝒆𝒈𝒂𝒕𝒊𝒗𝒆 𝑷𝒐𝒔𝒊𝒕𝒊𝒗𝒆
𝑵𝒆𝒈𝒂𝒕𝒊𝒗𝒆 𝑷𝒐𝒔𝒊𝒕𝒊𝒗𝒆

P a g e 57 | 97
Get all the files at: https://bit.ly/azizhandouts
Aziz Manva ([email protected])

Example 1.163: Solve using Critical Points


|𝑥 + 3| ≥ |6𝑥 + 9|
|𝑥 + 3| − |6𝑥 + 9| ≥ 0
Making the Cases
We want to see when the expressions inside the absolute value will be positive and negative.

𝑥+3
Positive Negative
6𝑥 + 9 Positive (𝑥
⏟ + 3) = (6𝑥 + 9) −(𝑥
⏟ + 3) = (6𝑥 + 9)
𝑰 𝑰𝑰
Negative (𝑥
⏟ + 3) = −(6𝑥 + 9) −(𝑥
⏟ + 3) = −(6𝑥 + 9)
𝑰𝑰𝑰 𝑰𝑽

𝐶𝑎𝑠𝑒 𝐼𝑉 𝑖𝑠 𝑡ℎ𝑒 𝑠𝑎𝑚𝑒 𝑎𝑠 𝐶𝑎𝑠𝑒 𝐼


𝐶𝑎𝑠𝑒 𝐼𝐼 𝑖𝑠 𝑡ℎ𝑒 𝑠𝑎𝑚𝑒 𝑎𝑠 𝐶𝑎𝑠𝑒 𝐼𝐼𝐼
Hence, we only to need to solve
𝐶𝑎𝑠𝑒 𝐼 𝑎𝑛𝑑 𝐶𝑎𝑠𝑒 𝐼𝐼
Finding the Critical Points
To find the critical points, we solve the equations that we found:
6 42
𝑥 + 3 = 6𝑥 + 9 ⇒ −6 = 5𝑥 ⇒ 𝑥 = − = −
5 35
12 60
𝑥 + 3 = −(6𝑥 + 9) ⇒ 𝑥 + 3 = −6𝑥 − 9 ⇒ 7𝑥 = −12 ⇒ 𝑥 = − =−
7 35
12 6
𝐶𝑟𝑖𝑡𝑖𝑐𝑎𝑙 𝑃𝑜𝑖𝑛𝑡𝑠 𝑎𝑟𝑒 − 𝑎𝑛𝑑 −
7 5
Finding the Intervals and checking them

12 12 6 6
(−∞, − ) ∪ (− , − ) ∪ (− , +∞)
⏟ 7 ⏟ 7 5 ⏟ 5
𝑰𝒏𝒕𝒆𝒓𝒗𝒂𝒍 𝑰 𝑰𝒏𝒕𝒆𝒓𝒗𝒂𝒍 𝑰𝑰 𝑰𝒏𝒕𝒆𝒓𝒗𝒂𝒍 𝑰𝑰𝑰
𝑵𝒆𝒈𝒂𝒕𝒊𝒗𝒆 𝑷𝒐𝒔𝒊𝒕𝒊𝒗𝒆 𝑵𝒆𝒈𝒂𝒕𝒊𝒗𝒆
Check Interval I by substituting −2:
|𝑥 + 3| − |6𝑥 + 9| = |−2 + 3| − |6(−2) + 9| = |1| − |−3| = 1 − 3 = −2 = −𝑣𝑒

Check Interval II by substituting −1.5:


|𝑥 + 3| − |6𝑥 + 9| = |−1.5 + 3| − |6(−1.5) + 9| = |1.5| − |0| = 1.5 = +𝑣𝑒

Check Interval III by substituting 𝑥 = 0:


|𝑥 + 3| − |6𝑥 + 9| = |0 + 3| − |6(0) + 9| = |3| − |9| = −6 = −𝑣𝑒

Final Solution
Reject the negative intervals and keep the positive intervals. Since the original inequality was

We will keep the endpoints.
12 6
[− , − ]
7 5
F. Compound Inequalities
Solving absolute value compound inequalities is a lengthy process. You will need to:
➢ Rewrite the compound inequality as two separate inequalities

P a g e 58 | 97
Get all the files at: https://bit.ly/azizhandouts
Aziz Manva ([email protected])

➢ Solve each inequality separately. Some of the inequalities may themselves have cases. Combine the solution
set for these cases
➢ Combine the final solution set using an intersection of solution sets.

Example 1.164
Solve
2 < |𝑥 − 1| ≤ 5

Approach
This is tricky because
➢ It is a compound inequality
➢ The middle term has absolute value
➢ The left-hand side has a < sign, which the right-hand side has a ≤ sign. In other words, from one part of it,
the endpoint is included, while from the other part, the endpoint is not included.

We have learnt how to handle each of these individually. But the chances of making a mistake go up as the number
of concepts to track increases.
Break up the compound inequality: Inequality I
2 < |𝑥 − 1|
Reverse the sign of the inequality:
|𝑥 − 1| > 2
Apply Property IV:
𝐶𝑎𝑠𝑒 𝐼: 𝑥 − 1 > 2 ⇒ 𝑥 > 3 ⇒ (3, ∞)
𝐶𝑎𝑠𝑒 𝐼𝐼: 𝑥 − 1 < −2 ⇒ 𝑥 < −1 ⇒ (−∞, −1)
(−∞,
⏟ −1) ∪ (3, ∞)
𝐹𝑟𝑜𝑚 𝐶𝑎𝑠𝑒 𝐼 𝑎𝑛𝑑 𝐼𝐼

Break up the compound inequality: Inequality II


|𝑥 − 1| ≤ 5 ⇒ −5 ≤ 𝑥 − 1 ≤ 5 ⇒ −4 ≤ 𝑥 ≤ 6 ⇒ 𝑥 ∈ [−4, 6]

Combine the solution sets using AND condition


Combine the two solution ranges. The final solution needs to satisfy both.
{(−∞, −1) ∪ (3, ∞)} ∩ [−4, 6] ⇒ [−4, −1) ∪ (3,6]

G. Nested Absolute Value

Example 1.165
Find the number of integer solutions to ||𝑥| − 2020| < 5. (IOQM 2021/5)

−5 < |𝑥| − 2020 < 5


2015 < |𝑥| < 2025
𝑃𝑜𝑠𝑖𝑡𝑖𝑣𝑒 𝑆𝑜𝑙𝑢𝑡𝑖𝑜𝑛𝑠: 𝑥 ∈ {2016,2017, … ,2024} → 2024 − 2016 + 1
𝑇𝑜𝑡𝑎𝑙 𝑆𝑜𝑙𝑢𝑡𝑖𝑜𝑛𝑠 = 9 × 2 = 18
Logic
||𝑥| − 2020| < 5 ⇒ |𝑥| − 2020 ∈ {−4, −3, … ,3,4} ⇒ 9 𝑉𝑎𝑙𝑢𝑒𝑠

|𝑥| − 2020 = −4 ⇒ |𝑥| = 2016 ⇒ 𝑥 = ±2016

P a g e 59 | 97
Get all the files at: https://bit.ly/azizhandouts
Aziz Manva ([email protected])

Similarly, each of the 9 values above has two solutions

𝑇𝑜𝑡𝑎𝑙 𝑆𝑜𝑙𝑢𝑡𝑖𝑜𝑛𝑠 = 9 × 2 = 18
Table

𝑥 2016 . . 2019 2020 2021 . . 2024


|𝑥| − 2020 −4 −1 0 1 4

P a g e 60 | 97
Get all the files at: https://bit.ly/azizhandouts
Aziz Manva ([email protected])

2. ABSOLUTE VALUE TOPICS


2.1 Quadratics
A. Type I Equations
Before you do this section, you should know Quadratics.

Example 2.1
|𝑥 2 + 5𝑥 + 5| = 1

A. Solve by Algebraic Method


B. Graph 𝑦 = |𝑥 2 + 5𝑥 + 5|

𝑥 2 + 5𝑥 + 5 = ±1

Case I: 𝑥 2 + 5𝑥 + 5 = 1
(𝑥 + 4)(𝑥 + 1) = 0
𝑥 ∈ {−1, −4}

Case II: 𝑥 2 + 5𝑥 + 5 = −1
(𝑥 + 2)(𝑥 + 3) = 0
𝑥 ∈ {−2, −3}

Combining the cases gives us:


𝑥 ∈ {−1, −2, −3, −4}

Example 2.2
|𝑥 2 + 5𝑥 + 5| = 𝑐, 𝑐∈ℝ

Determine the number of solutions of the equation above by considering various cases in the graph of
𝑦 = |𝑥 2 + 5𝑥 + 5|

𝑥 2 + 5𝑥 + 5 has vertex
𝑏 5
𝑥 𝑐𝑜𝑜𝑟𝑑𝑖𝑛𝑎𝑡𝑒 = −
=−
2𝑎 2
5 2 5 25 25 25 − 50 + 20 5
𝑦 𝑐𝑜𝑜𝑟𝑑𝑖𝑛𝑎𝑡𝑒 = (− ) + 5 (− ) + 5 = − +5= =−
2 2 4 2 4 4
5 5
𝑉𝑒𝑟𝑡𝑒𝑥 = (𝑥, 𝑦) = (− , − )
2 4

Since we are taking the absolute value, we need to take the absolute value of
the 𝑦 coordinate:
5 5
𝑉𝑒𝑟𝑡𝑒𝑥 = (𝑥, 𝑦) = (− , )
2 4

𝑐 < 0 ⇒ 𝑁𝑜 𝑆𝑜𝑙𝑢𝑡𝑖𝑜𝑛𝑠
𝑐 = 0 ⇒ 2 𝑆𝑜𝑙𝑢𝑡𝑖𝑜𝑛𝑠

P a g e 61 | 97
Get all the files at: https://bit.ly/azizhandouts
Aziz Manva ([email protected])

5
0 < 𝑐 < : 4 𝑆𝑜𝑙𝑢𝑡𝑖𝑜𝑛𝑠
4
5
𝑐 = : 3 𝑆𝑜𝑙𝑢𝑡𝑖𝑜𝑛𝑠
4
5
𝑐 > : 2 𝑆𝑜𝑙𝑢𝑡𝑖𝑜𝑛𝑠
4

Example 2.3
Determine the number of real solutions of 𝑦 = 𝑎𝑥 2 + 𝑏𝑥 + 𝑐 given that
|𝑎𝑥 2 + 𝑏𝑥 + 𝑐| = 𝑑, 𝑑>0
has either zero real solutions, one real solution, or two real solutions depending on the value of 𝑑.

If the equation had been of the type graphed in the previous question, then we
would have
𝑁𝑢𝑚𝑏𝑒𝑟 𝑜𝑓 𝑆𝑜𝑙𝑢𝑡𝑖𝑜𝑛𝑠 ∈ {0,2,3,4}

However, for the given equation, the possible number of solutions is:
𝑁𝑢𝑚𝑏𝑒𝑟 𝑜𝑓 𝑆𝑜𝑙𝑢𝑡𝑖𝑜𝑛𝑠 ∈ {0,1,2}

Hence, the graph of 𝑦 = 𝑎𝑥 2 + 𝑏𝑥 + 𝑐 must always be above the 𝑥 axis or always


be below the 𝑥 axis.
Since the graph of 𝑦 is always above the 𝑥 axis, it has:
𝑁𝑜 𝑅𝑒𝑎𝑙 𝑆𝑜𝑙𝑢𝑡𝑖𝑜𝑛𝑠

Example 2.4
𝐴𝑛𝑠𝑤𝑒𝑟 𝑒𝑎𝑐ℎ 𝑝𝑎𝑟𝑡 𝑠𝑒𝑝𝑎𝑟𝑎𝑡𝑒𝑙𝑦
Solve |(𝑥 + 1)2 + 5(𝑥 + 1) + 5| = 1 by using:
A. the algebraic method
B. transformation of functions

Part A
|(𝑥 + 1)2 + 5(𝑥 + 1) + 5| = 1
Use a change of variable. Let 𝑦 = 𝑥 + 1:
|𝑦 2 + 5𝑦 + 5| = 1

This is the same quadratic solved earlier in the chapter (with a different variable):
𝑦 ∈ {−1, −2, −3, −4}
𝑥 = 𝑦 − 1 ∈ {−2, −3, −4, −5}

Part B
Shift the graph of |(𝑥 + 1)2 + 5(𝑥 + 1) + 5| one unit to the right by substituting 𝑥 − 1 in place of 𝑥:
2
|((𝑥 − 1) + 1) + 5((𝑥 − 1) + 1) + 5| = 1
|𝑥 2 + 5𝑥 + 5| = 1

And then solve as in Part A.

P a g e 62 | 97
Get all the files at: https://bit.ly/azizhandouts
Aziz Manva ([email protected])

B. Type I Inequalities

Example 2.5
Solve
|𝑥 2 + 8𝑥 + 5| > 2

We split into cases using the property that


|𝑦| > 2 ⇒ 𝑦 > 2 𝑂𝑅 𝑦 < −2

Case I: 𝒙𝟐 + 𝟖𝒙 + 𝟓 > 𝟐
𝑥 2 + 8𝑥 + 3 > 0
−8 ± √64 − 12 −8 ± √52
𝑥= = = −4 ± √13
2 2
𝑥 ∈ (−∞, −4 − √13) ∪ (−4 + √13, ∞)

Case II: 𝒙𝟐 + 𝟖𝒙 + 𝟓 < −𝟐


𝑥 2 + 8𝑥 + 7 < 0
(𝑥 + 7)(𝑥 + 1) < 0
𝑥 ∈ (−7, −1)

The final answer is the union of the solutions from Case I and II:
(−∞, −4 − √13) ∪ (−7, −1) ∪ (−4 + √13, ∞)

Note that:
−4 + √13 > −4 + 3 = −1
−4 + √13 > −1

Example 2.6
Solve
|𝑥 2 + √47𝑥 − √53| ≤ −3

Absolute value always greater than zero


𝑥∈𝜙

Example 2.7
Solve
23 2 25
|√ 𝑥 + √23𝑥 + √ | > 0
4 4

Consider the discriminant of the quadratic expression:


2 √23 √25
𝐷 = 𝑏 2 − 4𝑎𝑐 = (√23) − 4 ( )( ) = 23 − √23√25 < 0
2 2

Hence, this quadratic is always positive:


𝐴𝑙𝑙 𝑟𝑒𝑎𝑙 𝑣𝑎𝑙𝑢𝑒𝑠

P a g e 63 | 97
Get all the files at: https://bit.ly/azizhandouts
Aziz Manva ([email protected])

Note: The coefficients of the quadratic are such that it is difficult to solve directly.

Example 2.8
Solve
17 2 29
|√ 𝑥 + √23𝑥 + √ | > 0
4 4

This is valid for all real numbers expect where:


17 29
√ 𝑥 2 + √23𝑥 + √ = 0
4 4

Applying the quadratic formula gives:


−√23 ± √23 − √17√29
𝑥=
2√17

Hence, the final answer is:


−√23 ± √23 − √17√29
𝑥 ∈ ℝ−{ }
2√17

And we can confirm the discriminant is positive in a couple of different ways:


2
(√17√29) = 17 × 29 = 493 < 529 = 232 ⇒ 23 − √17√29 > 0

Substitute 𝑎 = 17, 𝑏 = 29 in the 𝐴𝑀 − 𝐺𝑀 inequality 𝑎 + 𝑏 ≥ 2√𝑎𝑏


17 + 29 ≥ 2√17 ∙ √29
23 ≥ √17 ∙ √29
23 − √17 ∙ √29 ≥ 0
C. Type II Equations

Example 2.9
𝑥 2 + 5|𝑥| − 6 = 0

A. Solve the equation above by factoring


B. Considering cases for |𝑥|, and solving algebraically.
C. Uses the cases from Part B to graph the equation. Graph the “invalid” portions of the cases as well to see the
complete behaviour.

Part A
Use the property that |𝑥|2 = 𝑥 2
(|𝑥| + 6)(|𝑥| − 1) = 0
|𝑥| + 6 = 0 ⇒ |𝑥| = −6 ⇒ 𝑁𝑜 𝑆𝑜𝑙𝑢𝑡𝑖𝑜𝑛
|𝑥| − 1 = 0 ⇒ |𝑥| = 1 ⇒ 𝑥 = ±1

The final answer is:


𝑥 ∈ {−1, +1}
Part B

P a g e 64 | 97
Get all the files at: https://bit.ly/azizhandouts
Aziz Manva ([email protected])

Case I: 𝑥 > 0 ⇒ |𝑥| = 𝑥


𝑥 2 + 5𝑥 − 6 = 0 ⇒ (𝑥 + 6)(𝑥 − 1) = 0 ⇒ 𝑥 ∈ {−6, +1} ⇒ 𝑥 = +1

Case II: 𝑥 < 0 ⇒ |𝑥| = −𝑥


𝑥 2 − 5𝑥 − 6 = 0 ⇒ (𝑥 − 6)(𝑥 + 1) = 0 ⇒ 𝑥 ∈ {+6, −1} ⇒ 𝑥 = −1

The final answer is:


𝑥 ∈ {−1, +1}

Example 2.10
𝑥 2 − 5|𝑥| + 6 = 0

(|𝑥| − 2)(|𝑥| − 3) = 0
|𝑥| = 2 ⇒ 𝑥 = ±2
|𝑥| = 3 ⇒ 𝑥 = ±3

Example 2.11
𝑥 2 − 5|𝑥| − 6 = 0

(|𝑥| − 6)(|𝑥| + 1) = 0
|𝑥| = 6 ⇒ 𝑥 = ±6
|𝑥| = −1 ⇒ 𝑁𝑜 𝑆𝑜𝑙𝑢𝑡𝑖𝑜𝑛

Example 2.12
𝑥 2 + 5|𝑥| − 2 = 0

Using the quadratic formula


−5 ± √52 − 4(−2) −5 ± √33
|𝑥| = =
2 2

5 + √33
|𝑥| = − ⇒ 𝑁𝑜 𝑆𝑜𝑙𝑢𝑡𝑖𝑜𝑛𝑠
2
5 − √33 √33 − 5 √33 − 5
|𝑥| = − = ⇒𝑥=±
2 2 2

Example 2.13
𝑥 2 + 5|𝑥| + 3 = 0

Using the quadratic formula:


−5 ± √52 − 4(3) −5 ± √13
𝑥= =
2 2

5 + √13
|𝑥| = − ⇒ 𝑁𝑜 𝑆𝑜𝑙𝑢𝑡𝑖𝑜𝑛𝑠
2
5 − √13
|𝑥| = − ≈ −1.4 ⇒ 𝑁𝑜 𝑆𝑜𝑙𝑢𝑡𝑖𝑜𝑛𝑠
2
D. Type II Inequalities

P a g e 65 | 97
Get all the files at: https://bit.ly/azizhandouts
Aziz Manva ([email protected])

2.14: Quadratic Inequalities with Absolute Value


𝑥 2 + 𝑏|𝑥| + 𝑐, 𝑏>0

Example 2.15
Solve
𝑥 2 + 18|𝑥| + 77 ≤ 0

Always greater than zero because:


𝑥2 ≥ 0
18|𝑥| ≥ 0
77 > 0

𝑁𝑜 𝑆𝑜𝑙𝑢𝑡𝑖𝑜𝑛𝑠

Example 2.16
Solve
𝑥 2 + 5√3|𝑥| + 18 ≥ 0

Always greater than zero.


𝐴𝑙𝑙 𝑟𝑒𝑎𝑙 𝑛𝑢𝑚𝑏𝑒𝑟𝑠 ⇒ 𝑥 ∈ ℝ

Example 2.17
Solve
𝑥 2 − 5√3|𝑥| + 18 ≥ 0

(|𝑥| − 2√3)(|𝑥| − 3√3) = 0


𝑥 ∈ {−3√3, −2√3, 2√3, 3√3}

(−∞, −3√3] ∪ [−2√3, 2√3] ∪ [3√3, ∞)

Example 2.18
𝑥 2 + 3|𝑥| − 1 > 0

P a g e 66 | 97
Get all the files at: https://bit.ly/azizhandouts
Aziz Manva ([email protected])

√13 − 3 √13 − 3
(−∞, − )∪( , ∞)
2 2

Example 2.19
𝑥 2 − 5|𝑥| + 6 > 0

(|𝑥| − 2)(|𝑥| − 3) = 0
|𝑥| = 2 ⇒ 𝑥 = ±2
|𝑥| = 3 ⇒ 𝑥 = ±3

(−∞, −3) ∪ (−2, +2) ∪ (3, ∞)

E. Mixing Type I and Type II

Example 2.20
|𝑥 2 − 5|𝑥| + 5| ≥ 1

Solve the corresponding equation |𝑥 2 − 5|𝑥| + 5| = 1 into cases:


𝑥 2 − 5|𝑥| + 5 = 1 ⇒ (|𝑥| − 4)(|𝑥| − 1) = 0 ⇒ 𝑥 ∈ {−4, −1,1,4}
𝑥 2 − 5|𝑥| + 5 = −1 ⇒ (|𝑥| − 2)(|𝑥| − 3) = 0 ⇒ 𝑥 ∈ {−3, −2,2,3}

𝑥 ∈ {−4, −3, −2, −1,1,2,3,4}

Use the wavy curve method:

The final answer is:

P a g e 67 | 97
Get all the files at: https://bit.ly/azizhandouts
Aziz Manva ([email protected])

(−∞, −4] ∪ [−3, −2] ∪ [−1,1] ∪ [2,3] ∪ [4, ∞)

2.2 Logarithms
A. Equations
Before you do this section, you should know Logarithms.

2.21: Absolute Value Equations


|𝑥| = 𝑎, 𝑎 ∈ ℝ
𝑎 > 0 ⇒ 𝑥 = ±𝑎
𝑎=0⇒𝑥=0
𝑎<0⇒𝑥∈𝜙

|𝑥| = 5 ⇒ 𝑥 = ±5
|𝑥| = 0 ⇒ 𝑥 = 0
|𝑥| = −5 ⇒ 𝑥 ∈ 𝜙

Example 2.22
Solve for 𝑥 in each part.

Absolute Value in the Number


A. log 5 |𝑥 + 2| = 2
Absolute Value in the Answer
B. log 3 𝑥 = 𝑦, |𝑦| = 2
Absolute Value in the Base
C. log |𝑥| 4 = 3
Absolute Value Terms
D. |log 𝑥 4| = 2

Absolute Value in the Number


log 5|𝑥 + 2| = 2
Convert to exponential form
|𝑥 + 2| = 52 = 25
𝑥 + 2 = ±25
𝑥 ∈ {−27,23}
Absolute Value in the Answer
log 3 𝑥 = 𝑦, |𝑦| = 2
1
𝑥 = 3𝑦 = 3±2 ⇒ 𝑥 ∈ { , 9}
9
Absolute Value in the Base
log |𝑥| 4 = 3
|𝑥|3 = 4
Take cube roots:
3
|𝑥| = √4
3
𝑥 = ± √4
Absolute Value Terms
log 𝑥 4 = ±2
Convert to exponential form:

P a g e 68 | 97
Get all the files at: https://bit.ly/azizhandouts
Aziz Manva ([email protected])

𝑥 ±2 = 4
2
𝑥 = 4 ⇒ 𝑥 = {+2, −𝟐} ⇒ 𝑥 = 2
1 1 1
2
= 4 ⇒ 𝑥2 = ⇒ 𝑥 = ±
𝑥 4 2

Since is the base, 𝑥 > 0:


1
𝑥=
2

Example 2.23: Absolute Value


𝑎
A. The product of the solutions to log 4|𝑥 + 1| = −7 can be written in the form 𝑏 for integers 𝑎, 𝑏 with no
common factor other than 1. Find 𝑏 − 𝑎.
B. Find the sum of the solutions to |log 2 𝑥| + |log 3 𝑥| + ⋯ + |log 2022 𝑥| = −2022

Part A
log 4 |𝑥| = −7
1 1
|𝑥 + 1| = 4−7 = 7 = 14
4 2
1
𝑥 + 1 = ± 14
2
1
𝑥 = ± 14 − 1
2
1 1 − 214
𝑥 = 14 − 1 =
2 214
1 1 + 214
𝑥 = − 14 − 1 = −
2 214
1 + 214 1 − 214
𝑥 ∈ {− , }
214 214
(1 + 214 ) (1 − 214 )
𝑃𝑟𝑜𝑑𝑢𝑐𝑡 = − ×
214 214
28 28
1−2 2 −1
= 28
=
2 228
28 28
𝑏 − 𝑎 = 2 − (2 − 1) = 1
Part B
𝐿𝐻𝑆 ≥ 0
𝑅𝐻𝑆 < 0
𝐶𝑜𝑛𝑡𝑟𝑎𝑑𝑖𝑐𝑡𝑖𝑜𝑛
𝑥 ∈ {𝜙}

Sum of empty set is zero (by convention)


𝐴𝑛𝑠𝑤𝑒𝑟 = 0

Example 2.24: Absolute Value


If |𝑥 − log 𝑦| = 𝑥 + log 𝑦 where 𝑥 and log 𝑦 are real, then
A. 𝑥 = 0
B. 𝑦 = 1
C. 𝑥 = 0 and 𝑦 = 1
D. 𝑥(𝑦 − 1) = 0
E. None of these (AHSME 1972/8)

P a g e 69 | 97
Get all the files at: https://bit.ly/azizhandouts
Aziz Manva ([email protected])

𝑥 − log 𝑦 = ±(𝑥 + log 𝑦)


Case I:
𝑥 − log 𝑦 = 𝑥 + log 𝑦
− log 𝑦 = log 𝑦
2 log 𝑦 = 0
log 𝑦 = 0
𝑦=1
Case II:
𝑥 − log 𝑦 = −𝑥 − log 𝑦
𝑥 = −𝑥
2𝑥 = 0
𝑥=0
We combine the two cases to get:
𝑦 = 1 𝑂𝑅 𝑥 = 0
𝑦 − 1 = 0 𝑂𝑅 𝑥 = 0
And multiply the two equations to get:
𝑥(𝑦 − 1) = 0
B. Inequalities

Example 2.25
log 3(|𝑥 2 + 5𝑥 + 3|) > 1

|𝑥 2 + 5𝑥 + 3| > 0

|𝑥 2 + 5𝑥 + 3| > 3
𝑥 2 + 5𝑥 + 3 > 3
𝑥(𝑥 + 5) > 0
𝑥 ∈ (−∞, −5) ∪ (0, ∞)

𝑥 2 + 5𝑥 + 3 < −3
𝑥 2 + 5𝑥 + 6 < 0
(𝑥 + 2)(𝑥 + 3) < 0
𝑥 ∈ (−3, −2)

(−∞, −5) ∪ (−2, −3) ∪ (0, ∞)

Example 2.26
Given that 𝑦 = ln|𝑥| determine the solution set of

P a g e 70 | 97
Get all the files at: https://bit.ly/azizhandouts
Aziz Manva ([email protected])

|𝑦| > 1

Case I:
𝐶𝑎𝑠𝑒 𝐼: 𝑦 > 1 ⇒ ln|𝑥| > 1 ⇒ |𝑥| > 𝑒 ⇒ 𝑥 > 𝑒 𝑂𝑅 𝑥 < −𝑒

Case II:
1 1 1
𝑦 < −1 ⇒ ln|𝑥| < −1 ⇒ |𝑥| < ⇒− <𝑥<
𝑒 𝑒 𝑒

Final answer is the union of the above:


1 1
(−∞, −𝑒) ∪ (− , ) ∪ (𝑒, ∞)
𝑒 𝑒

2.3 Trigonometry
A. Inequalities versus Equalities

Example 2.27
Explain why the “solution below” is not correct:
𝜋 1 1 𝜋
0 < 𝜃 < : cos 𝜃 > ⇒ 𝜃 > cos−1 ⇒ 𝜃 >
2 2 2 3

𝐴𝑠 𝜃 ↑, cos 𝜃 ↓

B. Basic Inequalities

Example 2.28
Solve in radians:
1
sin 𝜃 cos 𝜃 >
4

Use the double angle identity:


1 1
sin 2𝜃 > ⇒ (sin 2𝜃 − ) > 0
2 2
360 360
Note that sin 2𝜃 has period 𝑏
= 2
= 180°

P a g e 71 | 97
Get all the files at: https://bit.ly/azizhandouts
Aziz Manva ([email protected])

Find the roots over [0,180°):


1
sin 2𝜃 − =0
2
1
sin 2𝜃 =
2
2𝜃 ∈ {30°, 150°}
𝜃 ∈ {15°, 75°}

The solution in the interval [0,180°)


(15°, 75°)

The general solution is:


(15°, 75°) + 180𝑛, 𝑛∈ℤ

Convert to radians:
𝜋 5𝜋
( , ) + 𝜋𝑛, 𝑛∈ℤ
12 12
C. Absolute Value Inequalities

Example 2.29
Solve in radians:
1
|cos 𝜃| >
2

1 1
cos 𝜃 > 𝑂𝑅 cos 𝜃 < −
2 2

(−60°, 60°) ∪ (120°, 240°) + 360𝑛, 𝑛∈ℤ

Convert to radians:
𝜋 𝜋 2𝜋 4𝜋
(− , ) ∪ ( , ) + 2𝜋𝑛, 𝑛∈ℤ
3 3 3 3

Example 2.30
Solve in radians:
1
|sin 2𝜃| >
2

D. Quadratic Inequalities

Example 2.31
Solve in radians:
2 sin 2𝜃 − 2 sin 𝜃 − 2 cos 𝜃 > −1

Factor the Inequality


Collate all terms on the LHS, and substitute sin 2𝜃 = 2 sin 𝜃 cos 𝜃
4 sin 𝜃 cos 𝜃 − 2 sin 𝜃 − 2 cos 𝜃 + 1 > 0
2 sin 𝜃 (2 cos 𝜃 − 1) − 1(2 cos 𝜃 − 1) > 0

P a g e 72 | 97
Get all the files at: https://bit.ly/azizhandouts
Aziz Manva ([email protected])

(2 cos 𝜃 − 1)(2 sin 𝜃 − 1) > 0

Principal Solutions
Note that both sin 𝜃 and cos 𝜃 have period 360°. Hence, we will find principal solutions, and then extend to the
real number line.

Find the roots of the expression on the LHS. Solve in degrees first:
1
2 sin 𝜃 − 1 = 0 ⇒ sin 𝜃 = ⇒ 𝑥 ∈ {30°, 150°}
2
1
2 cos 𝜃 − 1 = 0 ⇒ cos 𝜃 = ⇒ 𝑥 ∈ {60°, 300°}
2

The roots are:


{30°, 60°, 150°, 300°}

Check a point in the interval [0,30°]. Use 𝜃 = 0:


4 sin 0 cos 0 − 2 sin 0 − 2 cos 0 + 1 = 4(0) cos 0 − 2(0) − 2(1) + 1 = −2 + 1 = −1 < 0

(30°, 60°) ∪ (150°, 300°)

General Solution
Extend the principal solution to a general solution by using the periodicity (360°) of the trigonometric
functions involved:
{(30°, 60°) ∪ (150°, 300°)} + 360𝑛°, 𝑛∈ℤ
Convert to radians:
𝜋 𝜋 5𝜋 5𝜋
{( , ) ∪ ( , )} + 2𝜋𝑘, 𝑘∈ℤ
6 3 6 3
E. Strange Bases

Example 2.32
Solve the inequality
1
< |log cos 𝑥 sin 𝑥| < 2
2

Domain of the Expression


Determine the domain of the expression. The input for the log function must be positive:
sin 𝑥 > 0 ⇒ 𝑥 ∈ (0, 𝜋)
𝜋 3𝜋
cos 𝑥 > 0 ⇒ (0, ) ∪ ( , 2𝜋)
2 2

P a g e 73 | 97
Get all the files at: https://bit.ly/azizhandouts
Aziz Manva ([email protected])

Take the intersection of the above two to get the principal solution and then add 2𝜋𝑘 to get the general solution:
𝜋
(0, ) + 2𝜋𝑘, 𝑘∈ℤ
2
Determine log cos 𝑥 sin 𝑥 < 𝟎
𝜋
Consider the graphs of sin 𝑥 and cos 𝑥 in the domain (0, 2 ):
𝜋
𝑥 ∈ (0, ) ⇒ cos 𝑥 ∈ (0,1)
2

Since cos 𝑥 ∈ (0,1)


log cos 𝑥 sin 𝑥 < 0 𝑤ℎ𝑒𝑛 sin 𝑥 > 1 ⇒ 𝑥 ∈ 𝜙

Alternatively solve the inequality log cos 𝑥 sin 𝑥 < 0 by considering the critical points:
𝜋
log cos 𝑥 sin 𝑥 = 0 ⇒ sin 𝑥 = 1 ⇒ 𝑥 = ⇒ 𝑁𝑜𝑡 𝑖𝑛 𝑑𝑜𝑚𝑎𝑖𝑛 ⇒ 𝑥 ∈ 𝜙
2

Since log cos 𝑥 sin 𝑥 > 0 for all values of 𝑥, we can remove the absolute value:
1
< log cos 𝑥 sin 𝑥 < 2
2

Split the Inequality: Part I


1
log cos 𝑥 sin 𝑥 >
2

Consider the critical points:


1
log cos 𝑥 sin 𝑥 =
2
sin 𝑥 = √cos 𝑥
sin2 𝑥 = cos 𝑥
1 − cos 2 𝑥 = cos 𝑥
Substitute 𝑦 = cos 𝑥:
𝑦2 + 𝑦 − 1 = 0
1 − √5 1 + √5
cos 𝑥 = 𝑦 ∈ {− ,− }
2 2
1+√5
− 2
is not valid. Let:
1 − √5 √5 − 1
− = ≈ 0.61 𝑖𝑠 𝑣𝑎𝑙𝑖𝑑
2 2
√5 − 1 √5 − 1
cos 𝑥 = ⇒ 𝑥 = cos −1
2 2
𝜋
(And we only need solutions in the first quadrant since 𝑥 ∈ (0, 2 ))

Split the Inequality: Part II


log cos 𝑥 sin 𝑥 < 2

Consider the critical points:


log cos 𝑥 sin 𝑥 = 2
sin 𝑥 = cos2 𝑥
1 − sin2 𝑥 = sin 𝑥
Substitute 𝑦 = sin 𝑥:

P a g e 74 | 97
Get all the files at: https://bit.ly/azizhandouts
Aziz Manva ([email protected])

𝑦2 + 𝑦 − 1 = 0
1 − √5 1 + √5
sin 𝑥 = 𝑦 ∈ {− ,− }
2 2
1+√5
− 2
is not valid. Let:
1 − √5 √5 − 1
− = ≈ 0.61 𝑖𝑠 𝑣𝑎𝑙𝑖𝑑
2 2

√5 − 1 √5 − 1
sin 𝑥 = ⇒ 𝑥 = sin−1
2 2
𝜋
(And we only need solutions in the first quadrant since 𝑥 ∈ (0, ))
2

Consider
𝑥 = 0.001 ⇒ sin 𝑥 ≈ 0.001, cos 𝑥 ≈ 0.999
ln 0.001 𝑆𝑜𝑚𝑒 𝑁𝑢𝑚𝑏𝑒𝑟
log 0.999 0.001 = = → 𝑆𝑜𝑚𝑒 𝐿𝑎𝑟𝑔𝑒 𝑁𝑢𝑚𝑏𝑒𝑟
ln 0.999 𝑆𝑜𝑚𝑒 𝑁𝑢𝑚𝑏𝑒𝑟 𝑤𝑖𝑡ℎ 𝑆𝑚𝑎𝑙𝑙 𝐴𝑏𝑠𝑜𝑙𝑢𝑡𝑒 𝑉𝑎𝑙𝑢𝑒

𝜋
𝑥= − 0.001 ⇒ sin 𝑥 ≈ 0.999, cos 𝑥 ≈ 0.001
2
log 0.001 0.999 → 𝑆𝑜𝑚𝑒 𝑝𝑜𝑠𝑖𝑡𝑖𝑣𝑒 𝑛𝑢𝑚𝑏𝑒𝑟 𝑐𝑙𝑜𝑠𝑒 𝑡𝑜 𝑧𝑒𝑟𝑜

√5 − 1 √5 − 1
(sin−1 , cos−1 )
2 2

F. Nested Functions

Example 2.33
Solve the inequality
1
|sin(cos 𝑥)| >
2

Graph 𝑓(𝑥) = cos 𝑥:

P a g e 75 | 97
Get all the files at: https://bit.ly/azizhandouts
Aziz Manva ([email protected])

Graph 𝑓(𝑥) = sin(cos 𝑥):


cos 𝑥 has 𝑚𝑎𝑥𝑖𝑚𝑢𝑚 = 1 at 0, and 𝑚𝑖𝑛𝑖𝑚𝑢𝑚 = −1 at 𝜋:
√3 1.7
sin(cos 0) = sin 1 ≈ sin 57° ≈ sin 60° = ≈ = 0.85
2 2
√3 1.7
sin(cos 𝜋) = sin −1 ≈ sin −57° ≈ sin −60° = − ≈− = −0.85
2 2
𝜋 3𝜋
sin (cos ) = sin (cos ) = sin 0 ≈ 0
2 2

sin 𝑥 𝑖𝑠 𝑑𝑒𝑐𝑟𝑒𝑎𝑠𝑖𝑛𝑔 𝑎𝑛𝑑 𝑐𝑜𝑛𝑡𝑖𝑛𝑢𝑜𝑢𝑠 𝑓𝑟𝑜𝑚 sin 1 𝑡𝑜 sin −1


sin 𝑥 𝑖𝑠 𝑖𝑛𝑐𝑟𝑒𝑎𝑠𝑖𝑛𝑔 𝑎𝑛𝑑 𝑐𝑜𝑛𝑡𝑖𝑛𝑢𝑜𝑢𝑠 𝑓𝑟𝑜𝑚 sin −1 𝑡𝑜 sin 1

Split the given inequality into cases by removing the absolute value:
1 1
sin(cos 𝑥) > 𝑂𝑅 sin(cos 𝑥) < −
2 2
The regions that we want are marked in violet on the graph.

P a g e 76 | 97
Get all the files at: https://bit.ly/azizhandouts
Aziz Manva ([email protected])
1
Case I: sin(cos 𝑥) > 2
Solve the corresponding equation to find the critical points:
1 𝜋 5𝜋
sin(cos 𝑥) = ⇒ cos 𝑥 ∈ { , }
2 6 6
5𝜋 5(3)
≈ = 2.5 ⇒ 𝑁𝑜𝑡 𝑎 𝑣𝑎𝑙𝑖𝑑 𝑣𝑎𝑙𝑢𝑒 𝑜𝑓 cos 𝑥
6 6

Also, from the graph, we know that we are looking for a solution in the first and the fourth quadrant.
𝜋 𝜋 𝜋
cos 𝑥 = ⇒ 𝑥 ∈ {cos −1 ( ) , −cos −1 ( )}
6 6 6

Since this gives solutions which are in the principal domain, add 2𝜋 to the second solution:
𝜋 𝜋
𝑥 ∈ {cos −1 ( ) , 2𝜋 − cos−1 ( )}
6 6
The principal solution from Case I is:
𝜋 𝜋
[0, cos −1 ( )) ∪ (2𝜋 − cos −1 ( ) , 2𝜋)
6 6
The general solution from Case I is:
𝜋 𝜋
[0, cos−1 ( )) ∪ (2𝜋 − cos−1 ( ) , 2𝜋) + 2𝜋𝑘, 𝑘∈ℤ
6 6
1
Case II: sin(cos 𝑥) < − 2
Solve the corresponding equation to find the critical points:
1 7𝜋 11𝜋
sin(cos 𝑥) = − ⇒ cos 𝑥 ∈ { , }
2 6 6
7𝜋 7(3)
≈ ≈ 3 ⇒ 𝑁𝑜𝑡 𝑎 𝑣𝑎𝑙𝑖𝑑 𝑣𝑎𝑙𝑢𝑒 𝑜𝑓 cos 𝑥
6 6

11𝜋 11𝜋 𝜋
= − 2𝜋 = −
6 6 6
Also, from the graph, we know that we are looking for a solution in the first and the fourth quadrant.
𝜋 𝜋
cos 𝑥 = − ⇒ 𝑥 ∈ {cos−1 (− ) , }
6 6
The principal solution from Case I is:
𝜋 𝜋
(cos−1 (− ) , 2𝜋 − cos−1 (− ))
6 6
The general solution from Case I is:
𝜋 𝜋
(cos−1 (− ) , 2𝜋 − cos−1 (− )) + 2𝜋𝑘, 𝑘∈ℤ
6 6

Combine Case I and Case II:


Combine the cases to get:
𝜋 𝜋 𝜋 𝜋
[0, cos −1 ( )) ∪ (cos −1 (− ) , 2𝜋 − cos−1 (− )) ∪ (2𝜋 − cos−1 ( ) , 2𝜋) + 2𝜋𝑘, 𝑘∈ℤ
6 6 6 6

P a g e 77 | 97
Get all the files at: https://bit.ly/azizhandouts
Aziz Manva ([email protected])

P a g e 78 | 97
Get all the files at: https://bit.ly/azizhandouts
Aziz Manva ([email protected])

3. “NAMED” INEQUALITIES
3.1 Some Basics
A. Reciprocals in Inequalities

Example 3.1
Let 𝑝 > 𝑥 > 𝑦 > 𝑞 > 0.
𝑝 𝑥
A. Compare 𝑞 with 𝑦.
𝑞 𝑦
B. Compare 𝑝 with 𝑥 .
C. Does your answer change if 𝑝 < 𝑥 < 𝑦 < 𝑞 < 0.

𝑝 𝑥
>
𝑞 𝑦
𝑞 𝑦
<
𝑝 𝑥

Answer does not change.

B. Creating Numerators

Example 3.2
𝑝 𝑥
Given > , show that
𝑞 𝑦
𝑝−𝑞 𝑥−𝑦
>
𝑞 𝑦

Subtract one from both sides of the inequality.


𝑝 𝑥
−1> −1
𝑞 𝑦
This simplifies to the desired results when we combine fractions:
𝑝−𝑞 𝑥−𝑦
>
𝑞 𝑦

C. Moving all terms to one side

Example 3.3
If 𝑝 > 𝑞 and 𝑥 > 𝑦, then show that
𝑝𝑥 + 𝑞𝑦 > 𝑝𝑦 + 𝑞𝑥

(𝑝 − 𝑞)(𝑥 − 𝑦) > 0 ⇒ 𝑝𝑥 − 𝑝𝑦 − 𝑞𝑥 + 𝑞𝑦 > 0 ⇒ 𝑝𝑥 + 𝑞𝑦 > 𝑝𝑦 + 𝑞𝑥

D. Positive Numbers

Example 3.4
If 𝑎 < 𝑏, when can we say that
𝑎𝑥 < 𝑏𝑥

P a g e 79 | 97
Get all the files at: https://bit.ly/azizhandouts
Aziz Manva ([email protected])

If 𝑥 is positive.

Example 3.5
Show that for positive real numbers p, q and k:
𝑝 𝑝+𝑘
< 𝑖𝑓 𝑎𝑛𝑑 𝑜𝑛𝑙𝑦 𝑖𝑓 𝑝 < 𝑞
𝑞 𝑞+𝑘

𝑝𝑞 + 𝑝𝑘 < 𝑞𝑝 + 𝑞𝑘 ⇔ (𝑝 − 𝑞)𝑘 < 0


Since 𝑘 is positive, this is only possible if 𝑝 < 𝑞.

E. Applications

Example 3.6
The number of elements in the set (JEE Main, July 22, 2021-II)
{𝑛 ∈ {1,2,3, … ,100}, 11𝑛 > 10𝑛 + 9𝑛 }

Divide both sides of the given inequality by 11𝑛 :


10 𝑛 9 𝑛
1>( ) +( )
⏟ 11 11
𝑰𝒏𝒆𝒒𝒖𝒂𝒍𝒊𝒕𝒚 𝑰

10 𝑛 9 𝑛
As 𝑛 increases, ( ) + ( ) decreases. Hence, the inequality is satisfied for some value of 𝑛, and once it is
11 11
satisfied, then all larger values of 𝑛 also satisfy it.
The smallest value of 𝑛 such that Inequality I is satisfied is:
𝑛 = 1: 11 − 9 = 2 < 10
𝑛 = 2: 121 − 81 = 40 < 100
𝑛 = 3: 1331 − 729 < 1000
𝑛 = 4: 14641 − 6561 < 10,000
𝑛 = 5: 161051 − 59049 = 1,02,002 > 1,00,000

{5,6,7, … ,100} ⇒ 100 − 5 + 1 = 96 𝑉𝑎𝑙𝑢𝑒𝑠

Example 3.7
𝑎𝑛 > 𝑐1 𝑏1𝑛 + 𝑐2 𝑏2𝑛 + ⋯ + 𝑐𝑥 𝑏𝑥𝑛
Where
𝑐1 , 𝑐2 , … , 𝑐𝑥 𝑎𝑟𝑒 𝑐𝑜𝑛𝑠𝑡𝑎𝑛𝑡𝑠
𝑎 > 𝑏𝑖 , 1 ≤ 𝑖 ≤ 𝑥

Divide both sides by 𝑎𝑛 :


𝑏1 𝑛 𝑏2 𝑛 𝑏𝑥 𝑛
1 > 𝑐1 ( ) + 𝑐2 ( ) + ⋯ + 𝑐𝑥 ( )
𝑎 𝑎 𝑎

The above will be achieved at some value of 𝑛 and will hold for all values of 𝑛 thereafter.

P a g e 80 | 97
Get all the files at: https://bit.ly/azizhandouts
Aziz Manva ([email protected])

3.2 𝑹𝑴𝑺 ≥ 𝑨𝑴 ≥ 𝑮𝑴 ≥ 𝑯𝑴 (In)equality


A. Trivial Inequality

3.8: Trivial Inequality


The square of any real quantity is greater than or equal to zero:
𝑥2 ≥ 0

The trivial inequality forms the basis for proving many, many inequalities and solving many questions.

Example 3.9
What is the least possible value of (𝑥𝑦 − 1)2 + (𝑥 + 𝑦)2 for real numbers 𝑥 and 𝑦? (AMC 12A 2021/7, AMC 10A
2021/9)

𝑥 2 𝑦 2 − 2𝑥𝑦 + 1 + 𝑥 2 + 2𝑥𝑦 + 𝑦 2 = 𝑥 2 + 𝑦 2 + 𝑥 2 𝑦 2 + 1
𝑀𝑖𝑛𝑖𝑚𝑢𝑚 𝑜𝑓 𝑥 2 = 0 ⇒ 𝐴𝑐ℎ𝑖𝑒𝑣𝑒𝑑 𝑎𝑡 𝑥 = 0
𝑀𝑖𝑛𝑖𝑚𝑢𝑚 𝑜𝑓 𝑦 2 = 0 ⇒ 𝐴𝑐ℎ𝑖𝑒𝑣𝑒𝑑 𝑎𝑡 𝑦 = 0
𝐴𝑡 𝑥 = 𝑦 = 0 ⇒ 𝑥 2 𝑦 2 = 0

By the trivial inequality, we know that this is the best we can do for the first three terms.
Hence, the fourth term, which is a constant, is also the minimum.

Example 3.10
Identify the mistake in the “proof” below:
4 = 5 ⇒ 𝑀𝑢𝑙𝑡𝑖𝑝𝑙𝑦 𝑏𝑜𝑡ℎ 𝑠𝑖𝑑𝑒𝑠 𝑏𝑦 0 ⇒ 0 = 0

You cannot begin the proof by assuming the truth of what needs to be proved.

Example 3.11
Prove that:
A. 𝑎2 + 𝑏 2 ≥ 2𝑎𝑏
B. 𝑎2 + 𝑏 2 + 𝑐 2 ≥ 2𝑎𝑏 + 2𝑏𝑐 − 2𝑎𝑐
C. 𝑎2 + 𝑏 2 + 𝑐 2 ≥ 𝑎𝑏 + 𝑏𝑐 + 𝑎𝑐

Part A
Subtract 2𝑎𝑏 from both sides of the given inequality:
⇔ 𝑎2 − 2𝑎𝑏 + 𝑏 2 ≥ 0
⇔ (𝑎 − 𝑏)2 ≥ 0
Each step above is reversible, and hence the original equality holds.

Part B:
Expand, and move the required terms to RHS:
(𝑎 − 𝑏 + 𝑐)2 ≥ 0 ⇔ 𝑎2 + 𝑏 2 + 𝑐 2 ≥ 2𝑎𝑏 + 2𝑏𝑐 − 2𝑎𝑐
Part C:
(𝑎 − 𝑏)2 + (𝑏 − 𝑐)2 + (𝑐 − 𝑎)2 ≥ 0 ⇔ 𝑎2 + 𝑏 2 + 𝑐 2 ≥ 𝑎𝑏 + 𝑏𝑐 + 𝑎𝑐

3.12: Sum of a positive number and its reciprocal is always greater than 2.
For positive real numbers 𝑎 and 𝑏:

P a g e 81 | 97
Get all the files at: https://bit.ly/azizhandouts
Aziz Manva ([email protected])

𝑎 𝑏
+ ≥2
𝑏 𝑎

Add fractions:
𝑎2 + 𝑏 2
⇔ ≥2
𝑎𝑏
Clear the denominator. Note that we can multiply both sides by 𝒂, and 𝒃 only because both are positive:
⇔ 𝑎2 + 𝑏 2 ≥ 2𝑎𝑏
Collate all terms on one side and factor:
(𝑎 − 𝑏)2 ≥ 0
And the above is true by the trivial inequality.
(All steps are reversible, and hence the equality we started with is true.)

In the above, where was the condition that 𝑎 and 𝑏 are both positive used?

In one step, we multiplied by 𝑎𝑏.


If 𝑎𝑏 < 0, then the inequality will flip.

In the above, can 𝑎 and 𝑏 be both negative?

Yes.

3.13: Minimum Value of an expression


The minimum value of 𝑎2 + 𝑏 is:
𝑏

This is because, by the trivial inequality:


𝑎2 ≥ 0

Example 3.14: Completing the Square


Find the minimum and maximum value of 𝑙 2 + 𝑤 2 given that 𝑙 and 𝑤 are nonnegative real numbers and 𝑙 + 𝑤 =
2.

Let
𝑋 = 𝑙2 + 𝑤 2
Substitute 𝑙 + 𝑤 = 2 ⇒ 𝑙 = 2 − 𝑤 which is valid for 0 ≤ 𝑙, 𝑤 ≤ 2 and simplify:
(2 − 𝑤)2 + 𝑤 2 = 4 − 4𝑤 + 𝑤 2 + 𝑤 2 = 2𝑤 2 − 4𝑤 + 4 = 2(𝑤 2 − 2𝑤 + 2)
Complete the square to get:
2[(𝑤 2 − 1)2 + 1]
𝑀𝑖𝑛 𝑤ℎ𝑒𝑛: 𝑤 = 1 ⇒ 𝑋 = 2
𝑀𝑎𝑥 𝑤ℎ𝑒𝑛: 𝑤 = 2 ⇒ 𝑋 = 4

Example 3.15: Change of Variable


Let 𝑥 ≥ 0, 𝑦 ≥ 0 be real numbers with 𝑥 + 𝑦 = 2. Prove that 𝑥 2 𝑦 2 (𝑥 2 + 𝑦 2 ) ≤ 2. (Irish Math Olympiad
2000/P2/Q1)

We start with a change of variable that is not immediately recognizable as useful.


Let 𝑥 = 1 + 𝑡. Then
𝑥+𝑦 = 2⇒ 1+𝑡+𝑦 = 2⇒ 𝑦 =1−𝑡

P a g e 82 | 97
Get all the files at: https://bit.ly/azizhandouts
Aziz Manva ([email protected])

Substitute 𝑥 = 1 + 𝑡, 𝑦 = 1 − 𝑡 in 𝑥 2 𝑦 2 (𝑥 2 + 𝑦 2 ):
= (1 + 𝑡)2 (1 − 𝑡)2 [(1 + 𝑡)2 + (1 − 𝑡)2 ]
= (1 − 𝑡 2 )2 [2 + 2𝑡 2 ]
= 2(1 − 𝑡 2 )(1 − 𝑡 2 )[1 + 𝑡 2 ]
2 4
Since 𝑡 , 𝑡 ≥ 0:
=2 ⏟ (1 − 𝑡 2 ) (1 − 𝑡 4 )

𝑩𝒆𝒕𝒘𝒆𝒆𝒏 𝟎 𝒂𝒏𝒅 𝟏 𝑩𝒆𝒕𝒘𝒆𝒆𝒏 𝟎 𝒂𝒏𝒅 𝟏

B. AM-GM Inequality: Two Variables

3.16: Geometric Mean and Arithmetic Mean


𝐺𝑀(𝑎1 , 𝑎2 , … 𝑎𝑛 ) = 𝑛√𝑎1 × 𝑎2 × … × 𝑎𝑛
𝑎1 + 𝑎2 + ⋯ + 𝑎𝑛
𝐴𝑀(𝑎1 , 𝑎2 , … 𝑎𝑛 ) =
𝑛

The geometric mean is the 𝑛𝑡ℎ root of the product of 𝑛 numbers.


Square roots (and geometric mean) are only defined for positive numbers.
Hence, you cannot apply the AM-GM inequality to expressions that can also be negative.

Example 3.17
Consider a rectangle with perimeter 20 and integer side lengths. Find the value of the side lengths that
maximizes the area.

𝑃 = 2(𝑙 + 𝑤) = 20 ⇒ 𝑙 + 𝑤 = 10

Length 1 2 3 4 5 6 7 8 9
Width 9 8 7 6 5 4 3 2 1
Area 9 18 21 24 25 24 21 16 9

3.18: AM-GM Inequality


𝑎+𝑏
≥ √𝑎𝑏

⏟2
𝑮𝒆𝒐𝒎𝒆𝒕𝒓𝒊𝒄 𝑴𝒆𝒂𝒏
𝑨𝒓𝒊𝒕𝒉𝒎𝒆𝒕𝒊𝒄 𝑴𝒆𝒂𝒏

By the trivial inequality:


(𝑎 − 𝑏)2 ≥ 0
Square both sides:
𝑎2 − 2𝑎𝑏 + 𝑏 2 ≥ 0
Add 4𝑎𝑏 to both sides:
𝑎2 + 2𝑎𝑏 + 𝑏 2 ≥ 4𝑎𝑏
Take the square root both sides:
𝑎 + 𝑏 ≥ 2√𝑎𝑏
Divide by 2 both sides:
𝑎+𝑏
≥ √𝑎𝑏
2

3.19: AM-GM Equality


Equality of AM and GM is achieved 𝑖𝑓 𝑎𝑛𝑑 𝑜𝑛𝑙𝑦 𝑖𝑓 𝑎 = 𝑏.

P a g e 83 | 97
Get all the files at: https://bit.ly/azizhandouts
Aziz Manva ([email protected])

We first prove that 𝑎 = 𝑏 ⇒ 𝐴𝑀 = 𝐺𝑀:


𝑎 + 𝑎 2𝑎
𝐿𝐻𝑆 = = =𝑎
2 2
𝑅𝐻𝑆 = √𝑎𝑎 = √𝑎2 = 𝑎
Hence, equality is achieved when 𝑎 = 𝑏.

Now we prove the converse. If 𝐴𝑀 = 𝐺𝑀 ⇒ 𝑎 = 𝑏:


𝑎+𝑏
= √𝑎𝑏
2
Eliminate fractions:
𝑎 + 𝑏 = 2√𝑎𝑏
Square both sides:
(𝑎 + 𝑏)2 = 4𝑎𝑏
Collate all terms on one side:
𝑎2 − 2𝑎𝑏 + 𝑏 2 = 0
Factor:
(𝑎 − 𝑏)2 = 0
Take the square root:
𝑎−𝑏=0
𝑎=𝑏

Example 3.20
Find the minimum and maximum value of 𝑙 2 + 𝑤 2 given that 𝑙 and 𝑤 are nonnegative real numbers and 𝑙 + 𝑤 =
2.

Since 𝑙 and 𝑤 are nonnegative real numbers, we must have:


𝑙 ≥ 0, 𝑤 ≥ 0 ⇒ ⏟ 𝑙𝑤 ≥ 0
𝑰𝒏𝒆𝒒𝒖𝒂𝒍𝒊𝒕𝒚 𝑰
Use AM-GM on the given equality:
𝑙+𝑤 2
√𝑙𝑤 ≤ = =1⇒ ⏟𝑙𝑤 ≤ 1
2 2 𝑰𝒏𝒆𝒒𝒖𝒂𝒍𝒊𝒕𝒚 𝑰𝑰
Combine Inequality I and II:
0 ≤ 𝑙𝑤 ≤ 1
Complete the square on the expression we want to minimize:
𝑙 2 + 𝑤 2 = (𝑙 + 𝑤)2 − 2𝑙𝑤 = 22 − 2𝑙𝑤 = 4 − 2𝑙𝑤

This will be maximum when 𝑙𝑤 is minimum:


4−0=4
And it will be minimum when 𝑙𝑤 is maximum:
4 − 2𝑙𝑤 = 4 − 2(1) = 2

3.21: Finding Minimum


𝑎+𝑏
≥ √𝑎𝑏
2
Equality is achieved when
𝑎=𝑏

In the above inequality, if you are able to get a value on the RHS, it can be used to establish a lower bound for

P a g e 84 | 97
Get all the files at: https://bit.ly/azizhandouts
Aziz Manva ([email protected])

the LHS.

Example 3.22
A. Two positive real numbers 𝑎 and 𝑏 have a product of 100. What is the minimum value of their sum?
B. The area of a rectangle is 256 square meters. Find the minimum perimeter of the rectangle, in 𝑐𝑚.

Part A
𝑎+𝑏
≥ √𝑎𝑏 = √100 = 10 ⇒ 𝑎 + 𝑏 ≥ 20
2
Hence, the minimum value of the sum is:
20. 𝑤ℎ𝑒𝑛 𝑎 = 𝑏 = 10
Part B
𝑙+𝑤
≥ √𝑙𝑤 = √256 = 16 ⇒ 2(𝑙 + 𝑤) = 64
2
Minimum perimeter is 64, when
𝑙 = 𝑤 = 16

Example 3.23
9𝑥 2 sin2 𝑥+4
If 0 < 𝑥 < 𝜋, find the minimum value of 𝑥 sin 𝑥
(AIME 1983/9)

Use a change of variable. Let 𝑦 = 𝑥 sin 𝑥:


9𝑥 2 sin2 𝑥 + 4 9𝑦 2 + 4 4
= = 9𝑦 +
𝑥 sin 𝑥 𝑦 𝑦
The domain gives us a clue. Both 𝑥 and sin 𝑥 are positive in the interval 0 < 𝑥 < 𝜋. So, when we take the
product, the 𝑦’s will cancel.

Apply 𝑨𝑴 − 𝑮𝑴
4 4
9𝑦 + ≥ 2√9𝑦 × = 12
𝑦 𝑦
Equality holds when the two terms are equal:
4 2 2
9𝑦 = ⇒ 𝑦 = ⇒ 𝑥 sin 𝑥 =
𝑦 3 3
When 𝑥 = 0
𝜋 𝜋
𝑥 = 0 ⇒ 𝑥 sin 𝑥 = 0, 𝑥= ⇒ 𝑥 sin 𝑥 = ≈ 1.57
2 2
𝑥 sin 𝑥 is continuous.
2 𝜋
𝐻𝑒𝑛𝑐𝑒, 𝑖𝑡 𝑎𝑐ℎ𝑖𝑒𝑣𝑒𝑠 𝑏𝑒𝑡𝑤𝑒𝑒𝑛 𝑥 = 0, 𝑎𝑛𝑑 𝑥 =
3 2

Example 3.24
Find the dimensions of a rectangle with perimeter 𝑝 that maximize its area. Find that area.

Let
𝑝 𝑝
2(𝑙 + 𝑤) = 𝑝 ⇒ 𝑤 = − 𝑙 ⇒ 𝐴𝑟𝑒𝑎 = 𝑙 ( − 𝑙)
2 2
By AM-GM Inequality:
𝑝
𝑝 𝑙 + ( − 𝑙) 𝑝
√𝑙 ( − 𝑙) ≤ 2 =
2 2 4

P a g e 85 | 97
Get all the files at: https://bit.ly/azizhandouts
Aziz Manva ([email protected])

Square both sides:


𝑝 𝑝2
𝑙 ( − 𝑙) ≤
2 16
𝑝2
The maximum area (16) is achieved when:
𝑝 𝑝
𝑙=
−𝑙 ⇒𝑙 =
2 4
In other words, for a given perimeter, the area of a rectangle is maximized when it is a square.

Example 3.25
𝑦
1 𝑦
If log 2 𝑎 + log 2 𝑏 ≥ 4, then the minimum value of 𝑎 + 𝑏 can be written as 𝑥 𝑧 , where 𝑥 is a prime number, and 𝑧 is
an irreducible fraction. Find 𝑥 + 𝑦 + 𝑧.

Combine the logarithms using the product rule:


1
log 2 𝑎𝑏 ≥
4
Convert from logarithmic form to exponential form:
1
𝑎𝑏 ≥ 24
From the AM-GM inequality, we know that:
1 1 9
𝑎 + 𝑏 ≥ 2√𝑎𝑏 = 2√24 = 2 × 28 = 28
Finally:
𝑥 + 𝑦 + 𝑧 = 2 + 9 + 8 = 19

Example 3.26
Let 𝑥 ≥ 0, 𝑦 ≥ 0 be real numbers with 𝑥 + 𝑦 = 2. Prove that 𝑥 2 𝑦 2 (𝑥 2 + 𝑦 2 ) ≤ 2. (Irish Math Olympiad
2000/P2/Q1)2

Apply AM-GM on the given equality condition:


𝑥+𝑦
√𝑥𝑦 ≤ ⏟ 2 ≤ 𝑥𝑦 ≤ 1
= 1 ⇒ √𝑥𝑦 ≤ 1 ⇒ 𝑥𝑦 ≤ 1 ⇒ (𝑥𝑦)
2
𝑰𝒏𝒆𝒒𝒖𝒂𝒍𝒊𝒕𝒚 𝑰
The last inequality holds since the square of a number between 0 and 1 is less than the number (for example
0.92 = 0.81).

Work with the LHS of the inequality to be proved. Complete the square:
𝐿𝐻𝑆 = 𝑥 2 𝑦 2 (𝑥 2 + 𝑦 2 ) = 𝑥 2 𝑦 2 [(𝑥 + 𝑦)2 − 2𝑥𝑦]
Substitute 𝑥 + 𝑦 = 2, and factor:
= 𝑥 2 𝑦 2 [(2)2 − 2𝑥𝑦] = 2(𝑥𝑦)2 [2 − 𝑥𝑦]

Use Inequality I to create a chain of inequalities:


2(𝑥𝑦)2 [2 − 𝑥𝑦] ≤ 2𝑥𝑦[2 − 𝑥𝑦]
𝑎+𝑏 2
By AM-GM 𝑎𝑏 ≤ ( 2
) , which we apply to the last expression:
2
𝑥𝑦 + (2 − 𝑥𝑦) 2 2
⏟ [2
𝑥𝑦 ⏟ − 𝑥𝑦] ≤ ( ) = ( ) = (1)2 = 1
2 2
𝑎 𝑏
2𝑥𝑦[2 − 𝑥𝑦] ≤ 2

2
Also see the alternate solution that does not use the AM-GM inequality earlier in this document.

P a g e 86 | 97
Get all the files at: https://bit.ly/azizhandouts
Aziz Manva ([email protected])

Since 0 ≤ 𝑥𝑦 ≤ 1, we must have (𝑥𝑦)2 ≤ 𝑥𝑦:


2(𝑥𝑦)2 [2 − 𝑥𝑦] ≤ 2
And finally, we conclude:
𝑥 2 𝑦 2 (𝑥 2 + 𝑦 2 ) ≤ 2

Example 3.27
The number of elements in the set below is: (JEE Main, July 29, 2022-II)
𝑥2 + 𝑥
𝑆 = {𝑥 ∈ ℝ: 2 cos ( ) = 4𝑥 + 4−𝑥 }
6
Is:

𝑥2 + 𝑥 4𝑥 + 4−𝑥
cos ( )=
6 2

By AM-GM, the RHS has minimum value 1 since:


4𝑥 + 4−𝑥
≥ √4𝑥 ∙ 4−𝑥 = √4𝑥−𝑥 = √40 = √1 = 1
2

This value is achieved when:


4𝑥 = 4−𝑥 ⇒ 𝑥 = −𝑥 ⇒ 𝑥 = 0

The LHS has maximum value 1, and hence the only solution for equation is when:
𝐿𝐻𝑆 = 𝑅𝐻𝑆 = 1

The only solution that works for RHS is 𝑥 = 1, and this also work for the LHS.
𝑥2 + 𝑥 02 + 0
cos ( ) = cos ( ) = cos(0) = 1
6 6

Hence, there is exactly one element in S.

3.28: Cauchy Schwarz Inequality


𝑛 2 𝑛 𝑛

(∑ 𝑢𝑖 𝑣𝑖 ) ≤ (∑ 𝑢𝑖 ) (∑ 𝑣𝑖2 )
2

𝑖=1 𝑖=𝑖 𝑖=𝑖

➢ This is one of the most important and widely used inequalities in Maths.3
➢ It was first proved in the 19𝑡ℎ century.

Example 3.29: Proving the Two Variable Cauchy Schwarz4


A. Substitute 𝑛 = 2 in the Cauchy Schwarz equality to obtain the two-variable case of the inequality. Then
make the substitution 𝑢1 = 𝑎, 𝑣1 = 𝑏, 𝑢2 = 𝑐, 𝑣2 = 𝑑 to make the variables easier to work with.
B. Expand both sides of the inequality from Part A, and simplify, to get an equivalent inequality.
C. Apply the 𝐴𝑀 − 𝐺𝑀 inequality to prove Part B.

Part A

3
This inequality has a Vector form, and an integral form as well.
4
There are other ways of proving the general Cauchy Schwarz, which are not necessarily difficult. But our objective here is to
𝑎𝑝𝑝𝑙𝑦 the AM-GM inequality.

P a g e 87 | 97
Get all the files at: https://bit.ly/azizhandouts
Aziz Manva ([email protected])

Substitute 𝑛 = 2 to get:
(𝑢1 𝑣1 + 𝑢2 𝑣2 )2 ≤ (𝑢12 + 𝑢22 )(𝑣12 + 𝑣22 )
Substitute 𝑢1 = 𝑎, 𝑣1 = 𝑏, 𝑢2 = 𝑐, 𝑣2 = 𝑑
⇔ (𝑎𝑏 + 𝑐𝑑)2 ≤ (𝑎2 + 𝑐 2 )(𝑏2 + 𝑑2 )
Part B
⇔ 𝑎2 𝑏 2 + 2𝑎𝑏𝑐𝑑 + 𝑐 2 𝑑2 ≤ 𝑎2 𝑏 2 + 𝑎2 𝑑2 + 𝑐 2 𝑏2 + 𝑐 2 𝑑2
⇔ 2𝑎𝑏𝑐𝑑 ≤ 𝑎2 𝑑2 + 𝑐 2 𝑏2
Part C
𝑎2 𝑑 2 + 𝑐 2 𝑏 2
√𝑎2 𝑑2 𝑐 2 𝑏2 ≤
2
2|𝑎𝑏𝑐𝑑| ≤ 𝑎2 𝑑2 + 𝑐 2 𝑏2
2𝑎𝑏𝑐𝑑 ≤ 𝑎2 𝑑2 + 𝑐 2 𝑏 2

C. AM-GM Inequality: Three Variables

3.30: AM-GM: Three Variables


For positive real numbers 𝑎, 𝑏 and 𝑐:
𝑎+𝑏+𝑐 3
≥ √𝑎𝑏𝑐
3

We assume the four-variable case here.5


1 𝑤+𝑥+𝑦+𝑧
Substitute 𝑤 = 3 (𝑥 + 𝑦 + 𝑧) in 4
≥ 4√𝑤𝑥𝑦𝑧:
1 4
(𝑥 + 𝑦 + 𝑧) + 𝑥 + 𝑦 + 𝑧 (𝑥 + 𝑦 + 𝑧) 𝑥 + 𝑦 + 𝑧
𝐿𝐻𝑆 = 3 =3 =
4 4 3

And hence the inequality becomes:


𝑥+𝑦+𝑧 4 1
≥ √ (𝑥 + 𝑦 + 𝑧)𝑥𝑦𝑧
3 3
4 1
Divide both sides by √3 (𝑥 + 𝑦 + 𝑧):
3
𝑥+𝑦+𝑧 4 4
( ) ≥ √𝑥𝑦𝑧
3
4
Raise both sides to the power 3:
𝑥+𝑦+𝑧 3
≥ √𝑥𝑦𝑧
3

Example 3.31
Let 𝑥, 𝑦, and 𝑧 be positive real numbers such that (𝑥 ∙ 𝑦) + 𝑧 = (𝑥 + 𝑧) ∙ (𝑦 + 𝑧). What is the maximum possible
value of 𝑥𝑦𝑧? (HMMT 2005/Guts/2)

Expand the given expression on the RHS:


𝑥𝑦 + 𝑧 = 𝑥𝑦 + 𝑥𝑧 + 𝑧𝑦 + 𝑧 2
𝑧 = 𝑥𝑧 + 𝑧𝑦 + 𝑧 2
Since 𝑧 ≠ 0, divide both sides by z:

5
The four variable case is proved below.

P a g e 88 | 97
Get all the files at: https://bit.ly/azizhandouts
Aziz Manva ([email protected])

𝑥+𝑦+𝑧 = 1
But we know that, by the three variable AM-GM Inequality:
𝑥+𝑦+𝑧 3
≥ √𝑥𝑦𝑧
3
Substitute 𝑥 + 𝑦 + 𝑧 = 1:
1 3
≥ √𝑥𝑦𝑧
3
Cube both sides:
1 1 1
≥ 𝑥𝑦𝑧 ⇒ 𝑥𝑦𝑧 ≤ ⇒ 𝑀𝑎𝑥(𝑥𝑦𝑧) =
27 27 27

Example 3.32
Let 𝐴, 𝑀, and 𝐶 be nonnegative integers such that 𝐴 + 𝑀 + 𝐶 = 12. What is the maximum value of 𝐴 ∙ 𝑀 ∙ 𝐶 + 𝐴 ∙
𝑀 + 𝑀 ∙ 𝐶 + 𝐴 ∙ 𝐶? (AMC 12 2000/12)

Shortcut Method
Since the expression is symmetric, make the values of the variables equal:
12
𝐴=𝑀=𝐶= =4
3
Formal Method
Note that:
(𝐴 + 1)(𝑀 + 1)(𝐶 + 1) = 𝐴𝑀𝐶 + 𝐴𝑀 + 𝑀𝐶 + 𝐴𝐶 + 𝐴 + 𝑀 + 𝐶 + 1
Substitute 𝐴 + 𝑀 + 𝐶 = 12:
𝐴𝑀𝐶 + 𝐴𝑀 + 𝑀𝐶 + 𝐴𝐶 = (𝐴 + 1)(𝑀 + 1)(𝐶 + 1) − 13
Maximizing the LHS is the same as maximizing the RHS.
3 (𝐴 + 1) + (𝑀 + 1) + (𝐶 + 1)
√(𝐴 + 1)(𝑀 + 1)(𝐶 + 1) ≤
3
Cube both sides:
𝐴+𝑀+𝐶+3 3 12 + 3 3 15 3
(𝐴 + 1)(𝑀 + 1)(𝐶 + 1) ≤ ( ) =( ) = ( ) = 53 = 125
3 3 3
Equality is achieved when:
𝐴+1=𝑀+1=𝐶+1=5⇒𝐴 =𝑀 =𝐶 =4

3.33: Manipulating the Expression


➢ You may need to manipulate the expression to get into a form where AM-GM can be applied.
➢ This manipulation may not be obvious until you have experience with such expressions.

Example 3.34
A square sheet of metal of side length 𝑠 has smaller squares of side length 𝑥 cut at each of its corners, and the
resulting “flaps” folded to form an open-top box.
A. Show that the volume of the box is 𝑉 = (𝑠 − 2𝑥)2 (𝑥).
B. Apply AM-GM on 𝑉.
C. Apply AM-GM on 4𝑉. Find the value of 𝑥 in terms of 𝑠.

P a g e 89 | 97
Get all the files at: https://bit.ly/azizhandouts
Aziz Manva ([email protected])

Part A By AM-GM Inequality:


𝑉 = (𝑠 − 2𝑥)2 (𝑥) = (𝑠 − 2𝑥)(𝑠 − 2𝑥)(𝑥) (𝑠 − 2𝑥) + (𝑠 − 2𝑥) + 4𝑥 3
≥ √(𝑠 − 2𝑥)2 (4𝑥)
3
Simplify:
2𝑠 3
≥ √(𝑠 − 2𝑥)2 (4𝑥)
3
Cube:
8𝑠 3
≥ (𝑠 − 2𝑥)2 (4𝑥)
27
Divide both sides by 4:
Part B 2𝑠 3
Apply AM-GM to the above: ≥ (𝑠 − 2𝑥)2 (𝑥)
27
(𝑠 − 2𝑥) + (𝑠 − 2𝑥) + 𝑥 3 Hence, the maximum is:
≥ √(𝑠 − 2𝑥)2 (𝑥)
3 2𝑠 3
2𝑠 − 3𝑥 3
≥ √(𝑠 − 2𝑥)2 (𝑥) 27
3 And this maximum is achieved when the three
Now if we cube the expression, the LHS is the cube
terms are equal:
of a binomial making the expression very messy. 𝑠
Part C 𝑠 − 2𝑥 = 4𝑥 ⇒ 𝑥 =
6
4𝑉 = (𝑠 − 2𝑥)(𝑠 − 2𝑥)(4𝑥)

D. AM-GM Inequality: General Case

3.35: AM-GM: Four Variables


𝑤+𝑥+𝑦+𝑧 4
≥ √𝑤𝑥𝑦𝑧
4

By AM-GM, we must have:


𝑤+𝑥 𝑦+𝑧
≥ √𝑤𝑥, ≥ √𝑦𝑧
2 2
Add the two, and then divide by two to get:
𝑤 + 𝑥 + 𝑦 + 𝑧 √𝑤𝑥 + √𝑦𝑧

⏟ 4 2
𝑰𝒏𝒆𝒒𝒖𝒂𝒍𝒊𝒕𝒚 𝑰
Apply AM-GM on the RHS one more time to get:
√𝑤𝑥 + √𝑦𝑧 4
≥ √𝑤𝑥𝑦𝑧
⏟ 2
𝑰𝒏𝒆𝒒𝒖𝒂𝒍𝒊𝒕𝒚 𝑰𝑰𝑰
And then combine Inequalities I and II to get:
𝑤+𝑥+𝑦+𝑧 4
≥ √𝑤𝑥𝑦𝑧
4

Example 3.36
Splitting terms

3.37: AM-GM: General Case

P a g e 90 | 97
Get all the files at: https://bit.ly/azizhandouts
Aziz Manva ([email protected])

𝑥1 + 𝑥2 + ⋯ + 𝑥𝑛 𝑛
≥ √𝑥1 𝑥2 … 𝑥𝑛
𝑛

One way to prove the general case of the AM-GM inequality is by forward-backward induction.
➢ Prove the base case for 𝑛 = 2. We did this above.
➢ Forward Step: Prove that if it is true for 𝑛 = 𝑘, then it is true for 𝑛 = 2𝑘
➢ Backward: Prove that if it is true for 𝑛 = 𝑘, then it is true for 𝑛 = 𝑘 − 1

A. Relation between Means

3.38: AM, HM and GM: Two Variable Case


For two positive variables 𝑎 and 𝑏:
𝐺𝑀2 = 𝐴𝑀 ∙ 𝐻𝑀

2
𝐿𝐻𝑆 = 𝐺𝑀2 = (√𝑎𝑏) = 𝑎𝑏
𝑎 + 𝑏 2𝑎𝑏
𝑅𝐻𝑆 = 𝐴𝑀 ∙ 𝐻𝑀 = ∙ = 𝑎𝑏 = 𝐿𝐻𝑆
2 𝑎+𝑏

Example 3.39
The harmonic mean of two numbers is 4. Their arithmetic mean 𝐴 and the geometric mean 𝐺 satisfy the
relation 2𝐴 + 𝐺 2 = 27. Find the two numbers. (JEE Adv. 1979)

Let the numbers be 𝑥 and 𝑦. Since their harmonic mean is 4:


2𝑥𝑦
= 4 ⇒ 2𝑥𝑦 = 4𝑥 + 4𝑦 ⇒ 𝑥𝑦
⏟ = 2𝑥 + 2𝑦
𝑥+𝑦
𝑬𝒒𝒖𝒂𝒕𝒊𝒐𝒏 𝑰
Since 2𝐴 + 𝐺 2 = 27, we must have:
2(𝑥 + 𝑦) 2
+ (√𝑥𝑦) = 27 ⇒ 𝑥 + 𝑦 + 𝑥𝑦 = 27
2
Substitute the value of 𝑥𝑦 from Equation I in the above:
𝑥 + 𝑦 + 2𝑥 + 2𝑦 = 27 ⇒ 3𝑥 + 3𝑦 = 27 ⇒ ⏟𝑥+𝑦 =9
𝑬𝒒𝒖𝒂𝒕𝒊𝒐𝒏 𝑰𝑰
𝑥𝑦 = 2(𝑥 + 𝑦) = 18 ⇒ 𝑥𝑦
⏟ = 18
𝑬𝒒𝒖𝒂𝒕𝒊𝒐𝒏 𝑰𝑰𝑰
18
Solve the system of equations. Substitute 𝑥𝑦 = 18 ⇒ 𝑦 = 𝑥
in Equation II
to get a quadratic and solve:
18
𝑥+ = 9 ⇒ 𝑥 2 − 9𝑥 + 18 = 0 ⇒ (𝑥 − 6)(𝑥 − 3) = 0 ⇒ 𝑥 ∈ {3,6}
𝑥

Challenge 3.40
𝑀𝑎𝑟𝑘 𝑎𝑙𝑙 𝑐𝑜𝑟𝑟𝑒𝑐𝑡 𝑜𝑝𝑡𝑖𝑜𝑛𝑠
Let 𝑎, 𝑏 be positive real numbers. If 𝑎, 𝐴1 , 𝐴2 , 𝑏 are in arithmetic progression, 𝑎, 𝐺1 , 𝐺2 , 𝑏 are in geometric
progression and 𝑎, 𝐻1 , 𝐻2 , 𝑏 are in harmonic progression, and let:
𝐺1 𝐺2 𝐴1 + 𝐴2 (2𝑎 + 𝑏)(𝑎 + 2𝑏)
𝑋= , 𝑌= , 𝑍=
𝐻1 𝐻2 𝐻1 + 𝐻2 9𝑎𝑏
Then:
A. 𝑋 = 𝑌
B. 𝑌 = 𝑍
C. 𝑋 = 𝑍

P a g e 91 | 97
Get all the files at: https://bit.ly/azizhandouts
Aziz Manva ([email protected])

D. None of the above (JEE Adv. 2002, Adapted)

If the geometric progression is 𝑎, 𝐺1 = 𝑎𝑟, 𝐺2 = 𝑎𝑟 2 , 𝑏 = 𝑎𝑟 3 for some common ratio 𝑟:


𝐺1 𝐺2 = (𝑎𝑟)(𝑎𝑟 2 ) = 𝑎2 𝑟 2 = 𝑎(𝑎𝑟 3 ) = 𝑎𝑏

If the arithmetic progression is 𝑎, 𝐴1 = 𝑎 + 𝑑, 𝐴2 = 𝑎 + 2𝑑, 𝑏 = 𝑎 + 3𝑑 for some common difference then:


𝐴1 + 𝐴2 = (𝑎 + 𝑑) + (𝑎 + 2𝑑) = 𝑎 + 𝑎 + 3𝑑 = 𝑎 + 𝑏

Finally, since 𝑎, 𝐻1 , 𝐻2 , 𝑏 are in harmonic progression, their reciprocals are in arithmetic progression:
1 1 1 1 1 1 1 𝑎−𝑏
, , , ⇒ 𝐶𝑜𝑚𝑚𝑜𝑛 𝐷𝑖𝑓𝑓𝑒𝑟𝑒𝑛𝑐𝑒 = 𝐷 = ( − ) =
𝑎 𝐻1 𝐻2 𝑏 3 𝑏 𝑎 3𝑎𝑏
Then:
1 1 1 𝑎 − 𝑏 3𝑏 + 𝑎 − 𝑏 2𝑏 + 𝑎 3𝑎𝑏
= +𝐷 = + = = ⇒ 𝐻1 =
𝐻1 𝑎 𝑎 3𝑎𝑏 3𝑎𝑏 3𝑎𝑏 2𝑏 + 𝑎
1 1 1 𝑎−𝑏 3𝑏 + 2𝑎 − 2𝑏 2𝑎 + 𝑏 3𝑎𝑏
= + 2𝐷 = + 2 ( )= = ⇒ 𝐻2 =
𝐻2 𝑎 𝑎 3𝑎𝑏 3𝑎𝑏 3𝑎𝑏 2𝑎 + 𝑏
Hence:
3𝑎𝑏 3𝑎𝑏 9𝑎2 𝑏 2
𝐻1 𝐻2 = × =
2𝑏 + 𝑎 2𝑎 + 𝑏 (2𝑏 + 𝑎)(2𝑎 + 𝑏)
3𝑎𝑏 3𝑎𝑏 (3𝑎𝑏)(2𝑎 + 𝑏) + (3𝑎𝑏)(2𝑏 + 𝑎) (9𝑎𝑏)(𝑎 + 𝑏)
𝐻1 + 𝐻2 = + = =
2𝑏 + 𝑎 2𝑎 + 𝑏 (2𝑏 + 𝑎)(2𝑎 + 𝑏) (2𝑏 + 𝑎)(2𝑎 + 𝑏)

𝐺1 𝐺2 𝑎𝑏 (2𝑏 + 𝑎)(2𝑎 + 𝑏) (2𝑏 + 𝑎)(2𝑎 + 𝑏)


𝑋= = 2 2 = 𝑎𝑏 ∙ = =𝑍
𝐻1 𝐻2 9𝑎 𝑏 9𝑎2 𝑏 2 9𝑎𝑏
(2𝑏 + 𝑎)(2𝑎 + 𝑏)
𝐴1 + 𝐴2 𝑎+𝑏 (2𝑏 + 𝑎)(2𝑎 + 𝑏) (2𝑏 + 𝑎)(2𝑎 + 𝑏)
𝑌= = = (𝑎 + 𝑏) ∙ = =𝑍
𝐻1 + 𝐻2 (9𝑎𝑏)(𝑎 + 𝑏) (9𝑎𝑏)(𝑎 + 𝑏) 9𝑎𝑏
(2𝑏 + 𝑎)(2𝑎 + 𝑏)

Hence:
𝑋 = 𝑌 = 𝑍 ⇒ 𝑂𝑝𝑡𝑖𝑜𝑛𝑠 𝐴, 𝐵, 𝐶

B. 𝑯𝑴 ≤ 𝑮𝑴 ≤ 𝑨𝑴 < 𝑹𝑴𝑺 Inequality

3.41: HM-GM Inequality: Two Variable Case


2𝑎𝑏
≤ √𝑎𝑏
𝑎+𝑏

Divide both sides by √𝑎𝑏:


2√𝑎𝑏
⇔ ≤1
𝑎+𝑏
Multiply both sides by 𝑎 + 𝑏:
⇔ 2√𝑎𝑏 ≤ 𝑎 + 𝑏
Square both sides:
⇔ 4𝑎𝑏 ≤ 𝑎2 + 2𝑎𝑏 + 𝑏 2
Subtract 4𝑎𝑏 from both sides:
⇔ 0 ≤ 𝑎2 − 2𝑎𝑏 + 𝑏 2
Factor the RHS:

P a g e 92 | 97
Get all the files at: https://bit.ly/azizhandouts
Aziz Manva ([email protected])

⇔ 0 ≤ (𝑎 − 𝑏)2
Which is true by the trivial inequality.
And since each of the above steps is reversible, the inequality above is true.

Example 3.42
Show that if the harmonic mean and the geometric mean of two numbers are equal, then the numbers are also
equal.

2𝑎𝑏
= √𝑎𝑏
𝑎+𝑏
Divide both sides by √𝑎𝑏:
2√𝑎𝑏
=1
𝑎+𝑏
Multiply both sides by 𝑎 + 𝑏:
2√𝑎𝑏 = 𝑎 + 𝑏
Square both sides:
4𝑎𝑏 = 𝑎2 + 2𝑎𝑏 + 𝑏 2
Subtract 4𝑎𝑏 from both sides:
0 = 𝑎2 − 2𝑎𝑏 + 𝑏 2
Factor the RHS:
0 = (𝑎 − 𝑏)2
𝑎−𝑏=0
𝑎=𝑏
Challenge 3.43
𝐶ℎ𝑜𝑜𝑠𝑒 𝑒𝑥𝑎𝑐𝑡𝑙𝑦 𝑜𝑛𝑒 𝑠𝑡𝑎𝑡𝑒𝑚𝑒𝑛𝑡 𝑝𝑒𝑟 𝑔𝑟𝑜𝑢𝑝
Let 𝐴1 , 𝐺1 , 𝐻1 denote the arithmetic, geometric and harmonic means, respectively, of two distinct positive
numbers. For 𝑛 ≥ 2, Let 𝐴𝑛–1 and 𝐻𝑛–1 have arithmetic, geometric, and harmonic means as 𝐴𝑛 , 𝐺𝑛 , 𝐻𝑛
respectively.

Group I
A.
B.
C.
D. 𝐺1 < 𝐺3 < 𝐺5 < ⋯ 𝑎𝑛𝑑 𝐺2 > 𝐺4 > 𝐺6 > ⋯

Group II
A.
B.
C.
D. 𝐴1 < 𝐴3 < 𝐴5 < ⋯ 𝑎𝑛𝑑 𝐴2 > 𝐴4 > 𝐴6 > ⋯

Group III
A.
B.
C.
D. 𝐻1 < 𝐻3 < 𝐻6 < ⋯ 𝑎𝑛𝑑 𝐻2 > 𝐻4 > 𝐻6 > ⋯ (JEE Adv. 2007)

Let the numbers be:


𝑎 𝑎𝑛𝑑 𝑏 ⇒ 𝑎 < 𝐻1 < 𝐺1 < 𝐴1 < 𝑏

P a g e 93 | 97
Get all the files at: https://bit.ly/azizhandouts
Aziz Manva ([email protected])

Group II
𝐴2 = 𝐴𝑀(𝐻1 , 𝐴1 ) lies between 𝐻1 and 𝐴1
Similarly, 𝐴3 = 𝐴𝑀(𝐻2 , 𝐴2 ) lies between 𝐻2 and 𝐴2
In general:
𝐴1 > 𝐴2 > 𝐴3 ⇒ 𝑂𝑝𝑡𝑖𝑜𝑛 𝐴
Group III
𝐻2 = 𝐻𝑀(𝐻1 , 𝐴1 ) lies between 𝐻1 and 𝐴1
Similarly, 𝐻3 = 𝐻𝑀(𝐻2 , 𝐴2 ) lies between 𝐻2 and 𝐴2
In general:
𝐻1 < 𝐻2 < 𝐻3 ⇒ 𝑂𝑝𝑡𝑖𝑜𝑛 𝐵
Group I

𝐺12 = 𝑎𝑏 = 𝐴1 𝐻1
𝐺𝑛2 = 𝐴𝑛 𝐻𝑛
𝐺𝑛2 = 𝐴𝑛−1 𝐻𝑛−1

𝐴𝑛 𝐻𝑛 = 𝐴𝑛−1 𝐻𝑛−1

𝐴𝑛 𝐻𝑛 = 𝐴𝑛−1 𝐻𝑛−1 = ⋯ = 𝐴1 𝐻1 = 𝑎𝑏

𝐺𝑛2 = 𝐴𝑛 𝐻𝑛 = 𝑎𝑏 ⇒ 𝑂𝑝𝑡𝑖𝑜𝑛 𝐶

3.44: Root Mean Square


The root mean square of 𝑛 numbers is the square root of the arithmetic mean of the squares of the 𝑛 numbers.

Example 3.45
Find

3.46: AM-RMS Inequality: Two Variable Case


𝑎+𝑏 𝑎2 + 𝑏 2
≤√
2 2

𝑎2 + 2𝑎𝑏 + 𝑏 2 𝑎2 + 𝑏 2
⇔ ≤
4 2
⇔ 𝑎2 + 2𝑎𝑏 + 𝑏 2 ≤ 2𝑎2 + 2𝑏 2
⇔ 0 ≤ 𝑎2 − 2𝑎𝑏 + 𝑏 2
⇔ 0 ≤ (𝑎 − 𝑏)2
Which is true by the trivial inequality.
And since each of the above steps is reversible, the inequality above is true.

3.47: Cascading Inequalities: 𝑯𝑴 ≤ 𝑮𝑴 ≤ 𝑨𝑴 ≤ 𝑹𝑴𝑺


2𝑎𝑏 𝑎+𝑏 𝑎2 + 𝑏 2
≤ √𝑎𝑏 ≤ ≤√
𝑎+𝑏 2 2

P a g e 94 | 97
Get all the files at: https://bit.ly/azizhandouts
Aziz Manva ([email protected])

Example 3.48
The harmonic mean of 75, 85, 65 is between
A. 65 to 70
B. 70 to 75
C. 75 to 80
D. 80 to 85

AM = middle value = 75.


HM has to be less than 75, and can be approximated to be not so far away from AM6.
𝑜𝑝𝑡𝑖𝑜𝑛 𝐵

C. Minkowski Inequality

3.49: Minkowski Inequality: 𝒏 = 𝟐, 𝒑 = 𝟐


√𝑎12 + 𝑎22 + √𝑏12 + 𝑏22 ≥ √(𝑎1 + 𝑏1 )2 + (𝑎1 + 𝑏1 )2

(Calculator) Example 3.50


A main gas line runs through 𝑃 and Q. From some point 𝑇 on 𝑃𝑄, a supply line runs to a house at point 𝑀. A
second supply line from 𝑇 runs to a house at point 𝑁. What is the minimum total length of pipe required for the
two supply lines? (CEMC Cayley 1999/22)

Drop a perpendicular from N to MP. Then,


𝑀𝑅 = 105 − 55 = 50
By a Pythagorean Triplet
𝑁𝑅 = 120

Reflect 𝑁 across 𝑃𝑄 to get S.


𝑁𝑄 = 𝑄𝑆, ∠𝑇𝑄𝑁 = ∠𝑇𝑄𝑆 ⇒ Δ𝑇𝑄𝑁 ≅ 𝑇𝑄𝑆 ⇒ 𝑇𝑁 = 𝑇𝑆

We need to minimize
𝑀𝑇 + 𝑇𝑁 = 𝑀𝑇 + 𝑇𝑆
Which will get minimized when 𝑀, 𝑇 and S are collinear.

6 3 71
Actual value of 𝐻𝑀(65, 75, 85) = 1 1 1 = 74 is not a good expression to work with.
+ + 671
65 75 85

P a g e 95 | 97
Get all the files at: https://bit.ly/azizhandouts
Aziz Manva ([email protected])

𝑀𝑆 = √(105 + 55)2 + 1202 = 200

Example 3.51
A main gas line runs through 𝑃 and Q. From some point 𝑇 on 𝑃𝑄, a supply line runs to a house at point 𝑀. A
second supply line from 𝑇 runs to a house at point 𝑁. What is the minimum total length of pipe required for the
two supply lines? (CEMC Cayley 1999/22)

𝑃𝑖𝑝𝑒 𝐿𝑒𝑛𝑔𝑡ℎ = 𝑁𝑇 + 𝑀𝑇 = √552 + 𝑥 2 + √1052 + (120 − 𝑥)2

√𝑎12 + 𝑎22 + √𝑏12 + 𝑏22 ≥ √(𝑎1 + 𝑏1 )2 + (𝑎1 + 𝑏1 )2

√(𝑎1 + 𝑏1 )2 + (𝑎1 + 𝑏1 )2 = √(55 + 105)2 + (𝑥 + 120 − 𝑥)2


= √1602 + 1202 = 200

3.52: Minkowski Inequality: 𝒑 = 𝟐


𝑛 𝑛 𝑛

√∑ 𝑎𝑖2 + √∑ 𝑏𝑖2 ≥ √∑(𝑎𝑖 + 𝑏𝑖 )2


𝑖=1 𝑖=1 𝑖=1

3.53: Minkowski Inequality: General Case


𝑝
𝑛 𝑝
𝑛 𝑝
𝑛

√∑ 𝑎𝑖𝑝 + √∑ 𝑏𝑖𝑝 ≥ √∑(𝑎1 + 𝑏1 )2


𝑖=1 𝑖=1 𝑖=1

D. Jensen’s Inequality

3.54: Jensen’s Inequality


If 𝑓(𝑥) is a twice differentiable function that is concave over an interval then for points 𝑥1 , 𝑥2 , … 𝑥𝑛 in that
interval:

P a g e 96 | 97
Get all the files at: https://bit.ly/azizhandouts
Aziz Manva ([email protected])

𝑓(𝑥1 ) + 𝑓(𝑥2 ) + ⋯ + 𝑓(𝑥𝑛 ) 𝑥1 + 𝑥2 + ⋯ + 𝑥𝑛


≤ 𝑓( )
𝑛 𝑛

Example 3.55
𝐴 𝐵 𝐶
If 𝐴 + 𝐵 + 𝐶 = 𝜋, then find the maximum value of sin 2 sin 2 sin 2 .

𝑥
𝑥 ′
1 cos 2 1 𝑥 1 𝑥
𝑓(𝑥) = log (sin ) ⇒ 𝑓 = 𝑥 = cot ⇒ 𝑓 ′′ = − csc 2
2 2 sin 2 2 4 2
2
𝑥
Note that csc 2 2 is a perfect square and hence positive. This means that:

𝑥 𝑥 2 1 𝑥
csc 2 = (csc ) > 0 ⇒ 𝑓 ′′ = − csc 2 < 0 ⇒ 𝑓 𝑖𝑠 𝑐𝑜𝑛𝑐𝑎𝑣𝑒
2 2 4 2
Hence, by Jensen’s Inequality:
𝐴 𝐵 𝐶 𝐴 𝐵 𝐶
𝑓( )+𝑓( )+𝑓( )
2 2 2 ≤ 𝑓 (2 + 2 + 2)
3 3

𝑥
Apply 𝑓(𝑥) = log (sin ):
2

𝐴 𝐵 𝐶
log sin ( ) + log sin ( ) + log sin ( )
2 2 2 ≤ 3 log (sin 𝐴 + 𝐵 + 𝐶 )
3 3
Combine the LHS using log 𝑎 + log 𝑏 + log 𝑐 = log 𝑎𝑏𝑐, and multiply both sides by 3:
𝐴 𝐵 𝐶 𝜋
log (sin sin sin ) ≤ 3 log (sin )
2 2 2 6
𝜋 1 1
Since 𝑅𝐻𝑆 = 3 log (sin ( 6 )) = 3 log 2 = log 8:

𝐴 𝐵 𝐶 1
log (sin sin sin ) ≤ log
2 2 2 8
Taking anti-logs gives us the maximum value:
𝐴 𝐵 𝐶 1
sin sin sin ≤
2 2 2 8

3.3 Further Topics


56 Examples

P a g e 97 | 97

You might also like